Problemas Cap 3 Todos

Post on 29-Nov-2014

1.458 views 2 download

description

Exercícios resolvidos do cap. 3 Sakurai -Mec. Quantica

Transcript of Problemas Cap 3 Todos

1

Problema 3.1

Encontre os autovalores e os autovetores de 00y

ii

σ−⎛ ⎞

= ⎜ ⎟⎝ ⎠

. Suponha que o

elétron está no estado de spin αβ⎛ ⎞⎜ ⎟⎝ ⎠

. Se yS é medido, qual é a probabilidade

do resultado 2

?

Solução: O vetor de estado pode ser escrito como:

a bψ α β

αψ

β

= +

⎛ ⎞= ⎜ ⎟⎝ ⎠

........................................................................................................................... Lembre-se:

∑ +=='

''a

bbaaaa ψψψψ

........................................................................................................................... ........................................................................................................................... Lembre-se:

⎟⎟⎠

⎞⎜⎜⎝

⎛−+

=αα

α . (3.2.27a) ⎟⎟⎠

⎞⎜⎜⎝

⎛−+

=αα

χ (3.2.28)

........................................................................................................................... Para os autovalores temos:

( )det 0y Iσ λ− =

2

........................................................................................................................... Lembre-se:

⎟⎟⎠

⎞⎜⎜⎝

⎛ −=

00

2 ii

S y . (1.4.18b)

Portanto:

yyS σ2

= .

........................................................................................................................... Temos ainda que:

0

00

y

ii o

I

σ

λλ

λ

−⎛ ⎞= ⎜ ⎟⎝ ⎠

⎛ ⎞= ⎜ ⎟⎝ ⎠

Substituindo, temos:

2 2

0 0det 0

0 0

det 0

0

ii

ii

i

λλ

λλ

λ

⎡ ⎤−⎛ ⎞ ⎛ ⎞− =⎢ ⎥⎜ ⎟ ⎜ ⎟

⎝ ⎠ ⎝ ⎠⎣ ⎦

⎡ ⎤− −⎛ ⎞=⎢ ⎥⎜ ⎟−⎝ ⎠⎣ ⎦

+ =

Os autovalores são:

1 1λ = e 2 1λ = −

3

Para os autovetores, temos:

( ) 0

y

y

σ ψ λ ψ

σ λ ψ

=

− =

Substituindo, temos:

0

00

0

ii

ii

i

i

λ αλ β

λα βα λβ

λα β

βαλ

− −⎛ ⎞⎛ ⎞=⎜ ⎟⎜ ⎟−⎝ ⎠⎝ ⎠

− − =⎧⎨ − =⎩

− − =

= −

De acordo com a condição de normalização, temos:

2 2 1α β+ = .

4

Substituindo, temos:

( )

( )

( )

22

22

2

2 2 2 2

2 2 2

22

2

22

2

1

1

1

1

1

iβ βλ

β βλ

β λ β λ

β λ λ

λβλ

λβλ

− + =

+ =

+ =

+ =

=+

=+

Para α , temos:

( )2

21i λαλ λ

= −+

Substituindo 1λ = − , temos:

2

12

β

=

=

5

Substituindo 1λ = , temos:

2

12

β

= −

=

Portanto,

112i

ψ+

−⎛ ⎞= ⎜ ⎟

⎝ ⎠ 1=λ

1+=+ ⎟⎟

⎞⎜⎜⎝

⎛=

λβα

ψ

e

112i

ψ−

⎛ ⎞= ⎜ ⎟

⎝ ⎠ 1−=λ

1−=− ⎟⎟

⎞⎜⎜⎝

⎛=

λβα

ψ

........................................................................................................................... Lembre-se:

( )( ) −−

++

−==

+==

ψλψσ

ψλψσ

1

1

y

y

........................................................................................................................... Imagine que o sistema esta em um estado ψ . Qual é a probabilidade ou amplitude de transição para o sistema ser achado em +ψ quando yS é medido?

A probabilidade de que o elétron esteja no estado de spin αψ

β⎛ ⎞

= ⎜ ⎟⎝ ⎠

, se yS é

medido, pode ser escrita como:

2

yP Sψ ψ+= .

6

Temos ainda que:

⎟⎟⎠

⎞⎜⎜⎝

⎛ −=

00

2 ii

S y

Substituindo,

( )

( )

2

2

01 1022

12 2

iP i

i

P i

αβ

αβ

−⎛ ⎞⎛ ⎞= ⎜ ⎟⎜ ⎟

⎝ ⎠⎝ ⎠

⎛ ⎞= ⎜ ⎟

⎝ ⎠

temos a probabilidade de que o elétron seja achado em +ψ com autovalor

2/+ quando yS é medido:

( )2

22

2 2

8

P i

P i

α β

α β

= +

= +

.

1

Problema 3.10 a. Prove que a evolução temporal do operador densidade ρ (no quadro de Schrödinger) é dado por

†0 0 0( ) ( , ) ( ) ( , )t U t t t U t tρ ρ= .

b. Suponha que nós temos um conjunto puro em 0t = . Prove que ele não pode evoluir em um conjunto misto quando a evolução temporal é governada pela equação de Schrödinger. Solução: a. O operador densidade é definido por:

( ) ( )i ii

iwρ α α=∑ .

Para acharmos a evolução temporal de ρ devemos evoluir os kets e os brás destes estados:

( ) ( )0 0

( ) ( ) †0 0

, ; ( , )

, ; ( , )

i i

i i

t t U t t

t t U t t

α α

α α

=

=

.

Substituindo, temos:

( )

( ) ( )0 0

( ) ( ) †0 0

( ) ( ) †0 0

†0 0 0

( ) , ; , ;

( ) ( , ) ( , )

( ) ( , ) ( , )

( ) ( , ) ( , )

i ii

i

i ii

i

i ii

i

t w t t t t

t wU t t U t t

t U t t w U t t

t U t t t U t t

ρ α α

ρ α α

ρ α α

ρ ρ

=

=

⎛ ⎞= ⎜ ⎟

⎝ ⎠=

2

b. A função densidade para o estado puro pode ser escrito como: ρ α α= . Do item (a), temos que:

( ) †0 0 0

†0 0

0 0

( ) ( , ) ( , )

( ) ( , ) ( , )

( ) , ; , ;

t U t t t U t t

t U t t U t t

t t t t t

ρ ρ

ρ α α

ρ α α

=

=

=

A expressão acima ainda está mostrando que este é um estado puro. Podemos checar esta afirmação.

20 0 0 0

20 0

2

( ) , ; , ; , ; , ;

( ) , ; , ;

( ) ( )

t t t t t t t t t

t t t t t

t t

ρ α α α α

ρ α α

ρ ρ

=

=

=

e

( ) 1Tr tρ =

1

Problema 3.11 Considere um conjunto de sistemas de spin 1. A matriz densidade é agora uma matriz 3 3× . Quantos parâmetros reais independentes são necessários para caracterizar a matriz densidade? O que nós devemos conhecer em adição a [ ]xS , yS⎡ ⎤⎣ ⎦ e [ ]zS para caracterizar o conjunto completamente? Solução : Da equação (3.4.9),

∑=i

iii bbwbb '''''' )()( ααρ , (1)

podemos escrever a matriz densidade como

⎟⎟⎟

⎜⎜⎜

⎛=

fecedbcba

***ρ . (2)

Como a matriz densidade ρ é Hermitiana,

+= ρρ , (3) temos que a , d e f são reais, enquanto b , c e e devem ser complexos. Portanto, devemos ter uma matriz da forma:

⎟⎟⎟

⎜⎜⎜

−=−=+=−=+=+=

=fieeeiccc

ieeedibbbicccibbba

2121

2121

2121

***ρ . (4)

Logo, temos 9 variáveis independentes: a , d , f , 1b , 2b , 1c , 2c , 1e e 2e . No entanto, temos ainda da equação (3.4.11),

1=ρTr , (5)

2

ou seja,

1=++ fda . (6) Portanto, 8 parâmetros independentes são necessários para caracterizar a matriz densidade. Se conhecermos [ ]xS , [ ]yS e [ ]zS , então necessitaremos de apenas 5 quantidade independentes. ........................................................................................................................... Problema 3.9:

⎟⎟⎠

⎞⎜⎜⎝

⎛=

dcba

ρ .

A média de um conjunto de um operador A é [ ] [ ]ATrA ρ= . Calculando os valores médios:

[ ] ( )cbcdab

Trdcba

TrSx +=⎟⎟⎠

⎞⎜⎜⎝

⎛=⎥

⎤⎢⎣

⎡⎟⎟⎠

⎞⎜⎜⎝

⎛⎟⎟⎠

⎞⎜⎜⎝

⎛=

220110

2

[ ] ( )cbiicidiaib

Tri

idcba

TrS y −=⎟⎟⎠

⎞⎜⎜⎝

⎛−−

=⎥⎦

⎤⎢⎣

⎡⎟⎟⎠

⎞⎜⎜⎝

⎛ −⎟⎟⎠

⎞⎜⎜⎝

⎛=

2200

2

[ ] ( )dadcba

Trdcba

TrS z −=⎟⎟⎠

⎞⎜⎜⎝

⎛−−

=⎥⎦

⎤⎢⎣

⎡⎟⎟⎠

⎞⎜⎜⎝

⎛−⎟⎟

⎞⎜⎜⎝

⎛=

221001

2

...........................................................................................................................

3

Para [ ]xS , [ ]yS e [ ]zS , temos:

[ ]

[ ]

[ ]

[ ]

[ ] )(2

)22(2

*)*(2

****

2

010101010

2***

11

11

ebS

ebS

eebbS

efcedebdbcab

trS

fecedbcba

trS

x

x

x

x

x

+=

+=

+++=

⎥⎥⎥

⎢⎢⎢

+++

=

⎥⎥⎥

⎢⎢⎢

⎥⎥⎥

⎢⎢⎢

⎡=

(7)

........................................................................................................................... Lembre-se: Para sistemas de spin 1 as matrizes xS , yS e zS são:

⎟⎟⎟

⎜⎜⎜

⎛=

010101010

2xS

⎟⎟⎟

⎜⎜⎜

⎛−

−=

000

00

2 iii

iS y

⎟⎟⎟

⎜⎜⎜

−=

100000001

zS

...........................................................................................................................

4

[ ]

[ ]

[ ]

[ ][ ] )(2

)(22

*)*(2

****

2

000

00

2***

11

11

ebiS

ebiS

ieieibibS

ieificieidieibidibiciaib

trS

iii

i

fecedbcba

trS

y

y

y

y

y

+=

+=

−+−=

⎥⎥⎥

⎢⎢⎢

−+−−+−−+−

=

⎥⎥⎥

⎢⎢⎢

⎡−

⎥⎥⎥

⎢⎢⎢

⎡=

(8)

[ ]

[ ]

[ ] ( )[ ] [ ])()(

0*0*0

100000001

***

2211 faifaSfaS

fcebca

trS

fecedbcba

trS

z

z

z

z

−+−=−=

⎥⎥⎥

⎢⎢⎢

−−−

=

⎥⎥⎥

⎢⎢⎢

−⎥⎥⎥

⎢⎢⎢

⎡=

(9)

As outras quantidades necessárias são: [ ]yxSS , [ ]zy SS , [ ]xz SS , , [ ]2

xS e [ ]2yS . (10)

Podemos calcular estas quantidades utilizando a equação (3.4.10): [ ] AtrA ρ= . (11)

5

Para o caso de [ ]yxSS , temos:

[ ]

[ ]

[ ]

[ ]

[ ] )(2

)*0(2

*0**0*

0

2

0000

0

***

2

000

00

2010101010

***

21212121

2

2

2

2

fifciccicaiaSS

ificiciaSS

ificificieibieibiciaicia

SS

ii

ii

fecedbcba

trSS

iii

i

fecedbcba

trSS

yx

yx

yx

yx

yx

+−−−−+−=

−−++=

⎟⎟⎟

⎜⎜⎜

−−+−−+−−+

=

⎥⎥⎥

⎢⎢⎢

⎟⎟⎟

⎜⎜⎜

⎟⎟⎟

⎜⎜⎜

⎛=

⎥⎥⎥

⎢⎢⎢

⎟⎟⎟

⎜⎜⎜

⎛−

⎟⎟⎟

⎜⎜⎜

⎟⎟⎟

⎜⎜⎜

⎛=

. (12)

Para o caso de [ ]zy SS , temos:

[ ]

[ ]

[ ]

[ ]

[ ] )(2

*)(2

*0*00

2

0000

000

***

2

100000001

000

00

2***

2121

2

2

2

2

eiebibSS

ieibSS

ieieididibib

SS

iifecedbcba

trSS

iii

i

fecedbcba

trSS

zy

zy

zy

zy

zy

++−=

+=

⎟⎟⎟

⎜⎜⎜

⎛=

⎥⎥⎥

⎢⎢⎢

⎟⎟⎟

⎜⎜⎜

⎟⎟⎟

⎜⎜⎜

⎛=

⎥⎥⎥

⎢⎢⎢

⎟⎟⎟

⎜⎜⎜

−⎟⎟⎟

⎜⎜⎜

⎛−

⎟⎟⎟

⎜⎜⎜

⎛=

(13)

6

Para o caso de [ ]xz SS , temos:

[ ]

[ ]

[ ]

[ ]

[ ] )(2

)*(2

0*00*000

2

010000010

***

2

010101010

2100000001

***

2121

2

2

2

2

ieeibbSS

ebSS

fcebca

SS

fecedbcba

trSS

fecedbcba

trSS

xz

xz

xz

xz

xz

−−−=

−=

⎟⎟⎟

⎜⎜⎜

−−−

=

⎥⎥⎥

⎢⎢⎢

⎟⎟⎟

⎜⎜⎜

−⎟⎟⎟

⎜⎜⎜

⎛=

⎥⎥⎥

⎢⎢⎢

⎟⎟⎟

⎜⎜⎜

⎟⎟⎟

⎜⎜⎜

−⎟⎟⎟

⎜⎜⎜

⎛=

(14)

Para o caso de [ ]2

xS , temos:

[ ]

[ ]

[ ]

[ ]

[ ]

[ ] [ ])2()22(2

)22(2

)*2(2

**2**2*

2

2

101020101

***

2

010101010

2010101010

2***

2221111

22

2121212121

22

22

22

22

2

fdaifdcaS

ifficcdidicciaaS

fcdcaS

fcefcebdebcabca

S

fecedbcba

trS

fecedbcba

trS

x

x

x

x

x

x

−+++++=

−+−++++++=

++++=

⎟⎟⎟

⎜⎜⎜

++++++

=

⎥⎥⎥

⎢⎢⎢

⎟⎟⎟

⎜⎜⎜

⎟⎟⎟

⎜⎜⎜

⎛=

⎥⎥⎥

⎢⎢⎢

⎟⎟⎟

⎜⎜⎜

⎟⎟⎟

⎜⎜⎜

⎟⎟⎟

⎜⎜⎜

⎛=

. (15)

7

Para o caso de [ ]2yS , temos:

[ ]

[ ]

[ ]

[ ]

[ ]

[ ] ( ) ( )[ ]2221111

22

2121212121

22

22

22

22

2

2222

)22(2

)*2(2

**2**2*

2

2

101020101

***

2

000

00

2000

00

2***

fdaifdcaS

ifficcdidicciaaS

fcdcaS

fcefcebdebcabca

trS

fecedbcba

trS

iii

i

iii

i

fecedbcba

trS

y

y

y

y

y

y

+++−+−=

+−+−++−−+=

+−+−=

⎟⎟⎟

⎜⎜⎜

+−−+−−+−−

=

⎥⎥⎥

⎢⎢⎢

⎟⎟⎟

⎜⎜⎜

⎟⎟⎟

⎜⎜⎜

⎛=

⎥⎥⎥

⎢⎢⎢

⎟⎟⎟

⎜⎜⎜

⎛−

⎟⎟⎟

⎜⎜⎜

⎛−

⎟⎟⎟

⎜⎜⎜

⎛=

. (16) Temos 9 equações envolvendo os elementos de ρ : ( ) 1)( 222111 =+++++ fdaifda [ ] )(2 11 ebSx += [ ] )(2 11 ebiS y += [ ] [ ])()( 2211 faifaS z −+−=

[ ] )(2 21212121

2

fifciccicaiaSS yx +−−−−+−=

[ ] )(2 2121

2

ieeibbSS xz +−−=

[ ] )(2 2121

2

eiebibSS zy ++−=

8

[ ] [ ])2()22(2 2221111

2 fdaifdcaSx −+++++=

[ ] ( ) ( )[ ]2221111

2 2222

fdaifdcaS y +++−+−=

Resolvendo as 9 equações acima, seremos capazes de determinar os 9 parâmetros, que são: a , d , f , 1b , 2b , 1c , 2c , 1e e 2e .

1

Problema 3.12 Um autoestado de momento angular , maxj m m j= = é rotacionado por um ângulo infinitesimal ε em torno do eixo- y . Sem usar a forma explicita da função ( )

'j

m md , obtenha uma expressão para a probabilidade para o novo estado rotacionado ser achado no estado original até termos de ordem 2ε . Solução : Um estado rotacionado é dado por:

jmjJiJ

jmjU yyR =

⎥⎥⎦

⎢⎢⎣

⎡+−−== ,...

21, 2

22εε (1)

A amplitude de probabilidade para o sistema ser achado no estado original pode ser escrita como:

jmjUjmj R == ,, , (2) enquanto, a probabilidade pode ser estrita da forma:

2, ,RP j m U j m= . (3)

Da expressão acima percebemos que necessitamos dos valores esperados de

yJ e 2yJ . Estes operadores podem ser escritos como:

2yJ JJ

i+ −−

= (3.5.5)

[ ]+−−+−+ −−+−= JJJJJJJ y

222

41 (4)

2

Evidentemente, termos do tipo

0,

== jmjyJ , (5)

quando considerado as equações (3.5.39) e (3.5.40).

( )( )

( )( )

, 1 , 1

, 1 , 1

J j m j m j m j m

J j m j m j m j m

+

= − + + +

= + − + −

.

Por outro lado, termos envolvendo o operador 2

yJ , fornecerão os seguintes valores:

( )4

2,,41 2

,

2 jjmjJJjmjJjmjy ==== −+=

(6)

considerando que jm =max . A amplitude de probabilidade se torna:

41...

4221,,

22

2

2 εε jjjmjUjmj R −=+−=== (7)

Portanto, a probabilidade para termos de até 2ε , pode agora ser calculada:

( )

( ) 22

222

22

211

41

41

,,)(

εε

εεε

ε

jP

jjP

jmjUjmjP R

−=

⎟⎟⎠

⎞⎜⎜⎝

⎛−⎟⎟

⎞⎜⎜⎝

⎛−=

===

. (8)

1

Problema 3.13 Mostre que as matrizes 3 3x ( 1, 2,3)iG i = , cujos elementos são dados por ( )i ijkjkG i ε= − ,

onde j e k são os índices das linhas e das colunas, satisfazem as relações de comutação de momento angular. Qual é o significado físico (ou geométrico) da matriz transformação que conecta iG as representações mais usuais 3 3x do operador momento angular iJ , com 3J considerado diagonal. Relacione o seu resultado ao

ˆV V n Vδφ→ + × sob rotações infinitesimais. (Nota: Este problema pode ser útil na compreensão do spin do fóton.) Solução: Olhando para os elementos matriciais, temos:

( ) ( ) ( ) ( )ln ln

ln

,

,

i j i j j i

i j i j j ilm mnmn lm

G G G G G G

G G G G G G

⎡ ⎤ ⎡ ⎤= −⎣ ⎦ ⎣ ⎦

⎡ ⎤ = −⎣ ⎦ (1)

2

........................................................................................................................... Lembre-se:

i j i jn l

i j i jn l m

i j i jm n l

l G G n l l G G n n

l G G n l l G m m G n n

l G G n l l G m m G n n

=

⎛ ⎞= ⎜ ⎟

⎝ ⎠

=

∑ ∑

∑ ∑ ∑

∑ ∑ ∑

(2)

...........................................................................................................................

( ) ( ) ( ) ( )ln

2

ln

2

ln

,

,

,

i j i j j ilm mnmn lm

i j ilm jmn jlm imn

i j mil mnj mjl mni

G G G G G G

G G

G G

ε ε ε ε

ε ε ε ε

⎡ ⎤ = −⎣ ⎦

⎡ ⎤ ⎡ ⎤= − −⎣ ⎦ ⎣ ⎦

⎡ ⎤ ⎡ ⎤= − −⎣ ⎦ ⎣ ⎦

(3)

........................................................................................................................... Demonstração: Para duas permutações, temos que ε é positivo.

ilm iml mil

jmn mjn mnj

ε ε ε

ε ε ε

→ →

→ → (4)

3

min

jlm jml mjl

imn mni

ε ε ε

ε ε ε

→ →

→ → (5)

...........................................................................................................................

( ) ( )

2

ln

2ln lnln

,

,

i j mil mnj mjl mni

i j in lj ij jn li ji

G G

G G

ε ε ε ε

δ δ δ δ δ δ δ δ

⎡ ⎤ ⎡ ⎤= − −⎣ ⎦ ⎣ ⎦

⎡ ⎤⎡ ⎤ = − − − −⎣ ⎦ ⎣ ⎦ (6)

Vamos considerar agora a seguinte relação:

ln

ln

mil mnj in lj ij

mjl mni jn li ji

ε ε δ δ δ δ

ε ε δ δ δ δ

= −

= − (7)

........................................................................................................................... Lembre-se: Em três dimensões, o símbolo de Levi-Civita é definido como:

11

0ijkε

+⎧⎪= −⎨⎪⎩

(8)

em que ijkε é +1 para permutação par, -1 para permutação ímpar e 0 se algum índice for repetido.

4

Relação com o Delta de Kronecker O símbolo de Levi-Civita está relacionado ao delta de Kronecker. Em três dimensões, esta relação pode ser escrita como:

( ) ( ) ( )

det

det

il im in

ijk lmn jl jm jn

kl km kn

il im in il im

ijk lmn jl jm jn jl jm

kl km kn kl km

ijk lmn il jm kn jn km im jl kn jn kl in jm kl jl km

δ δ δε ε δ δ δ

δ δ δ

δ δ δ δ δε ε δ δ δ δ δ

δ δ δ δ δ

ε ε δ δ δ δ δ δ δ δ δ δ δ δ δ δ δ

⎡ ⎤⎢ ⎥= ⎢ ⎥⎢ ⎥⎣ ⎦

⎡ ⎤⎢ ⎥= ⎢ ⎥⎢ ⎥⎣ ⎦

= − − − − −

(9)

Considerando l i= , e fazendo o somatório, temos:

( ) ( ) ( )ijk imn ii jm kn jn km im ji kn jn ki in jm il ji kmi iε ε δ δ δ δ δ δ δ δ δ δ δ δ δ δ δ= − − − − −∑ ∑ . (10)

Considerando também que se i , j e k forem iguais, os termos serão nulos (da mesma forma para i , m e n ), então temos:

( )ijk imn jm kn jn kmi i

ε ε δ δ δ δ= −∑ ∑ (11)

Também, de acordo com a notação de Einstein, o símbolo do somatório pode ser omitido, ficando a seguinte expressão:

ijk imn jm kn jn kmε ε δ δ δ δ= − (12) ...........................................................................................................................

5

........................................................................................................................... Demonstração: Podemos testar também esta relação.

ln

ln

mil mnj in lj ij

mjl mni jn li ji

ε ε δ δ δ δ

ε ε δ δ δ δ

= −

= − (13)

Vamos considerar três situações em particular: I) Três índices iguais: 1m = , 1i = , 1j = , 1l = e 1n = Nesta situação temos:

ln

111 111 11 11 11 11

0 0

mil mnj in lj ijε ε δ δ δ δ

ε ε δ δ δ δ

= −

= −

=

(14)

II) Dois índices iguais: 1m = , 1i = , 3j = , 3l = e 2n = Nesta situação temos:

6

ln

113 123 12 33 13 32

0 0

mil mnj in lj ijε ε δ δ δ δ

ε ε δ δ δ δ

= −

= −

=

(15)

III) Três índices diferentes: 1m = , 2i = , 3j = , 3l = e 2n = Nesta situação temos:

ln

123 123 22 33 23 32

1 1

mil mnj in lj ijε ε δ δ δ δ

ε ε δ δ δ δ

= −

= −

=

(16)

Portanto, vale a relação acima. ........................................................................................................................... Voltando a relação principal, temos:

( ) ( )( )

( )( )

2ln lnln

2

ln

2lnln

lnln

lnln

,

,

,

,

,

i j in lj ij jn li ji

i j il jn in jl

i j kij k

i j ijk k

i j ijk k

G G

G G

G G

G G i i

G G i G

δ δ δ δ δ δ δ δ

δ δ δ δ

ε ε

ε ε

ε

⎡ ⎤⎡ ⎤ = − − − −⎣ ⎦ ⎣ ⎦⎡ ⎤ = −⎣ ⎦

⎡ ⎤ =⎣ ⎦

⎡ ⎤ = −⎣ ⎦

⎡ ⎤ =⎣ ⎦

(17)

7

Portanto,

,i j ijk kG G i Gε⎡ ⎤ =⎣ ⎦ . (18) A forma explicita de 3G (a partir de ( )i ijkjk

G i ε= − , onde j e k são os índices correspondentes a linhas e colunas) é dada por:

3

0 1 01 0 00 0 0

G i−⎛ ⎞

⎜ ⎟= ⎜ ⎟⎜ ⎟⎝ ⎠

. (19)

........................................................................................................................... Demonstração: Índice 3i = Linha 2: 2j = Coluna 1: 1k = ( )i ijkjkG i ε= − (20)

Temos 3 permutações para ε :

321 312 132 123ε ε ε ε→ → → (21) Portanto, ( )i ijkjkG i ε= − ( )3 32121

( 1)G i i iε= − = − − = (22) ...........................................................................................................................

8

Os autovalores e autovetores são obtidos a partir da equação ( )3 0G I rλλ− = , (23) em que λ é a raiz de

3 0G Iλ− = . (24) Os autovalores e autovetores ortonormais podem ser imediatamente mostrados. Para o cálculo dos autovalores, temos:

3 2

0et 0 0

0 0

00 0

0 0 0 0

0

iD i

i iDet i i

λλ

λ

λ λλ λ

λ

λ λ

− −⎛ ⎞⎜ ⎟− =⎜ ⎟⎜ ⎟−⎝ ⎠

− − − −⎛ ⎞⎜ ⎟− − =⎜ ⎟⎜ ⎟−⎝ ⎠

− + =

(25)

As raízes são:

0λ = , λ = + e λ = − . (26)

9

Para o cálculo dos autovetores, temos para 0λ = :

( )1 1

2 2

3 3

2

1

0 00 0 0

0 0 0

00

0

i x xi x x

x x

i xi x

λ−⎛ ⎞⎛ ⎞ ⎛ ⎞

⎜ ⎟⎜ ⎟ ⎜ ⎟= =⎜ ⎟⎜ ⎟ ⎜ ⎟⎜ ⎟⎜ ⎟ ⎜ ⎟⎝ ⎠⎝ ⎠ ⎝ ⎠

− =⎧⎪ =⎨⎪⎩

(27)

Normalizando,

0

00rc

⎛ ⎞⎜ ⎟= ⎜ ⎟⎜ ⎟⎝ ⎠

(28)

temos:

( )

0 0

*

2

1

00 0 0 1

1

1

r r

cc

c

c

=

⎛ ⎞⎜ ⎟ =⎜ ⎟⎜ ⎟⎝ ⎠

=

=

(29)

Portanto, o autovetor correspondente é:

10

0

001

r⎛ ⎞⎜ ⎟= ⎜ ⎟⎜ ⎟⎝ ⎠

. (30)

Para o cálculo dos autovetores, temos para λ = + :

( )1 1

2 2

3 3

2 1

1 2

3

2 1

1 2

3

0 00 0

0 0 0

0

0

i x xi x x

x x

i x xi x x

x

ix xix xx

λ−⎛ ⎞⎛ ⎞ ⎛ ⎞

⎜ ⎟⎜ ⎟ ⎜ ⎟= = +⎜ ⎟⎜ ⎟ ⎜ ⎟⎜ ⎟⎜ ⎟ ⎜ ⎟⎝ ⎠⎝ ⎠ ⎝ ⎠

− =⎧⎪ =⎨⎪ =⎩

− =⎧⎪ =⎨⎪ =⎩

(31)

Portanto,

1

1

0

xix⎛ ⎞⎜ ⎟⎜ ⎟⎜ ⎟⎝ ⎠

. (32)

Fazendo 1x c= , temos:

0

cic⎛ ⎞⎜ ⎟⎜ ⎟⎜ ⎟⎝ ⎠

(33)

11

Normalizando, temos:

1 1 1r r =

( )*

2

11 0 1

0

(1 1) 1

1/ 2

c i c i

c

c

⎛ ⎞⎜ ⎟− =⎜ ⎟⎜ ⎟⎝ ⎠

+ =

=

(34)

Portanto

11/ 2

0r i+

⎛ ⎞⎜ ⎟= ⎜ ⎟⎜ ⎟⎝ ⎠

. (35)

Para o cálculo dos autovetores, temos para λ = − :

( )1 1

2 2

3 3

2 1

1 2

3

0 00 0

0 0 0

0

i x xi x x

x x

i x xi x x

x

λ−⎛ ⎞⎛ ⎞ ⎛ ⎞

⎜ ⎟⎜ ⎟ ⎜ ⎟= = −⎜ ⎟⎜ ⎟ ⎜ ⎟⎜ ⎟⎜ ⎟ ⎜ ⎟⎝ ⎠⎝ ⎠ ⎝ ⎠

− = −⎧⎪ = −⎨⎪ = −⎩

(36)

12

2 1

1 2

3

2 1

1 2

3

0

0

ix xix x

x

ix xix x

x

− = −⎧⎪ = −⎨⎪ =⎩

=⎧⎪ = −⎨⎪ =⎩

(37)

Portanto,

1

1

0

xix

⎛ ⎞⎜ ⎟−⎜ ⎟⎜ ⎟⎝ ⎠

. (38)

Fazendo 1x c= , temos:

0

cic

⎛ ⎞⎜ ⎟−⎜ ⎟⎜ ⎟⎝ ⎠

(39)

Normalizando, temos:

13

( )

( )

*

2 2

2

11 0 1

0

1 1

(2) 1

1/ 2

c i c i

c i

c

c

⎛ ⎞⎜ ⎟− =⎜ ⎟⎜ ⎟⎝ ⎠

− =

=

=

(40)

Portanto,

11/ 2

0r i−

⎛ ⎞⎜ ⎟= −⎜ ⎟⎜ ⎟⎝ ⎠

. (41)

De uma maneira geral, os autovalores e autovetores são:

0λ = 0

001

r⎛ ⎞⎜ ⎟= ⎜ ⎟⎜ ⎟⎝ ⎠

(42)

λ = + 1

1/ 20

r i+

⎛ ⎞⎜ ⎟= ⎜ ⎟⎜ ⎟⎝ ⎠

(43)

λ = − 1

1/ 20

r i−

⎛ ⎞⎜ ⎟= −⎜ ⎟⎜ ⎟⎝ ⎠

(44)

14

........................................................................................................................... Demonstração: Podemos agora testar a ortogonalidade dos autovetores.

( ) ( )1

11/ 2 1 0 1/ 2 1 1 02

0r r i i− +

⎛ ⎞⎜ ⎟= = − =⎜ ⎟⎜ ⎟⎝ ⎠

(45)

( )0

10 0 1 1/ 2 0

0r r i+

⎛ ⎞⎜ ⎟= =⎜ ⎟⎜ ⎟⎝ ⎠

(46)

( )0

10 0 1 1/ 2 0

0r r i−

⎛ ⎞⎜ ⎟= − =⎜ ⎟⎜ ⎟⎝ ⎠

(47)

........................................................................................................................... Vamos agora encontrar a matriz unitária que transforma iG a iJ , com 3J diagonal, tal que †

i iJ U GU= , onde U é construído a partir dos autovetores de 3G .

Primeiramente, devemos ter em mente que:

3

0 1 01 0 00 0 0

G i−⎛ ⎞

⎜ ⎟= ⎜ ⎟⎜ ⎟⎝ ⎠

(48)

15

11/ 2

0r i+

⎛ ⎞⎜ ⎟= ⎜ ⎟⎜ ⎟⎝ ⎠

0

001

r⎛ ⎞⎜ ⎟= ⎜ ⎟⎜ ⎟⎝ ⎠

1

1/ 20

r i−

⎛ ⎞⎜ ⎟= −⎜ ⎟⎜ ⎟⎝ ⎠

(49)

1 01 0 0 22 1 0

i

Ui

⎛ ⎞⎜ ⎟

= ⎜ ⎟⎜ ⎟−⎝ ⎠

(50)

1 0 11 02

0 2 0

U i i⎛ ⎞⎜ ⎟

= −⎜ ⎟⎜ ⎟⎝ ⎠

(51)

Podemos mostrar que U é realmente unitária.

1 0 1 0 11 10 0 2 02 21 0 0 2 0

2 0 01 0 2 02

0 0 2

1

i

UU i ii

UU

UU

⎛ ⎞ ⎛ ⎞⎜ ⎟ ⎜ ⎟

= −⎜ ⎟ ⎜ ⎟⎜ ⎟ ⎜ ⎟− ⎝ ⎠⎝ ⎠

⎛ ⎞⎜ ⎟= ⎜ ⎟⎜ ⎟⎝ ⎠

=

(52)

Podemos agora calcular o operador momento angular 3J :

16

( ) ( )

( )

( )

3

3

3

1 01 0 1 0 1 01/ 2 0 1 0 0 1/ 2 0 0 2

0 0 0 1 00 2 0

1 0 1 0 0 2/ 2 0 1 0

0 0 00 2 0

0 0 2

/ 2 0 0 2

2 2 0

i

J i i ii

J i i i i

J i i

i

⎛ ⎞⎛ ⎞ −⎛ ⎞⎜ ⎟⎜ ⎟ ⎜ ⎟= − ⎜ ⎟⎜ ⎟ ⎜ ⎟

⎜ ⎟ ⎜ ⎟⎜ ⎟ −⎝ ⎠⎝ ⎠ ⎝ ⎠

⎛ ⎞⎛ ⎞ −⎜ ⎟⎜ ⎟

= − ⎜ ⎟⎜ ⎟⎜ ⎟⎜ ⎟

⎝ ⎠⎝ ⎠

⎛ ⎞−⎜ ⎟

= ⎜ ⎟⎜ ⎟⎜ ⎟⎝ ⎠

. (53)

Vamos agora diagonalizar a matriz 3J ,

3

0 0 2 / 2

0 0 2 / 2

2 / 2 2 / 2 0

i

J

i

⎛ ⎞−⎜ ⎟

= −⎜ ⎟⎜ ⎟⎜ ⎟−⎝ ⎠

. (54)

Vamos encontrar a equação característica.

17

( )

3

2 2 23

2 23

3 2

2 2

0

0 0 2 / 2

0 0 2 / 2 0

2 / 2 2 / 2 0

0 2 / 2 0

0 2 / 2 0 0

2 / 2 2 / 2 2 / 2 2 / 2

02 2

02 2

0

0

J I

i

i

i

i i

i

λ

λ

λ

λ

λ λ

λ λ

λ

λ λλ

λ λλ

λ λ

λ λ

− =

− −

− − =

− −

− − −

− − − =

− − −

− − + =

− + + =

− =

− = (55)

Portanto, as raízes são:

0λ = , λ = + e λ = − (56) A matriz diagonalizada é:

3

0 0 00 00 0

J⎛ ⎞⎜ ⎟= ⎜ ⎟⎜ ⎟−⎝ ⎠

(57)

18

A matriz unitária U transforma a representação do espaço cartesiano dos operadores momento angular, isto é, G , na representação da base esférica dela, J . Portanto, os J ’s e G ’s estão relacionados via uma rotação, e esta rotação finita pode ser obtida a partir da composição de rotações infinitesimais

n̂φ∇→∇+ ×∇ (ou ˆG G n Gφ→ + × ). (58)

1

Problema 3.13 Mostre que as matrizes 3 3x ( 1, 2,3)iG i = , cujos elementos são dados por ( )i ijkjkG i ε= − ,

onde j e k são os índices das linhas e das colunas, satisfazem as relações de comutação de momento angular. Qual é o significado físico (ou geométrico) da matriz transformação que conecta iG as representações mais usuais 3 3x do operador momento angular iJ , com 3J considerado diagonal. Relacione o seu resultado ao

ˆV V n Vδφ→ + × sob rotações infinitesimais. (Nota: Este problema pode ser útil na compreensão do spin do fóton.) Solução: Olhando para os elementos matriciais, temos:

( ) ( ) ( ) ( )ln ln

ln

,

,

i j i j j i

i j i j j ilm mnmn lm

G G G G G G

G G G G G G

⎡ ⎤ ⎡ ⎤= −⎣ ⎦ ⎣ ⎦

⎡ ⎤ = −⎣ ⎦

2

........................................................................................................................... Lembre-se:

i j i jn l

i j i jn l m

i j i jm n l

l G G n l l G G n n

l G G n l l G m m G n n

l G G n l l G m m G n n

=

⎛ ⎞= ⎜ ⎟

⎝ ⎠

=

∑ ∑

∑ ∑ ∑

∑ ∑ ∑

...........................................................................................................................

( ) ( ) ( ) ( )ln

2

ln

2

ln

,

,

,

i j i j j ilm mnmn lm

i j ilm jmn jlm imn

i j mil mnj mjl mni

G G G G G G

G G

G G

ε ε ε ε

ε ε ε ε

⎡ ⎤ = −⎣ ⎦

⎡ ⎤ ⎡ ⎤= − −⎣ ⎦ ⎣ ⎦

⎡ ⎤ ⎡ ⎤= − −⎣ ⎦ ⎣ ⎦

........................................................................................................................... Demonstração: Para duas permutações, temos que ε é positivo.

ilm iml mil

jmn mjn mnj

ε ε ε

ε ε ε

→ →

→ →

3

min

jlm jml mjl

imn mni

ε ε ε

ε ε ε

→ →

→ →

...........................................................................................................................

( ) ( )

2

ln

2ln lnln

,

,

i j mil mnj mjl mni

i j in lj ij jn li ji

G G

G G

ε ε ε ε

δ δ δ δ δ δ δ δ

⎡ ⎤ ⎡ ⎤= − −⎣ ⎦ ⎣ ⎦

⎡ ⎤⎡ ⎤ = − − − −⎣ ⎦ ⎣ ⎦

Vamos considerar agora a seguinte relação:

ln

ln

mil mnj in lj ij

mjl mni jn li ji

ε ε δ δ δ δ

ε ε δ δ δ δ

= −

= −

........................................................................................................................... Lembre-se:

4

........................................................................................................................... ........................................................................................................................... Demonstração: Podemos testar também esta relação.

ln

ln

mil mnj in lj ij

mjl mni jn li ji

ε ε δ δ δ δ

ε ε δ δ δ δ

= −

= −

Vamos considerar três situações em particular: I) Três índices iguais: 1m = , 1i = , 1j = , 1l = e 1n = Nesta situação temos:

5

ln

111 111 11 11 11 11

0 0

mil mnj in lj ijε ε δ δ δ δ

ε ε δ δ δ δ

= −

= −

=

II) Dois índices iguais: 1m = , 1i = , 3j = , 3l = e 2n = Nesta situação temos:

ln

113 123 12 33 13 32

0 0

mil mnj in lj ijε ε δ δ δ δ

ε ε δ δ δ δ

= −

= −

=

III) Três índices diferentes: 1m = , 2i = , 3j = , 3l = e 2n = Nesta situação temos:

ln

123 123 22 33 23 32

1 1

mil mnj in lj ijε ε δ δ δ δ

ε ε δ δ δ δ

= −

= −

=

Portanto, vale a relação acima. ...........................................................................................................................

6

Voltando a relação principal, temos:

( ) ( )( )

( )( )

2ln lnln

2

ln

2lnln

lnln

lnln

,

,

,

,

,

i j in lj ij jn li ji

i j il jn in jl

i j kij k

i j ijk k

i j ijk k

G G

G G

G G

G G i i

G G i G

δ δ δ δ δ δ δ δ

δ δ δ δ

ε ε

ε ε

ε

⎡ ⎤⎡ ⎤ = − − − −⎣ ⎦ ⎣ ⎦⎡ ⎤ = −⎣ ⎦

⎡ ⎤ =⎣ ⎦

⎡ ⎤ = −⎣ ⎦

⎡ ⎤ =⎣ ⎦

Portanto,

,i j ijk kG G i Gε⎡ ⎤ =⎣ ⎦ . A forma explicita de 3G (a partir de ( )i ijkjk

G i ε= − , onde j e k são os índices correspondentes a linhas e colunas) é dada por:

3

0 1 01 0 00 0 0

G i−⎛ ⎞

⎜ ⎟= ⎜ ⎟⎜ ⎟⎝ ⎠

.

........................................................................................................................... Demonstração: Índice 3i = Linha 2: 2j = Coluna 1: 1k =

7

( )i ijkjkG i ε= −

Temos 3 permutações para ε :

321 312 132 123ε ε ε ε→ → → Portanto, ( )i ijkjkG i ε= − ( )3 32121

( 1)G i i iε= − = − − = ........................................................................................................................... Os autovalores e autovetores são obtidos a partir da equação ( )3 0G I rλλ− = , em que λ é a raiz de

3 0G Iλ− = . Os autovalores e autovetores ortonormais podem ser imediatamente mostrados. Para o cálculo dos autovalores, temos:

8

3 2

0et 0 0

0 0

00 0

0 0 0 0

0

iD i

i iDet i i

λλ

λ

λ λλ λ

λ

λ λ

− −⎛ ⎞⎜ ⎟− =⎜ ⎟⎜ ⎟−⎝ ⎠

− − − −⎛ ⎞⎜ ⎟− − =⎜ ⎟⎜ ⎟−⎝ ⎠

− + =

As raízes são:

0λ = , λ = + e λ = − . Para o cálculo dos autovetores, temos para 0λ = :

( )1 1

2 2

3 3

2

1

0 00 0 0

0 0 0

00

0

i x xi x x

x x

i xi x

λ−⎛ ⎞⎛ ⎞ ⎛ ⎞

⎜ ⎟⎜ ⎟ ⎜ ⎟= =⎜ ⎟⎜ ⎟ ⎜ ⎟⎜ ⎟⎜ ⎟ ⎜ ⎟⎝ ⎠⎝ ⎠ ⎝ ⎠

− =⎧⎪ =⎨⎪⎩

Normalizando,

0

00rc

⎛ ⎞⎜ ⎟= ⎜ ⎟⎜ ⎟⎝ ⎠

9

temos:

( )

0 0

*

2

1

00 0 0 1

1

1

r r

cc

c

c

=

⎛ ⎞⎜ ⎟ =⎜ ⎟⎜ ⎟⎝ ⎠

=

=

Portanto, o autovetor correspondente é:

0

001

r⎛ ⎞⎜ ⎟= ⎜ ⎟⎜ ⎟⎝ ⎠

.

Para o cálculo dos autovetores, temos para λ = + :

( )1 1

2 2

3 3

2 1

1 2

3

2 1

1 2

3

0 00 0

0 0 0

0

0

i x xi x x

x x

i x xi x x

x

ix xix xx

λ−⎛ ⎞⎛ ⎞ ⎛ ⎞

⎜ ⎟⎜ ⎟ ⎜ ⎟= = +⎜ ⎟⎜ ⎟ ⎜ ⎟⎜ ⎟⎜ ⎟ ⎜ ⎟⎝ ⎠⎝ ⎠ ⎝ ⎠

− =⎧⎪ =⎨⎪ =⎩

− =⎧⎪ =⎨⎪ =⎩

10

Portanto,

1

1

0

xix⎛ ⎞⎜ ⎟⎜ ⎟⎜ ⎟⎝ ⎠

Fazendo 1x c= , temos:

0

cic⎛ ⎞⎜ ⎟⎜ ⎟⎜ ⎟⎝ ⎠

Normalizando, temos:

1 1 1r r =

( )*

2

11 0 1

0

(1 1) 1

1/ 2

c i c i

c

c

⎛ ⎞⎜ ⎟− =⎜ ⎟⎜ ⎟⎝ ⎠

+ =

=

Portanto

11/ 2

0r i+

⎛ ⎞⎜ ⎟= ⎜ ⎟⎜ ⎟⎝ ⎠

11

Para o cálculo dos autovetores, temos para λ = − :

( )1 1

2 2

3 3

2 1

1 2

3

0 00 0

0 0 0

0

i x xi x x

x x

i x xi x x

x

λ−⎛ ⎞⎛ ⎞ ⎛ ⎞

⎜ ⎟⎜ ⎟ ⎜ ⎟= = −⎜ ⎟⎜ ⎟ ⎜ ⎟⎜ ⎟⎜ ⎟ ⎜ ⎟⎝ ⎠⎝ ⎠ ⎝ ⎠

− = −⎧⎪ = −⎨⎪ = −⎩

2 1

1 2

3

2 1

1 2

3

0

0

ix xix x

x

ix xix x

x

− = −⎧⎪ = −⎨⎪ =⎩

=⎧⎪ = −⎨⎪ =⎩

Portanto,

1

1

0

xix

⎛ ⎞⎜ ⎟−⎜ ⎟⎜ ⎟⎝ ⎠

Fazendo 1x c= , temos:

0

cic

⎛ ⎞⎜ ⎟−⎜ ⎟⎜ ⎟⎝ ⎠

12

Normalizando, temos:

( )

( )

*

2 2

2

11 0 1

0

1 1

(2) 1

1/ 2

c i c i

c i

c

c

⎛ ⎞⎜ ⎟− =⎜ ⎟⎜ ⎟⎝ ⎠

− =

=

=

Portanto,

11/ 2

0r i−

⎛ ⎞⎜ ⎟= −⎜ ⎟⎜ ⎟⎝ ⎠

De uma maneira geral, os autovalores e autovetores são:

0λ = 0

001

r⎛ ⎞⎜ ⎟= ⎜ ⎟⎜ ⎟⎝ ⎠

λ = + 1

1/ 20

r i+

⎛ ⎞⎜ ⎟= ⎜ ⎟⎜ ⎟⎝ ⎠

13

λ = − 1

1/ 20

r i−

⎛ ⎞⎜ ⎟= −⎜ ⎟⎜ ⎟⎝ ⎠

........................................................................................................................... Demonstração: Podemos agora testar a ortogonalidade dos autovetores.

( ) ( )1

11/ 2 1 0 1/ 2 1 1 02

0r r i i− +

⎛ ⎞⎜ ⎟= = − =⎜ ⎟⎜ ⎟⎝ ⎠

( )0

10 0 1 1/ 2 0

0r r i+

⎛ ⎞⎜ ⎟= =⎜ ⎟⎜ ⎟⎝ ⎠

( )0

10 0 1 1/ 2 0

0r r i−

⎛ ⎞⎜ ⎟= − =⎜ ⎟⎜ ⎟⎝ ⎠

........................................................................................................................... Vamos agora encontrar a matriz unitária que transforma iG a iJ , com 3J diagonal, tal que †

i iJ U GU= , onde U é construído a partir dos autovetores de 3G .

Primeiramente, devemos ter em mente que:

14

3

0 1 01 0 00 0 0

G i−⎛ ⎞

⎜ ⎟= ⎜ ⎟⎜ ⎟⎝ ⎠

1

1/ 20

r i+

⎛ ⎞⎜ ⎟= ⎜ ⎟⎜ ⎟⎝ ⎠

0

001

r⎛ ⎞⎜ ⎟= ⎜ ⎟⎜ ⎟⎝ ⎠

1

1/ 20

r i−

⎛ ⎞⎜ ⎟= −⎜ ⎟⎜ ⎟⎝ ⎠

1 01 0 0 22 1 0

i

Ui

⎛ ⎞⎜ ⎟

= ⎜ ⎟⎜ ⎟−⎝ ⎠

1 0 11 02

0 2 0

U i i⎛ ⎞⎜ ⎟

= −⎜ ⎟⎜ ⎟⎝ ⎠

Podemos mostrar que U é realmente unitária.

1 0 1 0 11 10 0 2 02 21 0 0 2 0

2 0 01 0 2 02

0 0 2

1

i

UU i ii

UU

UU

⎛ ⎞ ⎛ ⎞⎜ ⎟ ⎜ ⎟

= −⎜ ⎟ ⎜ ⎟⎜ ⎟ ⎜ ⎟− ⎝ ⎠⎝ ⎠

⎛ ⎞⎜ ⎟= ⎜ ⎟⎜ ⎟⎝ ⎠

=

Podemos agora calcular o operador momento angular 3J :

15

( ) ( )

( )

( )

3

3

3

1 01 0 1 0 1 01/ 2 0 1 0 0 1/ 2 0 0 2

0 0 0 1 00 2 0

1 0 1 0 0 2/ 2 0 1 0

0 0 00 2 0

0 0 2

/ 2 0 0 2

2 2 0

i

J i i ii

J i i i i

J i i

i

⎛ ⎞⎛ ⎞ −⎛ ⎞⎜ ⎟⎜ ⎟ ⎜ ⎟= − ⎜ ⎟⎜ ⎟ ⎜ ⎟

⎜ ⎟ ⎜ ⎟⎜ ⎟ −⎝ ⎠⎝ ⎠ ⎝ ⎠

⎛ ⎞⎛ ⎞ −⎜ ⎟⎜ ⎟

= − ⎜ ⎟⎜ ⎟⎜ ⎟⎜ ⎟

⎝ ⎠⎝ ⎠

⎛ ⎞−⎜ ⎟

= ⎜ ⎟⎜ ⎟⎜ ⎟⎝ ⎠

.

Vamos agora diagonalizar a matriz 3J ,

3

0 0 2 / 2

0 0 2 / 2

2 / 2 2 / 2 0

i

J

i

⎛ ⎞−⎜ ⎟

= −⎜ ⎟⎜ ⎟⎜ ⎟−⎝ ⎠

.

Vamos encontrar a equação característica.

16

( )

3

2 2 23

2 23

3 2

2 2

0

0 0 2 / 2

0 0 2 / 2 0

2 / 2 2 / 2 0

0 2 / 2 0

0 2 / 2 0 0

2 / 2 2 / 2 2 / 2 2 / 2

02 2

02 2

0

0

J I

i

i

i

i i

i

λ

λ

λ

λ

λ λ

λ λ

λ

λ λλ

λ λλ

λ λ

λ λ

− =

− −

− − =

− −

− − −

− − − =

− − −

− − + =

− + + =

− =

− = .

Portanto, as raízes são:

0λ = , λ = + e λ = − A matriz diagonalizada é:

3

0 0 00 00 0

J⎛ ⎞⎜ ⎟= ⎜ ⎟⎜ ⎟−⎝ ⎠

17

A matriz unitária U transforma a representação do espaço cartesiano dos operadores momento angular, isto é, G , na representação da base esférica dela, J . Portanto, os J ’s e G ’s estão relacionados via uma rotação, e esta rotação finita pode ser obtida a partir da composição de rotações infinitesimais

n̂φ∇→∇+ ×∇ (ou ˆG G n Gφ→ + × ).

1

Problema 3.14 a) Considere que J seja o momento angular. Ele pode ser o orbital L , spin S , ou o totalJ . Usando o fato de que xJ , yJ , zJ ( yx iJJJ ±≡± ) satisfaça as relações usuais de comutação de momento angular, prove

zz JJJJJ −+= −+22 .

b) Usando (a) (ou outra forma), derive a “famoso” expressão para o coeficiente −c que aparece em

1, −−− = mjjm cJ ψψ . Solução: a) Temos que:

)(

)(

)(

))((

22

22

22

22

xyyxz

xyyxz

xyyxyx

yyxxyx

yxyx

JJJJiJJJJ

JJJJiJJJJ

JJJJiJJJJ

JJiJJiJJJJ

iJJiJJJJ

−++=

−−−=

−−+=

+−+=

−+=

−+

−+

−+

−+

−+

(1)

........................................................................................................................... Lembre-se: Através da equação (3.1.20), temos:

[ ] zyxxyyx JiJJJJJJ ==− ,)( . ...........................................................................................................................

2

Podemos então reescrever a expressão acima como:

zz

zz

xyyxz

JJJJJ

iiJJJJJ

JJJJiJJJJ

−+=

++=

−++=

−+

−+

−+

22

22

22

)(

)(

(2)

b) Temos por um lado que

2−−+ = cjmJJjm , (3)

enquanto, que usando a relação

zz JJJJJ +−=−+22 (4)

temos por outro lado que

[ ] 22))1( mmjjjmJJjm +−+=−+ . (5) Então, temos que,

[ ] 222 ))1( mmjjc +−+=− , (6) ou

22 )1)(( +−+=− mjmjc . (7) Por convenção, vamos escolher

)1)(( +−+=− mjmjc . (8) Então,

1,

1,

−−−

−−

=

−=

mjjm cJ

mjcjmJ

ψψ (9)

1

Problema 3.15 A função de onda de uma partícula sujeita a um potencial esfericamente simétrico ( )V r é dado por:

( ) ( 3 ) ( )x x y z f rψ = + + . a.) ψ é uma autofunção de 2L ? Em caso afirmativo, qual é o valor l ? Se não, quais são os possíveis valores de l que podemos obter quando 2L é medido? b.) Quais são as probabilidades para a partícula ser achada nos vários estados

lm ? c.) Suponha que conhecemos que ( )xψ é uma autofunção de energia com autovalor E . Indique como nós podemos achar ( )V r ? Solução : a.) Temos que: ( ) ( )3 ( )x x x y z f rψ ψ= = + + . (1)

Para verificarmos se ( )xψ dada por (1) é uma autofunção de 2L aplicaremos este operador ao ket ( )xψ e veremos o resultado. Trabalharemos com coordenadas esféricas (3.6.15):

( )2

2 22 2

1 1( )x L x sen xsen sen

ψ θ ψθ φ θ θ θ

⎧ ⎫∂ ∂ ∂⎛ ⎞= − +⎨ ⎬⎜ ⎟∂ ∂ ∂⎝ ⎠⎩ ⎭. (2)

Escrevendo ( )xψ em coordenadas esféricas,

cos

cos

x rseny rsen senz r

θ φθ φθ

===

, (3)

2

temos: ( ) ( )( cos 3cos )x rf r sen sen senψ θ φ θ φ θ= + + . (4)

Com a equação (4), podemos calcular cada termo separadamente para a equação (1):

( ) [ ]

( ) ( )

( )

2 2

2 2 2 2

2

2 2 2

2

2 2

1 1 ( )( cos 3cos )

1 ( ) cos

1 ( ) (cos )

x rf r sen sen sensen sen

rf r senx sensen sen

rf rx sensen sen

ψ θ φ θ φ θθ φ θ φ

θψ φ φθ φ θ φ

ψ φ φθ φ θ

∂ ∂= + +

∂ ∂

∂ ∂= −

∂ ∂

∂= − +

(5)

e

( ) [ ]

( )

( )

1 1 ( )( cos 3cos )

1 ( ) ( cos 3cos )

1 ( ) co

sen x sen rf r sen sen sensen sen

rf rsen x sen sen sen sensen sen

rf rsen x sensen sen

θ ψ θ θ φ θ φ θθ θ θ θ θ θ

θ ψ θ θ φ θ φ θθ θ θ θ θ θ

θ ψ θθ θ θ θ θ

∂ ∂ ∂ ∂⎛ ⎞ ⎛ ⎞= + +⎜ ⎟ ⎜ ⎟∂ ∂ ∂ ∂⎝ ⎠ ⎝ ⎠⎡ ⎤∂ ∂ ∂ ∂⎛ ⎞ ⎛ ⎞= + +⎜ ⎟ ⎜ ⎟⎢ ⎥∂ ∂ ∂ ∂⎝ ⎠ ⎝ ⎠⎣ ⎦

∂ ∂ ∂⎛ ⎞ =⎜ ⎟∂ ∂ ∂⎝ ⎠

( )

( )

2

2

2 2

s cos cos 3

1 ( ) 3 (cos ) cos

1 ( ) 6 cos (cos )(cos )

sen sen sen

rf rsen x sen sen sensen sen

rf rsen x sen sen sensen sen

φ θ θ φ θ θ

θ ψ θ φ φ θ θθ θ θ θ θ

θ ψ θ θ φ φ θ θθ θ θ θ

⎡ ⎤+ −⎣ ⎦

∂ ∂ ∂⎛ ⎞ ⎡ ⎤= − + +⎜ ⎟ ⎣ ⎦∂ ∂ ∂⎝ ⎠∂ ∂⎛ ⎞ ⎡ ⎤= − + + −⎜ ⎟ ⎣ ⎦∂ ∂⎝ ⎠

Substituindo em (1), temos:

3

( )

( )

2 2 2 2

2 2 2 2

2 2 2

1 1( ) (cos ) 6 cos (cos )(cos )

1 1( ) (cos ) cos (cos ) 6cos

1( ) cos ( 1 cos

x L rf r sen sen sen sensen sen

x L rf r sen sen sensen sen

x L rf r sensen

ψ φ φ θ θ φ φ θ θθ θ

ψ φ φ φ φ θ θ θθ θ

ψ φ φ θθ

⎧ ⎫⎡ ⎤ ⎡ ⎤= − − + + − + + −⎨ ⎬⎣ ⎦⎢ ⎥⎣ ⎦⎩ ⎭⎡ ⎤= − − + + + − −⎢ ⎥⎣ ⎦

= − + − + −

( )[ ]

( )

2

2 2 2

2 2

2 2

) 6cos

1( ) 2 (cos ) 6cos

2 ( ) cos 3cos

2

sen

x L rf r sen sensen

x L rf r sen sen sen

x L x

θ θ

ψ θ φ φ θθ

ψ θ φ θ φ θ

ψ ψ

⎡ ⎤−⎢ ⎥⎣ ⎦⎡ ⎤= − − + −⎢ ⎥⎣ ⎦

= + +

=

ou

( ) ( )( )( )

2 2

2 2

2 2

2

1(1 1) ( )

( 1) ( )

L x x

L x x

L x l l x

ψ ψ

ψ ψ

ψ ψ

=

= +

= +

(8)

o que significa que ( )xψ é uma autofunção de 2L com autovalor 1l = . ........................................................................................................................... Lembre-se:

2 2 2 2 21 cos 1 1 2sen sen sen senθ θ θ θ θ− + − = − + − − = − (9) ........................................................................................................................... b.) Como nos já conhecemos que 1l = , podemos tentar escrever ( )xψ em termos dos harmônicos esféricos 1 ( , )mY θ φ . Sabemos que (3.6.39):

01

3 3cos4 4

zYr

θπ π

= = . (10)

4

Isolando z , temos:

01

43

z r Yπ= . (11)

Temos também que (A.5.7),

( ) ( )

( ) ( )

11

11

3 3 3cos8 8 83 3 3cos

8 8 8

i

i

Y sen e sen isen sen co isen sen

Y sen e sen isen sen co isen sen

φ

φ

θ θ φ φ θ φ θ φπ π π

θ θ φ φ θ φ θ φπ π π

+

− −

= − = − + = − +

= = − = −

. (12)

Como ( ) ( )

( ) ( )

x iysen co isen sen

rx iy

sen co isen senr

θ φ θ φ

θ φ θ φ

+= +

−= −

, (13)

........................................................................................................................... Lembre-se: ( )

( )

cos

cos

x iy rsen irsen senr r

x iysen isen sen

r

θ φ θ φ

θ φ θ φ

+ +=

+= +

(14)

........................................................................................................................... temos:

( )

( )

11

11

383

8

x iyY

rx iy

Yr

π

π

+

+= −

−=

. (15)

5

Isolando x e y , temos:

( )

( )

1 11 1

1 11 1

2323

x r Y Y

y ir Y Y

π

π

− +

− +

= −

= +

. (16)

Podemos agora escrever

( ) ( 3 ) ( )x x y z f rψ = + + (17) considerando que

( )

( )

1 11 1

1 11 1

2323

x r Y Y

y ir Y Y

π

π

− +

− +

= −

= +

(18)

01

43

z r Yπ=

como:

( ) ( )

( ) ( )

( )

( ) ( ) ( )

1 1 1 1 01 1 1 1 1

1 1 1 1 01 1 1 1 1

0 1 1 1 11 1 1 1 1

0 1 11 1 1

( ) ( 3 ) ( )

2 2 4( ) ( )( 3 )3 3 3

2 2 2 2( ) ( )( 3 )3 3 3

2 ( ) 3 23

2 ( ) 3 2 1 13

x x y z f r

x f r r Y Y ir Y Y r Y

x rf r Y Y i Y Y Y

x r f r Y Y Y iY iY

x r f r Y i Y i Y

ψ

π π πψ

π π πψ

πψ

πψ

− + − +

− + − +

− + − +

− +

= + +

= − + + +

= − + + +

⎡ ⎤= + − + +⎣ ⎦

⎡ ⎤= + + + −⎣ ⎦

. (19)

Temos que a parte do estado que depende dos valores de m pode ser escrito na seguinte forma:

( ) ( )3 2 1, 0 1 1, 1 1 1, 1m

N l m i l m i l mψ ⎡ ⎤= = = + + = = − + − = =⎣ ⎦ . (20)

6

Normalizando ele, podemos escrever:

( ) ( )

( )

2 22 2

2

3 2 1 (1 ) 1

18 2 2 1

122

N i i

N

N

⎡ ⎤+ + + − =⎢ ⎥⎣ ⎦

+ + =

=

. (21)

Ou seja,

( ) ( )1 3 2 1, 0 1 1, 1 1 1, 122

l m i l m i l mψ ⎡ ⎤= = = + + = = − + − = =⎣ ⎦ . (22)

Então,

( )

( )

( )

22

22

22

1 9 2 90 1, 0 3 222 1122

1 2 11 1, 1 (1 )22 1122

1 2 11 1, 1 (1 )22 1122

xP m l m

P m l m i

P m l m i

ψ

ψ

ψ

= = = = = = =

= + = = = + = − = =

= − = = = − = + = =

. (23)

c.) Se ( )E xψ é uma autofunção de energia, então ela pode ser usada para resolver a equação de Schrödinger (2.4.11):

22 ( ) ( ) ( )

2 E E Eu x Vu x Eu xm

⎛ ⎞− ∇ + =⎜ ⎟⎝ ⎠

. (24)

7

Considerando que o operador 2∇ pode ser escrito em coordenadas esféricas com apenas a componente radial da seguinte forma (3.6.21),

22 2

2 22 2

2 2 2

2 2 22

2 2 2

12 2

2 12

22 E E E

x p xm m

x x x x Lm r r r r

Lu u um r r r r

α α

α α α α

⎛ ⎞= − ∇⎜ ⎟

⎝ ⎠

⎛ ⎞ ⎛ ⎞∂ ∂− ∇ = + −⎜ ⎟ ⎜ ⎟∂ ∂⎝ ⎠ ⎝ ⎠

⎛ ⎞⎛ ⎞ ∂ ∂− ∇ = + −⎜ ⎟⎜ ⎟ ∂ ∂⎝ ⎠ ⎝ ⎠

(25)

temos:

( ) ( ) ( ) ( ) ( ) ( )

( ) ( ) ( ) ( ) ( ) ( )

2 2 2

2 2 2

2 2

2 2

22

2 22

E E E E E

m m ml l l

Lx x x V r x E xm r r r r

d dY rf r rf r rf r V r rf r Y Erf r Ym dr r dr r

ψ ψ ψ ψ ψ⎡ ⎤∂ ∂

− + − + =⎢ ⎥∂ ∂⎣ ⎦

⎡ ⎤⎡ ⎤ ⎡ ⎤ ⎡ ⎤− + − + =⎢ ⎥⎣ ⎦ ⎣ ⎦ ⎣ ⎦

⎣ ⎦

(26)

Isolando ( )V r , temos:

( ) [ ] [ ]

( ) [ ]

( ) [ ]

( )

2

2

2

2

1 2 2( ) '( ) ( ) '( ) ( )( ) 21 ( ) '( ) 2 '( )( ) 21 '( ) '( ) ''( ) 2 '( )( ) 2

''( ) 4 '( )2 ( )

dV r E f r rf r f r rf r f rrf r m dr r r

dV r E f r rf r f rrf r m dr

V r E f r f r rf r f rrf r m

rf r f rV r Em rf r

⎡ ⎤= + + + + −⎢ ⎥⎣ ⎦

⎡ ⎤= + + +⎢ ⎥⎣ ⎦

= + + + +

+= +

. (27)

1

Problema 3.16 Uma partícula em um potencial esfericamente simétrico é conhecida estar em um autoestado de 2L e zL com autovalores 2 ( 1)l l + e m , respectivamente. Prove que os valores esperados entre os estados lm satisfazem

0x yL L= = , 2 2 2

2 2( 1)

2x y

l l mL L

⎡ ⎤+ −⎣ ⎦= = .

Interprete este resultado semi-classicamente. Solução: Temos que:

( )

( )

12

2

x

y

L L L

iL L L

+ −

− +

= +

= −. (1)

Também, a partir da (3.5.39) e (3.5.40), temos:

( )( )

( )( )

, 1 , 1

, 1 , 1

J j m j m j m j m

J j m j m j m j m

+

= − + + +

= + − + −

. (2)

Logo:

( ) ( ) ( )1 1, , , , , , , , 02 2x xL j m L j m j m L L j m j m L j m j m L j m+ − + −⎡ ⎤= = + = + =⎣ ⎦

O mesmo raciocínio pode ser aplicado para yL . Desta forma, demonstramos a primeira parte do problema.

2

Vamos agora calcular 2xL .

( )( )

( )

2

2 2 2

1414

x

x

L L L L L

L L L L L L L

+ − + −

+ − + − − +

= + +

= + + + (4)

Para o valor esperado, temos:

2 21,4xL j m L+= 2L−+( )

( )( ) ( )( ){ }

{ }

{ }{ }

2

2

2 2 2

22 2 2 2 2

22 2 2

2 2 22

,

1 1, , , ,4 41 1, , , ,4 41 1 14

4

2 2 24

2

x

x

x

x

x

x

L L L L j m

L j m L L j m j m L L j m

L j m L L j m j m L L j m

L j m j m j m j m

L j jm j mj m m j jm j mj m m

L j m j

j m jL

+ − − +

+ − − +

+ − − +

⎡ ⎤+ +⎢ ⎥⎣ ⎦

= +

= +

= + − + + − + +

= − + + − + + + + − − −

= − +

− +=

(5)

Na realidade, nos não temos spin, então:

lj = . (6) Desta forma, a equação acima pode ser escrita como:

( )[ ]

[ ]2

)1(2

222

2222

mllL

mllL

x

x

−+=

−+=

(7)

3

Vamos agora calcular 2yL .

( )( )

( )

2

2 2 2

141

4

y

y

L L L L L

L L L L L L L

− + − +

− + + − − +

−= − −

−= + − −

(8)

Para o valor esperado, temos:

2 21,4yL j m L+= − 2L−−( )

2

2 2

,

1 1, , , ,4 4y

y x

L L L L j m

L j m L L j m j m L L j m

L L

+ − − +

+ − − +

⎡ ⎤+ +⎢ ⎥⎣ ⎦

= +

=

(9)

Interpretação Semi-clássica. Conhecemos que

lmlllmL )1(22 += , (10) e,

lmmlmLz222 = . (11)

Então, os valores esperados são:

22 )1( += llL , (12) e,

222 mLz = . (13) Dentro da correspondência clássica,

2222zyx LLLL ++= , (14)

4

podemos expressar em termos dos valores esperados correspondentes:

[ ] [ ] 22222222222 )1()1(21)1(

21 LllmmllmllLLL zyx =+=+−++−+=++

1

Problema 3.17 Suponha que um valor de l semi-inteiro, digo ½, fosse permitido para o momento angular. A partir de

( )1/2,1/2 , 0L Y θ φ+ = , nós podemos deduzir, como usual

/21/2,1/2 ( , ) iY e senφθ φ θ∝ .

Agora, tentemos por construção 1/2, 1/2 ( , )Y θ φ− ; por a.) aplicando L− a ( )1/2,1/2 ,Y θ φ ; b.) usando ( )1/2, 1/2 , 0L Y θ φ− − = . Mostre que os dois procedimentos levam a resultados contraditórios. (Isto da um argumento contra valores l semi-inteiros para o momento angular orbital.) Solução: A partir da (3.6.13), temos para L+ :

cotiL i e iφ θθ φ

++

⎛ ⎞∂ ∂= − + −⎜ ⎟∂ ∂⎝ ⎠

.

Podemos também deduzir que

( ) 21/2,1/2 ,

iY e sen

φ

θ φ θ∝ a partir da equação

1/2,1/2 ( , ) 0L Y θ φ+ = .

2

........................................................................................................................... Demonstração: Para o caso m l= , temos:

, 0L l l+ = ou

ˆcot , 0ii e i n l lφ θθ φ

⎡ ⎤∂ ∂− − =⎢ ⎥∂ ∂⎣ ⎦

Considerando que

ˆ , ( , )lln l l Y θ φ= ,

podemos resolver a equação diferencial para ( , )l

lY θ φ :

cot 0

cot 0

cot 0

cot

i ll

l li il l

l ll l

l ll l

i e i Y

Y Ye i e

Y Yi

Y Yi

φ

φ φ

θθ φ

θθ φ

θθ φ

θθ φ

⎡ ⎤∂ ∂− − =⎢ ⎥∂ ∂⎣ ⎦

∂ ∂+ =

∂ ∂

∂ ∂+ =

∂ ∂

∂ ∂= −

∂ ∂

.

Considerando que

( ) ( ) ( ) ( ),l illY T R e Rφθ φ φ θ θ= = ,

temos:

cot

( ) cot ( )

l ll l

ilil

Y Yi

R ee i Rφ

φ

θθ φ

θ θ θθ φ

∂ ∂= −

∂ ∂

∂ ∂= −

∂ ∂

3

( ) cot ( )

( ) cot ( )

il ilRe i R ile

R i R il

φ φθ θ θθ

θ θ θθ

∂= −

∂∂

= −∂

Isolando para ( )R θ e θ , temos:

( )( )( ) ( )

( )

( ) cot

ln ln

ln ln l

l

R lR

R l sen

R sen

R sen

θ θ θθ

θ θ

θ θ

θ θ

∂= ∂

=

=

=

.

Para a solução da equação acima, foi considerado que:

( )cot lnd senθ θ θ= −∫ . Portanto,

( ),l il llY e senφθ φ θ= ,

ou, com a constante de integração, temos:

( ),l il l il ll lY e sen c e senφ φθ φ θ θ∝ = ,

em que a constante de integração vale:

( )1 (2 1)(2 )!2 ! 4

l

l l

l lcl π

⎡ ⎤− += ⎢ ⎥⎢ ⎥⎣ ⎦

.

Para l semi-inteiro, temos:

( )

( ) 21/2,1/2

( , ) ,

,

l l il ll l

i

Y l m l Y e sen

Y e sen

φ

φ

θ φ θ

θ φ θ

= = ∝

.

...........................................................................................................................

4

a. Aplicando L− a função 1/2,1/2Y , temos:

( ) ( )

( )

( ) ( ) ( )

( ) ( )

1/2,1/2 1/2, 1/2

21/2, 1/2

2 21/2, 1/2

21/2, 1/2

, ,

, cot

, cot

1 cos, cot2

ii

i ii i

i i

L Y Y

Y i e i e sen

Y i e i e sen i e e sen

Y e i esen

φφ

φ φφ φ

φφ

θ φ θ φ

θ φ θ θθ φ

θ φ θ θ θθ φ

θθ φθ

− −

−−

− −−

− −−

=

⎡ ⎤∂ ∂= − − −⎢ ⎥∂ ∂⎣ ⎦

⎛ ⎞ ⎛ ⎞⎛ ⎞∂ ∂⎛ ⎞= − − + − −⎜ ⎟ ⎜ ⎟⎜ ⎟⎜ ⎟∂ ∂⎝ ⎠ ⎝ ⎠⎝ ⎠ ⎝ ⎠⎛ ⎞

= − +⎜ ⎟⎝ ⎠

( )

( ) ( ) ( )

2

2 21/2, 1/2

2

1 cos cos,2 2

i

i ii

i e sen

iY e i e e sensensen

φ

φ φφ

θ θ

θ θθ φ θθθ

− −−

⎛ ⎞⎜ ⎟⎝ ⎠

⎛ ⎞ ⎛ ⎞= − +⎜ ⎟ ⎜ ⎟⎝ ⎠ ⎝ ⎠

( )

( ) ( )

21/2, 1/2

1/221/2, 1/2

1 cos 1 cos,2 2

, cos

i

i

Y esen sen

Y e sen

φ

φ

θ θθ φθ θ

θ φ θ θ

− −−

⎛ ⎞⎛ ⎞= − +⎜ ⎟⎜ ⎟

⎝ ⎠⎝ ⎠

= −

b. Aplicando L− a 1/2, 1/2 ( , )Y θ φ− , temos:

1/2, 1/2

1/2, 1/2

( , ) 0

cot ( , ) 0i

L Y

i e i Yφ

θ φ

θ θ φθ φ

− −

−−

=

⎛ ⎞∂ ∂− − − =⎜ ⎟∂ ∂⎝ ⎠

.

Considerando ainda que

21/2, 1/2 ( , ) ( )

i

Y e fφ

θ φ θ−

− ∝ , temos para a solução da equação diferencial:

( )

1/2, 1/2

2

cot ( , ) 0

cot 0

i

ii

i e i Y

i e i e f

φ

φφ

θ θ φθ φ

θ θθ φ

−−

−−

⎛ ⎞∂ ∂− − − =⎜ ⎟∂ ∂⎝ ⎠

⎛ ⎞∂ ∂− − − =⎜ ⎟∂ ∂⎝ ⎠

5

( )

( ) ( ) ( ) ( )

2

2 2

3 32 2

cot 0

cot 0

cot 02

1 cot 02

ii

i ii i

i i

i e i e f

i e e i f i e f e

f ie i e f

f f

φφ

φ φφ φ

φ φ

θ θθ φ

θ θ θθ φ

θθ

θθ

−−

− −− −

− −

⎛ ⎞∂ ∂− − − =⎜ ⎟∂ ∂⎝ ⎠

⎛ ⎞ ⎛ ⎞∂ ∂⎛ ⎞− − + − − =⎜ ⎟ ⎜ ⎟⎜ ⎟∂ ∂⎝ ⎠ ⎝ ⎠⎝ ⎠∂ −⎛ ⎞− + =⎜ ⎟∂ ⎝ ⎠

∂− + =∂

Resolvendo para f e θ , temos:

1 cot 02

1 cot2

1ln ln2

f f

ff

f sen

θθ

θ θ

θ

∂− + =∂

∂= ∂

= −

ou

( ) 1/2f senθ −= . Temos que a resposta final é:

(a) ( ) ( ) 1/221/2, 1/2 , cos

iY e sen

φ

θ φ θ θ− −

− = −

(b) 1/221/2, 1/2 ( )

i

Y e senφ

θ− −

− ∝ . Comparando as equações (a) e (b), podemos perceber os resultados contraditórios. Então, isto é um outro argumento contra o valor semi-inteiro l para o momento angular orbital.

1

Problema 3.18 Considere um autoestado do momento angular orbital 2, 0l m= = . Suponha que este estado seja rotacionado por um ângulo β em torno do eixo- y . Encontre a probabilidade para o novo estado ser achado em 0m = , 1± e 2± . (Os harmônicos esféricos para 0l = , 1 e 2 dados no Apêndice A podem ser úteis.)

Figura 1: Ângulos azimutal e polar que caracteriza n̂ . Solução : A partir da (3.6.46), temos:

ˆ ˆ( )n D R z= .

2

Também, da (3.6.48), temos:

ˆ ˆ( ) , ,l m

n D R l m l m z=∑∑ .

Podemos ainda escrever como:

( )'

' '( ) , , ' , ' ( ) , , ' ( )l

m mm m

D R l m l m l m D R l m l m D R= =∑ ∑ .

Considerando 0m = inicialmente, temos para uma rotação arbitraria ( )D R , a probabilidade como:

22 ( ),0, ( ) ,0 l

ml m D R l D= . Equação (3.6.52) nos fornece ( )

,0l

mD :

( ) ( )*( )0

,

4, , 0 ,(2 1)

l mm lD Y

l θ β φ α

πα β γ θ φ= =

= =+

.

Com 2l = , obtemos os seguintes resultados para 0m = :

* 22(2) 000 2

4 ( ,0)5

D Yπ β=

Para o harmônico esférico, temos:

( )0 22

5 3cos 116

Y θπ

= − .

Substituindo, temos:

3

( )

( )

( )

22(2) 2

00

22(2) 200

22(2) 200

4 5 3 15 16

4 5 3 15 16

1 3 14

D cos

D cos

D cos

π βπ

π βπ

β

= −

= −

= −

.

Com 2l = , obtemos os seguintes resultados para 1m = ± :

* 22(2) 110 2

4 ( ,0)5

D Yπ β±± =

Para o harmônico esférico, temos:

( )12

15 cos8

iY sen e φβ βπ

± ±= ∓ .

Substituindo, temos:

( )

( )

( )

22(2)

10

2 2(2)10

2 2(2)10

4 15 cos5 8

4 15 cos5 8

3 cos2

iD sen e

D sen

D sen

φπ β βπ

π β βπ

β β

±±

±

±

=

=

=

.

Com 2l = , obtemos os seguintes resultados para 2m = ± :

* 22(2) 220 2

4 ( ,0)5

D Yπ β±± =

Para o harmônico esférico, temos:

( )2 2 22

1532

iY sen e φβπ

± ±= .

4

Substituindo, temos:

( )

( )

( )

β

β

βπ

π

βπ

π φ

42)2(20

222)2(20

222)2(20

2

222)2(20

8383

3215

54

3215

54

senD

senD

senD

esenD i

=

=

=

=

±

±

±

±±

.

Podemos agora mostrar que a probabilidade total somada sobre 'm deve ser unitária. Considerando,

0m = → ( )22(2) 200

1 3 14

D cos β= −

1m = ± → ( )2 2(2)

103 cos2

D senβ β± =

2m = ± → β42)2(

20 83 senD =±

temos:

( ) ( )

( )

( )

2 2 2 2 2(2) (2) (2) (2) (2)00 10 10 20 20

2 22 4

4 2 2 2 4

4 2 2 2 4

4 2 2 2 4

1 3 33 1 2 cos 24 2 81 39cos 1 6cos 3 cos4 49 1 6 3cos cos 3 cos4 4 4 49 1 6 3cos cos 3 14 4 4 4

P D D D D D

P cos sen sen

P sen sen

P sen sen

P sen sen sen

β β β β

β β β β β

β β β β β

β β β β

+ − + −= + + + +

= − + +

= + − + +

= + − + +

= + − + − + β

.

5

4 2 2 4 4

4 2 2 4

4 2 2 2 2

4 2 2 4 2

4

9 1 6 3cos cos 3 34 4 4 49 1 6 9cos cos 34 4 4 49 1 6 9cos cos 3(1 cos ) (1 cos )4 4 4 49 1 6 9cos cos 3 3cos (1 cos 2cos )4 4 4 49 cos4

P sen sen sen

P sen sen

P

P

P

β β β β β

β β β β

β β β β

β β β β β

β

= + − + − +

= + − + −

= + − + − − −

= + − + − − + −

= 2 2 41 6 9 9cos 3 3cos cos4 4 4 4

β β β+ − + − − − 2

2 2 2

2

9 cos2

1 6 9 9cos 3 3cos cos4 4 4 21 9 6 93 3 cos4 4 4 2

1

P

P

P

β

β β β

β

+

= − + − − +

⎛ ⎞ ⎛ ⎞= + − + − − +⎜ ⎟ ⎜ ⎟⎝ ⎠ ⎝ ⎠

=

1

Problema 3.19 Qual é o significado físico dos operadores

† †K a a+ + −≡ e K a a− + −≡ no esquema de Schwinger para o momento angular? Dê os elementos da matriz não nulos de K± . Solução : O operador K+ , atuando sobre o ket ,n n+ − , resulta em:

( )( )† †, , 1 1 1, 1K n n a a n n n n n n+ + − + − + − + − + −= = + + + + (1) e

, , 1, 1K n n a a n n n n n n− + − + − + − + − + −= = − − (2) a+ e †a+ são os operadores aniquilação e criação, respectivamente. Logo, os operadores K+ ( K− ) criam (destroem) duas partículas de “spins opostos”, não alterando o momento angular total. Considere também que

2−+ +=

nnj (3)

e

2−+ −=

nnm , (4)

e também que

mjnn ,, →−+ . (5)

2

Então, equação (1) pode ser escrita como

( )( ) mjmjmjmjK ,11(1(, ++−+++ (6) e

( )( ) mjmjmjmjK ,1((, −−+− (7) isto é, +K e −K , são os operadores de levantamento e abaixamento para

2−+ +=

nnj onde −+ + nn correspondem ao numero total de “partículas” de

spin ½. Os elementos da matriz destes dois operadores são dados por:

( )( ) mmjjmjmjmjKmj ,'1,'11,',' δδ ++ +−++= (8) e

( )( ) mmjjmjmjmjKmj ,'1,',',' δδ −− −+= (9)

1

Problema 3.2 Considere uma matriz 2 2× definida por

0

0

a i aUa i a

σσ

+=

−ii

,

onde 0a é um número real e a é um vetor tridimensional com componentes reais. a. Prove que U é unitário e unimodular. b. Em geral, uma matriz unimodular untária 2 2× representa uma rotação em três dimensões. Encontre os eixos e o ângulo de rotação apropriados para U em termos de 0a , 1a , 2a e 3a . Solução: a. Primeiramente, vamos reescrever U como:

( )( )

( )

10 0

1†

U a i a a i a

U A A

σ σ −

= + −

=

i i (1)

........................................................................................................................... Lembre-se:

† *A A= ........................................................................................................................... Então, temos para †UU a seguinte expressão:

( ) ( )1 1† † 1 † † †UU A A A A A AA A− −−= = , (2)

ou

† †2 2 2 20 1 2 3

1 1UU A Aa a a a

= =+ + +

(3)

2

........................................................................................................................... Lembre-se: ( )† † †XY Y X=

( ) ( )† †1 1† † † † 1 †U A A A A A A

− − −⎡ ⎤ ⎡ ⎤= = =⎢ ⎥ ⎢ ⎥⎣ ⎦ ⎣ ⎦

........................................................................................................................... Da mesma forma, podemos mostrar também que † 1U U = .

( ) 1† 1 † †U U A A A A−−= , (4)

Como A e †A comutam, temos que:

( )( ) ( )

1† 1 † †

1† 1 † †

† 1

U U A A A A

U U A A A A

U U

−−

−−

=

⎡ ⎤= ⎢ ⎥⎣ ⎦=

(5)

........................................................................................................................... Demonstração: Calculando †AA :

( )( )( ) ( ) ( )

†0 0

2† 2 20 0 0

2† 20

† 2 2 2 20 1 2 3

AA a i a a i a

AA a a i a i a a i a

AA a a

AA a a a a

σ σ

σ σ σ

= + −

= − + −

= +

= + + +

i i

i i i

3

Calculando †A A :

( )( )( ) ( ) ( )

†0 0

2† 2 20 0 0

2† 20

† 2 2 2 20 1 2 3

A A a i a a i a

A A a a i a i a a i a

A A a a

A A a a a a

σ σ

σ σ σ

= − +

= + − −

= +

= + + +

i i

i i i

........................................................................................................................... Ou seja, provamos que U é unitário. Vamos provar agora que U é unimodular. ........................................................................................................................... Lembrar:

1

0 11 0

σ⎛ ⎞

= ⎜ ⎟⎝ ⎠

2

00i

−⎛ ⎞= ⎜ ⎟⎝ ⎠

3

1 00 1

σ⎛ ⎞

= ⎜ ⎟−⎝ ⎠

1 2 3

ˆˆ ˆa a i a j a k= + + ( )2 2.a aσ = (3.2.41) ........................................................................................................................... Primeiramente:

3 1 2

1 2 3

a a iaa

a ia aσ

−⎛ ⎞= ⎜ ⎟+ −⎝ ⎠

i , (6)

então

0 3 1 20

1 2 0 3

a ia ia aA a i a

ia a a iaσ

+ +⎛ ⎞= + = ⎜ ⎟− −⎝ ⎠

i (7)

4

0 3 1 2†0

1 2 0 3

a ia ia aA a i a

ia a a iaσ

− − −⎛ ⎞= − = ⎜ ⎟− + +⎝ ⎠

i (8)

0 3 1 2† 1

1 2 0 3

a ia ia aA

ia a a ia− − − +⎛ ⎞= ⎜ ⎟− − +⎝ ⎠

(9)

........................................................................................................................... Lembrar:

1TA A−= ........................................................................................................................... Temos também que:

† 2 2 2 20 1 2 3DetA DetA a a a a= = + + + (10)

enquanto que:

( )( ) ( )1† † † † 1 1Det A A DetA Det A− −= = , (11)

ou seja, ambos os determinantes, ( )( )1† †Det A A

− e ( )( )1† †Det A A− , são iguais.

Portanto, temos que:

( ) ( )† 1

2 2 2 2†0 1 2 3

1 1Det Aa a a aDet A

− = =+ + +

(12)

Então:

† 1 † 1( ) ( ) 1DetU Det A A DetADet A− −⎡ ⎤= = =⎣ ⎦ , (13) portanto U é unimodular.

5

b. Desde que

† † 2 2 2 20 1 2 3 1AA A A a a a a α= = + + + = (14)

........................................................................................................................... Demonstração:

( )( )( ) ( ) ( )

†0 0

2† 2 20 0 0

2† 20

† 2 2 2 20 1 2 3

AA a i a a i a

AA a a i a i a a i a

AA a a

AA a a a a

σ σ

σ σ σ

= + −

= − + −

= +

= + + +

i i

i i i

........................................................................................................................... temos que:

( ) 1†

2

220 0 3 0 2 0 1

220 2 0 1 0 0 3

2 2 212 2 2

U A A

A AUA AAU

a a ia a a a ia aU

a a ia a a a ia a

α

α

−=

=

=

⎛ ⎞− + +⎜ ⎟=⎜ ⎟− + − −⎝ ⎠

(15)

........................................................................................................................... Demonstração:

0 3 1 2

1 2 0 3

a ia ia aA

ia a a ia+ +⎛ ⎞

= ⎜ ⎟− −⎝ ⎠

0 3 1 2 0 3 1 22

1 2 0 3 1 2 0 3

220 0 3 0 2 0 12

220 2 0 1 0 0 3

2 2 2

2 2 2

a ia ia a a ia ia aA

ia a a ia ia a a ia

a a ia a a a ia aA

a a ia a a a ia a

+ + + +⎛ ⎞⎛ ⎞= ⎜ ⎟⎜ ⎟− − − −⎝ ⎠⎝ ⎠⎛ ⎞− + +⎜ ⎟=⎜ ⎟− + − −⎝ ⎠

6

Elemento 1.1:

( )( ) ( )( )( ) ( )

11 0 3 0 3 1 2 1 2

2 2 2 2 2 211 0 0 3 3 0 3 1 2 1 2 1 2

2 2 2 211 0 0 3 3 1 2

2211 0 0 3

2

2

E a ia a ia ia a ia a

E a a ia ia a i a i a a ia ia a a

E a ia a a a a

E a a ia a

= + + + + −

= + + + + − + −

= + − − −

= − +

........................................................................................................................... Comparando com as equações (3.3.7) e (3.3.10),

220 0 3 0 2 0 1

220 2 0 1 0 0 3

2 2 21( , )* * 2 2 2

a a ia a a a ia aa bU a b

b a a a ia a a a ia aα

⎛ ⎞− + +⎛ ⎞ ⎜ ⎟= =⎜ ⎟ ⎜ ⎟− − + − −⎝ ⎠ ⎝ ⎠ (16)

Re( ) cos2

a φ⎛ ⎞= ⎜ ⎟⎝ ⎠

Im( )2za n sen φ⎛ ⎞= − ⎜ ⎟

⎝ ⎠ (17)

Re( )2yb n sen φ⎛ ⎞= ⎜ ⎟

⎝ ⎠ Im( )

2xb n sen φ⎛ ⎞= − ⎜ ⎟⎝ ⎠

(18)

temos para o ângulo e os eixos de rotação apropriados para U :

( )2 20cos

2

a aφα

−= (19)

........................................................................................................................... Demonstração: Considerando que:

Re( ) cos2

a φ⎛ ⎞= ⎜ ⎟⎝ ⎠

22

0Re( )a a

aα−

=

7

temos:

( )220

cos2

a aφα

−=

...........................................................................................................................

022

a asenφ

α= (20)

1

xana

= − (21)

2

yana

= − (22)

3

zana

= − (23)

1

Problema 3.20 Adicione momento angular 1 1j = e 2 1j = para formar os estados 2j = , 1 e 0 . Use um ou outro método, do operador escada ou da relação de recursão, e expresse todos autokets { },j m (nove) em termos de 1 2 1 2;j j m m . Escreva sua resposta como

1 11, 1 0 0 ,...,2 2

j m= = = + − +

em que + e 0 significam 1,2 1,0m = , respectivamente. Solução: Vamos adicionar momento angular com 1 1j = e 2 1j = para formar estados com valores para o momento angular total igual a 2,1,0j = . ........................................................................................................................... Lembre-se: Visualizando J como a soma vetorial, temos:

1 2 1 2j j j j j− ≤ ≤ + (3.7.38) Logo, podemos ter estados com 2,1,0j = . ........................................................................................................................... Nosso objetivo é expressar todos os nove (9) autokets { },j m em termos de

1 2 1 2,j j m m .

2

........................................................................................................................... Lembre-se: A dimensionalidade do espaço é dada por:

1 2(2 1)(2 1)N j j= + + (3.7.40) Para o caso especifico de 1 1j = e 2 1j = , temos:

9N = (1) ........................................................................................................................... ........................................................................................................................... Lembre-se: Da seção relativa a adição de momento angular (seção 3.7 - página 205), podemos expandir um ket qualquer correspondente a um estado de spin arbitrário em termos dos autokets de 2S e zS ou dos autokets de 1zS e 2zS . Os autovalores associados aos operadores acima são:

( )( )

22 21 2

1 2

1 1

2 2

( 1)

z Z z

z

z

S S S s s

S S S m

S m

S m

= + +

= + (2)

Portanto, temos duas possibilidades: I) Representação baseada em { }1 2,m m , associada aos autokets de 1zS e

2zS : + + , + − , − + e − − (3)

3

II) Representação baseada em { },s m , ou representação tripleto-singleto, associada aos autokets de 2S e zS :

1, 1,0s m= = ± e 0, 0s m= = . (4) É importante ressaltar ainda que existe uma relação entre as duas representações:

( )

( )

1, 1

11, 02

1, 1

10, 02

s m

s m

s m

s m

= = = + +

⎛ ⎞= = = + − + − +⎜ ⎟⎝ ⎠

= = − = − −

⎛ ⎞= = = + − − − +⎜ ⎟⎝ ⎠

(5)

Para o nosso caso em particular devemos ter algo deste tipo:

,j m (6) e

1 2,m m (7) e relações entre as representações, tal como:

1 11, 1 0 02 2

j m= = = + − + (8)

ou

1 2 1 21 11, 1 1, 0 0, 12 2

j m m m m m= = = = = − = = (9)

...........................................................................................................................

4

Considerando

1 1j = → 1 1m = ± (10) ,

2 1j = → 2 1m = ± (11) temos para os casos mais simples:

2, 2j m= = = + + ou 1 22, 2 1, 1j m m m= = = = = (12) e

2, 2j m= = − = − − . ou 1 22, 2 1, 1j m m m= = − = = − = − (13) Vamos agora utilizar o método do operador escada. Primeiramente, vamos lembrar a soma vetorial:

1 2J J J− − −= + . (14) E também, devemos nos lembrar da (3.5.40):

( )( ), 1 , 1J j m j m j m j m− = + − + − , (3.5.40) fazendo 1= por conveniência. Então, utilizando as informações acima, temos:

( )1 2 1 2 1 22, 2 4 2, 1 1, 1; 1, 1 2 0 2 0J j m j m J J j j m m− − −= = = = = = + = = = = = + + +

isto é,

( )12, 1 0 02

j m= = = + + + . (16)

5

........................................................................................................................... Demonstração: Considerando a equação (3.5.40),

( )( ), 1 , 1J j m j m j m j m− = + − + − , (3.5.40) podemos obter a expressão:

( )( ) ( )( )2, 2 1 2, 1 2 2 2 2 1 2, 1 4 2, 1J j m j m j m j m j m j m− = = = + − + = = = + − + = = = = =

Por outro lado, considerando novamente a expressão (3.5.40), temos: ( ) ( )1 2 1 2 1 2 1 2 1 2 1 21, 1; 1, 1 0 0J J j j m m J J J J a a− − − − − −+ = = = = = + + + = + + + + + = + + +

Calculando a constante 1a :

1 1 0J a− + + = + . (19) Para isto, vamos considerar que 1j = e 1 1m = . Então:

( )( ) ( )( )1 1 1 1 1 10 1 0 1 1 1 1 1 0 2 0J a j m j m− + + = + = + − + + = + − + + = + , (20) fornecendo 1 2a = . Da mesma forma, podemos calcular 2a , e mostrar também que 2 2a = . ........................................................................................................................... Agora

( ) ( )1 212, 1 6 2, 0 0 0 2 002

J j m j m J J− − −= = = = = = + + + + = − + + + + − .

6

........................................................................................................................... Demonstração:

( )( ), 1 , 1J j m j m j m j m− = + − + − , (3.5.40) Aplicando

( ) ( ) ( )1 2 1 2 3 41 10 0 00 002 2

J J c c c c− −⎛ ⎞+ + + + = − + + + + + −⎜ ⎟⎝ ⎠

. (22)

Vamos agora calcular as constantes 1c , 2c , 3c e 4c utilizando a (3.5.40). CÁLCULO DE 1c Devemos considerar que 1 1j = e 1 0m = . Calculando temos:

( )( )1 1 1 2 1 1 1 10 0, 1 ( )( 1) 1 0 1 0 1 2J J m m j m j m− −+ = = = + = + − + − + = + − + − + = − +

com

1 2c = . (24) CÁLCULO DE 2c Devemos considerar que 1 1j = e 1 1m = + . Calculando temos:

( )( )1 1 1 2 1 1 1 10 1, 0 ( )( 1) 00 1 1 1 1 1 00 2 00J J m m j m j m− −+ = = + = = + − + = + − + =

com

2 2c = . (26)

7

CÁLCULO DE 3c Devemos considerar que 2 1j = e 2 1m = + . Calculando temos:

( )( )2 2 1 2 2 2 2 20 0, 1 ( )( 1) 00 1 1 1 1 1 00 2 00J J m m j m j m− −+ = = = + = + − + = + − + =

com

3 2c = . (28) CÁLCULO DE 4c Devemos considerar que 2 1j = e 2 0m = . Calculando temos:

( )( )2 2 1 2 2 2 2 20 1, 0 ( )( 1) 1 0 1 0 1 2J J m m j m j m− −+ = = + = = + − + + − = + − + + − = + −

com

4 2c = . (30) ........................................................................................................................... Então

( )12, 0 2 006

j m= = = − + + + + − . (31)

Também,

( )12, 0 6 2, 1 2 0 2 2 0 2 2 0 2 06

J j m j m− = = = = = − = − + − + − + − .

8

Portanto,

( )12, 1 0 02

j m= = − = − + − . (33)

........................................................................................................................... Demonstração:

( )( ), 1 , 1J j m j m j m j m− = + − + − , (3.5.40) Aplicando a equação acima, temos:

12, 0 2, 1J j m b j m− = = = = = − (34) CÁLCULO DE 1b Devemos considerar que 2j = e 0m = . Calculando temos:

2, 0 ( )( 1) 2, 1 (2 0)(2 0 1) 2, 1 6 2, 1J j m j m j m j m j m j m− = = = + − + = = − = + − + = = − = = = −

com

1 6b = . (36) Por outro lado, temos ainda que:

( ) ( )1 2

1

12, 0 2 006

12, 06

J j m J J

J j m J

− − −

− −

= = = + − + + + + −

= = = − + 1 1 2 2 22 00 2 00J J J J J− − − − −+ + + − + − + + + + −( )( )

( )

1 1 2 2

1 2 3 4

12, 0 2 00 2 006

12, 0 2 0 0 0 2 06

J j m J J J J

J j m g g g g

− − − − −

= = = + + − + − + +

= = = − + − + − + −

9

CÁLCULO DE 1g Devemos considerar que 1 1j = e 1 0m = . Calculando temos:

1 100 0 (1 0)(1 0 1) 0 2 0J g− = − = + − + − = − (38)

com

1 2g = . (39) CÁLCULO DE 2g Devemos considerar que 1 1j = e 1 1m = . Calculando temos:

1 2 0 (1 1)(1 1 1) 0 2 0J g− + − = − = + − + − = − (40)

com

2 2g = . (41) CÁLCULO DE 3g Devemos considerar que 2 1j = e 2 1m = . Calculando temos:

2 3 0 (1 1)(1 1 1) 0 2 0J g− − + = − = + − + − = − (42)

com

3 2g = . (43)

10

CÁLCULO DE 4g Devemos considerar que 2 1j = e 2 0m = . Calculando temos:

2 400 0 (1 0)(1 0 1) 0 2 0J g− = − = + − + − = − (44)

com

4 2g = . (45) Substituindo o valor das constantes, a expressão se torna:

( )

( )

( )

( )

( )

1 2 3 412, 0 2 0 0 0 2 06

12, 0 2 2 0 2 0 2 0 2 2 06

12, 0 3 2 0 3 2 06

3 22, 0 0 06

32, 0 0 03

J j m g g g g

J j m

J j m

J j m

J j m

= = = − + − + − + −

= = = − + − + − + −

= = = − + −

= = = − + −

= = = − + −

(46)

No entanto,

( )

( )

( )

32, 0 6 2, 1 0 03

32, 1 0 06 332, 1 0 018

J j m j m

j m

j m

− = = = = = − = − + −

= = − = − + −

= = − = − + −

(47)

( )

( )

2

32, 1 0 03 2

12, 1 0 02

j m

j m

= = − = − + −

= = − = − + −

...........................................................................................................................

11

Para os estados 1j = , vamos considerar que

1, 1 0 0j m a b= = = + + + (48) com a condição de normalização

2 2 1a b+ = . (49) Desde que

2, 1 1, 1 0j m j m= = = = = (50) devido a ortogonalidade, temos que:

0a b+ = . (51) ........................................................................................................................... Demonstração: Temos:

1, 1 0 0j m a b= = = + + + (52) e

( )12, 1 0 02

j m= = = + + + → ( )12, 1 0 02

j m= = = + + + (53)

Fazendo o produto escalar, temos:

2, 1 1, 1 0 0 0 0 02 2

0 0 0 0 02 2

0

a bj m j m

a b

a b

= = = = = + + + + + =

+ + + + + =

+ =

(54)

...........................................................................................................................

12

Portanto, podemos escrever

( )11, 1 0 02

j m= = = + − + . (55)

Aplicando agora

1 2J J J− − −= + (56) aos dois lados respectivamente, temos:

( )11, 02

j m= = = + − − − + . (57)

........................................................................................................................... Demonstração: Vamos agora aplicar

( )( ), 1 , 1J j m j m j m j m− = + − + − (3.5.40) a equação

( )11, 1 0 02

j m= = = + − + . (58)

Temos:

( )

( )( )1 2

1 1 2 2

1 2 3 4

11, 1 0 02

11, 0 0 02

11, 0 0 0 0 02

11, 0 00 002

J j m J

j m J J

j m J J J J

j m d d d d

− −

− −

− − − −

⎡ ⎤= = = + − +⎢ ⎥⎣ ⎦

⎛ ⎞ ⎡ ⎤= = = + + − +⎜ ⎟ ⎣ ⎦⎝ ⎠⎛ ⎞ ⎡ ⎤= = = + − + + + − +⎜ ⎟ ⎣ ⎦⎝ ⎠⎛ ⎞ ⎡ ⎤= = = − − + + + − −⎜ ⎟ ⎣ ⎦⎝ ⎠

. (59)

13

CÁLCULO DE 1d Devemos considerar que 1 1j = e 1 1m = + . Calculando temos:

( )( )1 1 1 2 1 1 1 10 1, 0 ( )( 1) 00 1 1 1 1 1 00 2 00J J m m j m j m− −+ = = + = = + − + = + − + =

com

1 2d = . (61) CÁLCULO DE 2d Devemos considerar que 1 1j = e 1 0m = . Calculando temos:

( )( )1 1 1 2 1 1 1 10 0, 1 ( )( 1) 1 0 1 0 1 2J J m m j m j m− −+ = = = + = + − + − + = + − + − + = − +

com

2 2d = . (63) CÁLCULO DE 3d Devemos considerar que 2 1j = e 2 0m = . Calculando temos:

( )( )2 2 1 2 2 2 2 20 1, 0 ( )( 1) 1 0 1 0 1 2J J m m j m j m− −+ = = = = + − + + − = + − + + − = + −

com

3 2d = . (65)

14

CÁLCULO DE 4d Devemos considerar que 2 1j = e 2 1m = + . Calculando temos:

( )( )2 2 1 2 2 2 2 20 0, 1 ( )( 1) 00 1 1 1 1 1 00 2 00J J m m j m j m− −+ = = = + = + − + = + − + =

com

4 2d = . (67) Finalmente, substituindo as constantes:

1 2 3 411, 0 00 002

11, 0 2 00 2 2 2 002

1, 0

1, 0

j m d d d d

j m

j m

j m

⎛ ⎞ ⎡ ⎤= = = − − + + + − −⎜ ⎟ ⎣ ⎦⎝ ⎠⎛ ⎞ ⎡ ⎤= = = − − + + + − −⎜ ⎟ ⎣ ⎦⎝ ⎠⎡ ⎤= = = − − + + + −⎣ ⎦⎡ ⎤= = = + − − − +⎣ ⎦

Falta 12

(68)

........................................................................................................................... E similarmente, temos:

( )11, 1 0 02

j m= = − = − − − . (69)

........................................................................................................................... Demonstração: Vamos agora aplicar

( )( ), 1 , 1J j m j m j m j m− = + − + − (3.5.40)

15

a equação

( )11, 02

j m= = = + − − − + . (70)

Temos:

( )

( )( )1 2

1 1 2 2

1 1

11, 02

11, 12

11, 12

11, 1 02

J j m J

j m J J

j m J J J J

j m e J

− −

− −

− − − −

⎡ ⎤= = = + − − − +⎢ ⎥⎣ ⎦

⎛ ⎞ ⎡ ⎤= = − = + + − − − +⎜ ⎟ ⎣ ⎦⎝ ⎠⎛ ⎞ ⎡ ⎤= = − = + − − − + + + − − − +⎜ ⎟ ⎣ ⎦⎝ ⎠⎛ ⎞= = − = − − − +⎜ ⎟⎝ ⎠

2J −+ + − 2

1 2

0

11, 1 0 02

e

j m e e

⎡ ⎤− −⎣ ⎦

⎛ ⎞ ⎡ ⎤= = − = − − −⎜ ⎟ ⎣ ⎦⎝ ⎠

. (71)

CÁLCULO DE 1e Devemos considerar que 1 1j = e 1 1m = + . Calculando temos:

( )( )1 1 1 2 1 1 1 11, 1 ( )( 1) 0 1 1 1 1 1 0 2 0J J m m j m j m− −+ − = = + = − = + − + − = + − + − = −

com

1 2e = . (73) CÁLCULO DE 2e Devemos considerar que 2 1j = e 2 1m = + . Calculando temos:

16

( )( )2 2 1 2 2 2 2 21, 1 ( )( 1) 0 1 1 1 1 1 0 2 0J J m m j m j m− −− + = = − = + = + − + − = + − + − = −

com

2 2e = . (75) Finalmente, substituindo as constantes:

1 211, 1 0 02

11, 1 2 0 2 02

1, 1 0 0

j m e e

j m

j m

⎛ ⎞ ⎡ ⎤= = − = − − −⎜ ⎟ ⎣ ⎦⎝ ⎠⎛ ⎞ ⎡ ⎤= = − = − − −⎜ ⎟ ⎣ ⎦⎝ ⎠⎡ ⎤= = − = − − −⎣ ⎦

(76)

........................................................................................................................... Finalmente, podemos escrever

0, 0 00j m α β γ= = = + − + + − + , (77) onde podemos determinar α , β e γ por normalização, isto é,

2 2 2 1α β γ+ + = (78) e ortogonalidade para

1, 0j m= = (79) e

2, 0j m= = . (80) Escolhendo α , β e γ serem reais por convenção, temos que

17

( )10, 0 003

j m= = = + − − + − + . (81)

Portanto, os nove estados são:

0j =

( )10, 0 003

j m= = = + − − + − + (74)

1j =

( )11, 1 0 02

j m= = = + − + (55)

( )11, 02

j m= = = + − − − + (57)

( )11, 1 0 02

j m= = − = − − − (69)

2j =

2, 2j m= = = + + (12)

( )12, 1 0 02

j m= = = + + + (16)

( )12, 0 2 006

j m= = = − + + + + − (31)

18

( )12, 1 0 02

j m= = − = − + − (33)

2, 2j m= = − = − − (13)

1

Problema 3.21 a. Calcule

( ) 2( )'

jj

mmm j

d mβ=−∑

para algum j (inteiro ou semi-inteiro); então verifique a sua resposta para

1/ 2j = . b. Prove, para algum j ,

( ) ( )22 ( ) 2 2 2'

1 1( 1) ' 3cos 12 2

jj

m mm j

m d j j sen mβ β β=−

= + + −∑ .

[Dica: Isto pode ser provado de muitas maneiras. Você pode, por exemplo, examinar as propriedades rotacionais de 2

zJ usando a linguagem do tensor esférico (irredutível).] Solução: a. Primeiramente, vamos relembrar as equações (3.5.50) e (3.5.51):

( )

( ) ( )

( )'

'( )'

, , , ' exp exp exp ,

, , , ' exp ,

yj z zm m

yi m mjm m

iJiJ iJD j m j m

iJD e j m j mα γ

βα γα β γ

βα β γ − +

−⎛ ⎞− −⎛ ⎞ ⎛ ⎞= ⎜ ⎟⎜ ⎟ ⎜ ⎟⎝ ⎠ ⎝ ⎠⎝ ⎠

−⎛ ⎞= ⎜ ⎟

⎝ ⎠

(3.5.50)

e

( )( )' , ' exp ,yj

m m

iJd j m j m

ββ

−⎛ ⎞= ⎜ ⎟

⎝ ⎠ (3.5.51)

2

Com as equações acima em mente, temos que:

( ) ( ) ( )( )' 0, , 0 ' 'j

mmd jm D jm jm D R jmβ α β γ= = = = . (1) ........................................................................................................................... Lembre-se: Tensor esférico de ordem k (no lugar de l ) com número quântico magnético q (no lugar de m ),

( )k m qq l kT Y =

== (3.10.15) Temos também da equação (3.10.22a) que

*† ( ) ( ) ( )' '

'( ) ( ) ( )

kk k k

q qq qq k

D R T D R D R T=−

= ∑ . (3.10.22a)

........................................................................................................................... Aplicando ( )k

qT para zJ , e considerando que ele é um tensor de primeira ordem, com 0q = , isto é, (1)

0T , temos:

2† †1 1 1' ( ) ( ) ' ' ( ) ( ) ' ( ) 'j j

z zm j m j

jm D R J D R jm jm D R J jm jm D R jm jm D R jm m=− =−

= =∑ ∑ Similarmente, e desde que apenas ' 0q = contribui, temos:

( ) ( ) ( )* * 1/2(1) 1 (1) 00 ' ' 00 1

'

1 1' ' ' ' 4 / 2 1 , 0 ' 'cosq q zq

jm D T jm jm D R J jm l Y m mπ θ β φ β= = + = = =∑

3

........................................................................................................................... Lembre-se: Da equação (3.6.52) temos:

( )( ) *0

,

4, , 0(2 1)

l mm lD Y

lθ β φ α

πα β γ= =

= =+

(3.6.52)

0

13 cos

4Y θ

π= (3.10.16)

........................................................................................................................... Finalmente, das equações acima temos:

( ) 2( )' 'cos

jj

mmm j

d m mβ β=−

=∑ , (4)

ou

( ) ( )2 2( ) ( )1/2 ' 1/2 '

1 1 'cos2 2

j jm md d mβ β β−− + = (5)

Podemos confirmar os resultados acima para 1/ 2j = . Então, especificamente para 1/ 2j = , temos: ( ) ( )2 2(1/2) (1/2)

1/2 ' 1/2 '1 1 'cos2 2m md d mβ β β−− + = (6)

........................................................................................................................... Lembre-se: Relembrando a equação (3.5.52):

4

(1/2)

cos2 2

cos2 2

send

sen

β β

β β

⎛ ⎞⎛ ⎞ ⎛ ⎞−⎜ ⎟ ⎜ ⎟⎜ ⎟⎝ ⎠ ⎝ ⎠⎜ ⎟=

⎜ ⎟⎛ ⎞ ⎛ ⎞⎜ ⎟⎜ ⎟ ⎜ ⎟

⎝ ⎠ ⎝ ⎠⎝ ⎠

. (3.5.52)

........................................................................................................................... Temos dois casos a considerar: I) caso ' 1 / 2m = : ( ) ( )2 2(1/2) (1/2)

1/2,1/2 1/2,1/21 1 1 os2 2 2

d d cβ β β−− + = (7)

Considerando os elementos matriciais,

1/2

1/2

' 1/2 ' 1/2

(1/2)cos

2 2

cos2 2

m

m

m m

send

sen

β β

β β

=−

=+

=− =+⎛ ⎞⎛ ⎞ ⎛ ⎞⎜ ⎟−⎜ ⎟ ⎜ ⎟⎜ ⎟⎝ ⎠ ⎝ ⎠= ⎜ ⎟⎛ ⎞ ⎛ ⎞⎜ ⎟⎜ ⎟ ⎜ ⎟⎜ ⎟⎝ ⎠ ⎝ ⎠⎝ ⎠

, (8)

temos finalmente:

2 2

2 2

1 1 1cos cos2 2 2 2 2

cos cos2 2

sen

sen

β β β

β β β

− =

− =. (9)

........................................................................................................................... Lembre-se: cos( ) cos cosA B A B senAsenB+ = − (10) Para

2A B β= = , temos:

5

2 2

cos cos cos2 2 2 2 2 2

cos cos2 2

sen sen

sen

β β β β β β

β ββ

⎛ ⎞+ = −⎜ ⎟⎝ ⎠

= − (11)

........................................................................................................................... II) caso ' 1 / 2m = − : ( ) ( )2 2(1/2) (1/2)

1/2, 1/2 1/2, 1/21 1 1 os2 2 2

d d cβ β β− − −− + = − (12)

2 21 1 1cos cos

2 2 2 2sen ββ β− = − (13)

Multiplicando por (− ), temos:

2 21 1 1cos cos2 2 2 2

sen ββ β− + = , (14)

que é idêntico ao termo para o primeiro caso.

b. A partir da equação (3.5.51),

( )( )' , ' exp ,yj

m m

iJd j m j m

ββ

−⎛ ⎞= ⎜ ⎟

⎝ ⎠, (3.5.51)

com 1= , temos:

( )( )' ' yi Jj

m md jm e jmββ −= . (15) Agora

( ) 22 ( ) 2' ' 'y y

j ji J i Jj

m mm j m j

m d m jm e jm jm e jmβ ββ −

=− =−

=∑ ∑ (16)

6

Vamos olhar apenas para o segundo termo:

2 2 2 †' ' ' ' ' ( ) ( ) 'y y y yj

i J i J i J i Jz z z

m jjm e J jm jm e jm jm e J e jm jm D R J D R jmβ β β β− −

=−

= =∑ ........................................................................................................................... Lembre-se: Se nós examinarmos as propriedades rotacionais de 2

zJ usando a linguagem do tensor esférico, encontramos:

( )2 2 (2)0

13zJ J Y= + , (18)

em que 2J é um escalar sob rotação e (2)

0Y é um tensor esférico de ordem 2. ........................................................................................................................... Utilizando a relação acima, temos:

2 † 2 (2) †0

1 1( ) ( ) ( ) ( )3 3zD R J D R J D R Y D R= + (19)

........................................................................................................................... Lembre-se:

2 2 †( ) ( )y yi J i Jz ze J e D R J D Rβ β− = (20)

........................................................................................................................... com

2(2) † (2) (2)

0 '0 '' 2

( ) ( ) k kk

D R Y D R D Y=−

= ∑ ' ' 2q k= = − (21)

7

........................................................................................................................... Lembre-se:

*† ( ) ( ) ( )' '

'( ) ( ) ( )

kk k k

q qq qq k

D R T D R D R T=−

= ∑ . (3.10.22a)

( )k m q

q l kT Y === (3.10.15)

( ) † ( ) ( )

' ''

( ) ( ) ( )k

k k kq q q q

q kD R T D R D R T

=−

= ∑ . (3.10.22b)

........................................................................................................................... Portanto, a equação

( ) 22 ( ) 2' ' 'y y

j ji J i Jj

m mm j m j

m d m jm e jm jm e jmβ ββ −

=− =−

=∑ ∑ (22)

com

2 2 2 †' ' ' ' ' ( ) ( ) 'y y y yj

i J i J i J i Jz z z

m j

jm e J jm jm e jm jm e J e jm jm D R J D R jmβ β β β− −

=−

= =∑ juntamente com

2 † 2 (2) †0

1 1( ) ( ) ( ) ( )3 3zD R J D R J D R Y D R= + (24)

pode ser reescrita como:

( ) 22 ( ) 2 † 2 (2) †' 0

1 1' ( ) ( ) ' ' ( ) ( ) '3 3

jj

m m zm j

m d jm D R J D R jm jm J D R Y D R jmβ=−

⎡ ⎤= = +⎢ ⎥⎣ ⎦∑

( ) 22 ( ) 2 (2) † 2 (2) †' 0 0

1 1 1 1' ( ) ( ) ' ' ' ' ( ) ( ) '3 3 3 3

jj

m mm j

m d jm J D R Y D R jm jm J jm jm D R Y D R jmβ=−

⎡ ⎤= + = +⎢ ⎥⎣ ⎦∑

( )222 ( ) (2) (2)

' '0 '' 2

1 1( 1) ' '3 3

jj

m m k km j k

m d j j jm D Y jmβ=− =−

= + +∑ ∑

8

No último termo, apenas ' 0k = contribui, isto é,

( )2(2)00

3cos 1

2D

β −= . (28)

........................................................................................................................... Lembre-se:

( )( )00 0

(cos )lld P

ββ θ

== (3.6.53)

( )

( ) ( )

( )'

'( )'

, , , ' exp exp exp ,

, , , ' exp ,

yj z zm m

yi m mjm m

iJiJ iJD j m j m

iJD e j m j mα γ

βα γα β γ

βα β γ − +

−⎛ ⎞− −⎛ ⎞ ⎛ ⎞= ⎜ ⎟⎜ ⎟ ⎜ ⎟⎝ ⎠ ⎝ ⎠⎝ ⎠

−⎛ ⎞= ⎜ ⎟

⎝ ⎠

(3.5.50)

( )( )' , ' exp ,yj

m m

iJd j m j m

ββ

−⎛ ⎞= ⎜ ⎟

⎝ ⎠ (3.5.51)

........................................................................................................................... Portanto, temos:

( ) ( )

( )

( )

( )

22 ( ) (2) 2 2' 00

22 ( ) (2) 2 (2) 2' 00 00

22 ( ) 2 (2) (2)' 00 00

22 ( ) 2'

1 1( 1) ' 3 '3 3

1 1( 1) ' ' ' '3 3

1 ( 1)( 1) ' ' ' ' '3 3

3c1 ( 1) ' '3

jj

m m zm j

jj

m m zm j

jj

m mm j

jm m

m d j j jm D J J jm

m d j j jm D J jm jm D J jm

j jm d j j m jm D jm jm D jm

m d j j m jm

β

β

β

β

=−

=−

=−

= + + −

= + + −

+= + + −

= + +

( ) ( )2 2os 1 3cos 1( 1)' ' '2 3 2

j

m j

j jjm jm jmβ β

=−

− −+−∑

9

( ) ( ) ( )

( )

( ) ( )

2 222 ( ) 2

'

22 ( ) 2 2 2 2'

222 ( ) 2'

3cos 1 3cos 11 ( 1)( 1) ' ' ' ' '3 2 3 2

1 3 1 3 ( 1) ( 1)( 1) ' cos ' cos3 2 2 2 3 6

1 ( 1) ' 3 ( 1)( 1) 3cos 1 (13 6 2 2 3

jj

m mm j

jj

m mm j

jj

m mm j

j jm d j j m jm jm jm jm

j j j jm d j j m m

j j m j jm d j j

β ββ

β β β

β β

=−

=−

=−

− −+= + + −

+ += + + − − +

+ +⎡ ⎤= + + + − − −⎢ ⎥⎣ ⎦

( ) ( )

( ) ( )

( ) ( )

2

222 ( ) 2 2'

222 ( ) 2 2'

222 ( ) 2 2'

)

1 ' 1( 1) 3cos 1 ( 1)(1 )2 2 2

' 13cos 1 ( 1)2 2

1 '( 1) 3cos 12 2

jj

m mm j

jj

m mm j

jj

m mm j

sen

mm d j j j j sen

mm d j j sen

mm d j j sen

β

β β β

β β β

β β β

=−

=−

=−

= + + − − + −

= − + +

= + + −

1

Problema 3.22 a. Considere um sistema com 1j = . Explicitamente escrevemos

1, ' 1,yj m J j m= = na forma de uma matriz 3 3× . b. Mostre que para 1j = apenas, é legitimo trocar /yiJe β− por

2

1 (1 cos )y yJ Ji senβ β⎛ ⎞ ⎛ ⎞

− − −⎜ ⎟ ⎜ ⎟⎝ ⎠ ⎝ ⎠

.

c. Usando (b), prove

( )

( ) ( )

( ) ( )

( 1)

1 1 11 cos 1 cos2 22

1 1cos2 2

1 1 11 cos 1 cos2 22

j

sen

d sen sen

sen

β β β

β β β β

β β β

=

⎛ ⎞⎛ ⎞⎛ ⎞ ⎛ ⎞+ − −⎜ ⎟⎜ ⎟ ⎜ ⎟⎜ ⎟⎝ ⎠ ⎝ ⎠⎝ ⎠⎜ ⎟⎜ ⎟⎛ ⎞ ⎛ ⎞= −⎜ ⎟⎜ ⎟ ⎜ ⎟

⎝ ⎠ ⎝ ⎠⎜ ⎟⎜ ⎟⎛ ⎞⎛ ⎞ ⎛ ⎞⎜ ⎟− +⎜ ⎟ ⎜ ⎟⎜ ⎟⎜ ⎟⎝ ⎠ ⎝ ⎠⎝ ⎠⎝ ⎠

.

Solução: a. Vamos considerar a matriz

( , , ) ( ) ( ) ( )z y zD D D Dα β γ α β γ= ,

que para um j arbitrário:

( )'

( ) ( ' )'

( , , ) , ' exp exp exp ,

( , , ) , ' exp ,

yj z zm m

yj i m mm m

iJiJ iJD j m j m

iJD e j m j mα γ

βα γα β γ

βα β γ − +

−⎛ ⎞− −⎛ ⎞ ⎛ ⎞= ⎜ ⎟⎜ ⎟ ⎜ ⎟⎝ ⎠ ⎝ ⎠⎝ ⎠

−⎛ ⎞= ⎜ ⎟

⎝ ⎠

2

Note que apenas a parte não trivial é a rotação do meio em torno do eixo- y , o qual mistura diferentes valores de m . É conveniente definir uma nova matriz ( ) ( )jd β como

( )( )' , ' exp ,yj

m m

iJd j m j m

ββ

−⎛ ⎞≡ ⎜ ⎟

⎝ ⎠.

O caso mais simples é 1j = , o qual consideraremos em algum detalhe. Antes, devemos obter a representação matricial 3 3× de yJ . Como

( )2y

J JJ

i+ −−

= ,

........................................................................................................................... Demonstração:

x y

x y

J J iJ

J J iJ+

= +

= −

Subtraindo ambas as equações, temos:

( )2y

J JJi

+ −−=

........................................................................................................................... Portanto,

1' ' , ' ' , ' ' ,2yj m J j m j m J j m j m J j mi + −⎡ ⎤= −⎣ ⎦

Podemos usar a equação (3.5.39) e (3.5.40):

' ', 1' ' , ( )( 1) j j m mj m J j m j m j m δ δ± ±= ± +∓ ,

3

para obter

' ', 1 ' ', 1

, ' , ( 1) ( 1) ' , 1 ( 1) ( 1) ' , 12

, ' , ( 1) ( 1) ( 1) ( 1)2

y

y j j m m j j m m

j m J j m j j m m jm j m j j m m jm j mi

j m J j m j j m m j j m mi

δ δ δ δ+ −

⎡ ⎤= + − + + − + − − −⎣ ⎦

⎡ ⎤= + − + − + − −⎣ ⎦

Utilizando a expressão acima encontramos a matriz para ( 1)j

yJ = (3.5.54):

01 1

' 1

( 1)' 0

' 1

0 2 0

2 0 22

0 2 0

mm m

m

jmy

m

i

J i i

i

== =−

=

==

=−

⎛ ⎞−⎜ ⎟

⎜ ⎟⎛ ⎞= −⎜ ⎟⎜ ⎟⎝ ⎠⎜ ⎟⎜ ⎟⎝ ⎠

........................................................................................................................... Demonstração: Elemento 12 (linha 1, coluna 2), ou seja,

0' 1

mm==

para 1j = .

Temos:

' ', 1 ' ', 1

1'1 1',0 1 1'1 1',0 1

11 1,1 11 1, 1

, ' , ( 1) ( 1) ( 1) ( 1)2

1,1 1,0 1(1 1) 0(0 1) 1(1 1) 0(0 1)2

1,1 1,0 1(1 1) 0(0 1) 1(1 1) 0(0 1)2

y j j m m j j m m

y

y

j m J j m j j m m j j m mi

Ji

Ji

δ δ δ δ

δ δ δ δ

δ δ δ δ

+ −

+ −

⎡ ⎤= + − + − + − −⎣ ⎦

⎡ ⎤= + − + − + − −⎣ ⎦

= + − + − + − −

1,1 1,0 1(1 1) 0(0 1)2yJi

⎡ ⎤⎢ ⎥⎣ ⎦

⎡ ⎤= + − +⎣ ⎦

4

( )

21,1 1,02

1,1 1,0 22

y

y

Ji

J i

=

⎛ ⎞= −⎜ ⎟⎝ ⎠

...........................................................................................................................

b. Nosso próximo objetivo é fazer a expansão de Taylor de

exp yiJ β−⎛ ⎞⎜ ⎟⎝ ⎠

.

Pode-se mostrar que

3( 1) ( 1)j jy yJ J= =⎛ ⎞

=⎜ ⎟⎜ ⎟⎝ ⎠

.

........................................................................................................................... Demonstracao :

( )

( )

( )

( )

2( 1)

2 222( 1) 2

2 2

22( 1)

2( 1) 2

0 2 0 0 2 0

2 0 2 2 0 22 2

0 2 0 0 2 0

2 0 20 4 0

42 0 2

2 0 20 4 0

42 0 2

1/ 2 0 1/ 20 1 01/ 2 0 1/ 2

jy

jy

jy

jy

i i

J i i i i

i i

i iJ i

i i

J

J

=

=

=

=

⎛ ⎞ ⎛ ⎞− −⎜ ⎟ ⎜ ⎟

= − −⎜ ⎟ ⎜ ⎟⎜ ⎟ ⎜ ⎟⎜ ⎟ ⎜ ⎟⎝ ⎠ ⎝ ⎠⎛ ⎞−⎜ ⎟

= −⎜ ⎟⎜ ⎟−⎝ ⎠

−⎛ ⎞⎜ ⎟= ⎜ ⎟⎜ ⎟−⎝ ⎠

−⎛ ⎞⎜ ⎟= ⎜ ⎟⎜ ⎟−⎝ ⎠

...........................................................................................................................

........................................................................................................................... Demonstração:

5

Para 3( 1)j

yJ =⎛ ⎞⎜ ⎟⎜ ⎟⎝ ⎠

, temos:

3( 1)

3

3( 1)

0 2 0 0 2 0 0 2 01 2 0 2 2 0 2 2 0 2

2 2 20 2 0 0 2 0 0 2 0

0 2 0 0 2 0 0 2 01 2 0 2 2 0 2 2 0 28

0 2 0 0 2 0 0 2 0

jy

jy

i i iJ

i i i i i i

i i i

i i iJ

i i i i i i

i i i

=

=

⎛ ⎞ ⎛ ⎞ ⎛ ⎞− − −⎜ ⎟ ⎜ ⎟ ⎜ ⎟⎛ ⎞

= − − −⎜ ⎟ ⎜ ⎟ ⎜ ⎟⎜ ⎟⎜ ⎟ ⎜ ⎟ ⎜ ⎟ ⎜ ⎟⎝ ⎠ ⎜ ⎟ ⎜ ⎟ ⎜ ⎟⎝ ⎠ ⎝ ⎠ ⎝ ⎠

⎛ ⎞⎛ ⎞⎛− − −⎜ ⎟⎜ ⎟⎜⎛ ⎞

= − − −⎜ ⎟⎜ ⎟⎜ ⎟⎜ ⎟ ⎜ ⎟⎜ ⎟⎝ ⎠ ⎜ ⎟⎜ ⎟⎝ ⎠⎝ ⎠⎝

2 23( 1)

2

2 2

33( 1)

3 3

3

3( 1)

( 1)

0 2 02 0 21 0 4 0 2 0 28

2 0 2 0 2 0

0 4 2 01 4 2 0 4 28

0 4 2 0

0 2 01 2 0 22

0 2 0

jy

jy

jy

jy

ii iJ

i i ii i i

iJ

i i

i

iJ

i i

i

J

=

=

=

=

⎞⎟

⎜ ⎟⎜ ⎟⎜ ⎟

⎠⎛ ⎞−⎛ ⎞− ⎜ ⎟⎛ ⎞ ⎜ ⎟

= − −⎜ ⎟⎜ ⎟ ⎜ ⎟⎜ ⎟ ⎜ ⎟⎝ ⎠ ⎜ ⎟− ⎜ ⎟⎝ ⎠⎝ ⎠⎛ ⎞⎜ ⎟⎛ ⎞

= −⎜ ⎟⎜ ⎟⎜ ⎟ ⎜ ⎟⎝ ⎠ ⎜ ⎟−⎝ ⎠⎛ ⎞−⎜ ⎟⎛ ⎞

= −⎜ ⎟⎜ ⎟⎜ ⎟ ⎜ ⎟⎝ ⎠ ⎜ ⎟⎝ ⎠

3 ( 1)jyJ =⎛ ⎞

=⎜ ⎟⎜ ⎟⎝ ⎠

........................................................................................................................... Consequentemente, para 1j = apenas, é legitimo substituir

2

exp 1 (1 cos )y y yiJ J Ji sen

ββ β

−⎛ ⎞ ⎛ ⎞ ⎛ ⎞→ − − −⎜ ⎟ ⎜ ⎟ ⎜ ⎟

⎝ ⎠ ⎝ ⎠ ⎝ ⎠.

como o leitor pode verificar em detalhe.

6

........................................................................................................................... Demonstração : Temos que:

( ) ( )

2 3

2 41 2 2 2 3 3 3 3 4 4 4 5 5 5 5

2 3 4 5

2 2 4 4 3 5

2 2

exp 1 ...1! 2! 3!

( ) ( ) ( )exp 1 ...

1! 2! 3! 4! 5!

exp 1 ... ...2! 4! 1! 3! 5!

e

y y y y y y

y y y y y y

x x xx

iJ iJ i J i J i J i J

iJ J J J J Ji

β β β β β β

β β β β β β

= + + + +

− − − − − −⎛ ⎞= + + + + +⎜ ⎟

⎝ ⎠⎡ ⎤ ⎡ ⎤−⎛ ⎞

= − + − − +⎢ ⎥ ⎢ ⎥⎜ ⎟⎢ ⎥ ⎢ ⎥⎝ ⎠ ⎣ ⎦ ⎣ ⎦

2 2 4 3 5

2

2 2 4

2

2 2 4

2

xp 1 ... ...2! 4! 1! 3! 5!

exp 1 1 1 ...2! 4!

exp 1 1 1 ...2! 4!

exp

y y y

y y y

y y y

y

iJ J Ji

iJ J Ji sen

iJ J Ji sen

iJ

β β β β β β

β β β β

β β β β

−⎛ ⎞ ⎡ ⎤ ⎡ ⎤= − − − − +⎜ ⎟ ⎢ ⎥ ⎢ ⎥

⎣ ⎦ ⎣ ⎦⎝ ⎠−⎛ ⎞ ⎛ ⎞⎡ ⎤

= − − + − −⎜ ⎟ ⎜ ⎟⎢ ⎥⎣ ⎦⎝ ⎠ ⎝ ⎠⎡ ⎤−⎛ ⎞ ⎛ ⎞⎛ ⎞

= − − − + −⎢ ⎥⎜ ⎟ ⎜ ⎟⎜ ⎟⎝ ⎠⎝ ⎠ ⎝ ⎠⎣ ⎦

−( )

2

1 1 cosy yJ Ji sen

ββ β

⎛ ⎞⎛ ⎞ ⎛ ⎞= − − −⎜ ⎟⎜ ⎟ ⎜ ⎟⎜ ⎟⎝ ⎠ ⎝ ⎠⎝ ⎠

........................................................................................................................... c. Explicitamente temos:

(1)

1 1 1(1 cos ) (1 cos )2 22

1 1( ) cos2 2

1 1 1(1 cos ) (1 cos )2 22

sen

d sen sen

sen

β β β

β β β β

β β β

⎛ ⎞⎛ ⎞⎛ ⎞ ⎛ ⎞+ − −⎜ ⎟⎜ ⎟ ⎜ ⎟⎜ ⎟⎝ ⎠ ⎝ ⎠⎝ ⎠⎜ ⎟⎜ ⎟⎛ ⎞ ⎛ ⎞= −⎜ ⎟⎜ ⎟ ⎜ ⎟

⎝ ⎠ ⎝ ⎠⎜ ⎟⎜ ⎟⎛ ⎞⎛ ⎞ ⎛ ⎞⎜ ⎟− +⎜ ⎟ ⎜ ⎟⎜ ⎟⎜ ⎟⎝ ⎠ ⎝ ⎠⎝ ⎠⎝ ⎠

7

........................................................................................................................... Demonstração: Como

( )2

exp 1 1 cosy y yiJ J Ji sen

ββ β

⎛ ⎞−⎛ ⎞ ⎛ ⎞= − − −⎜ ⎟⎜ ⎟ ⎜ ⎟⎜ ⎟⎝ ⎠ ⎝ ⎠⎝ ⎠

,

temos:

( )2

(1)' ( ) 1, ' 1 1 cos ,1y y

m m

J Jd m i sen mβ β β

⎡ ⎤⎛ ⎞ ⎛ ⎞⎢ ⎥≡ − − −⎜ ⎟ ⎜ ⎟⎜ ⎟⎢ ⎥⎝ ⎠⎝ ⎠⎣ ⎦.

Como , ' 1m m = , 0 e 1− , teremos 9 elementos para ( )(1)

'm md β . Para o elemento 1.1 temos:

( )

( )

2

(1)1,1

(1) 21,1 2

( ) 1,1 1 1 cos 1,1

1 cos( ) 1,1 1 1,1 1,1 1,1 1,1 1,1

y y

y y

J Jd i sen

id J sen J

β β β

ββ β

⎡ ⎤⎛ ⎞ ⎛ ⎞⎢ ⎥≡ − − −⎜ ⎟ ⎜ ⎟⎜ ⎟⎢ ⎥⎝ ⎠⎝ ⎠⎣ ⎦−

≡ − −

Como

( )2y

J JJi

+ −−= ,

8

temos

( ) ( ) ( )

( ) ( ) ( )

( ) ( )

2(1)1,1 2 2 2

(1) 2 21,1 2

(1) 2 21,1 2 2

1 cos 1 1( ) 1,1 1 1,1 1,1 1,1 1,1 1,12 2

1 cos 1( ) 1,1 1 1,1 1,1 1,1 1,1 1,14 2

1 cos 1 cos( ) 1,1 1 1,1 1,1 1,1 1,1

4 4

id J J sen J Ji i

d J J J J J J sen J J

d J J

ββ β

ββ β

β ββ

+ − + −

+ − + − − + + −

+ −

− ⎡ ⎤ ⎡ ⎤≡ − − − −⎢ ⎥ ⎢ ⎥⎣ ⎦ ⎣ ⎦

−≡ + + − − − −

− −≡ + +

( )

( )2

2

1 cos1,1 1,1 1,1

41 cos 1 11,1 1,1 1,1 1,1 1,1 1,1

4 2 2

J J

J J sen J sen J

β

ββ β

+ −

− + + −

−− −

−− +

Vamos agora calcular cada termo separadamente: 1,1 1 1,1 1=

1 1,1 1,1 0

2sen Jβ − =

1 1,1 1,1 0

2sen Jβ + =

Com estes três termos calculados temos:

( ) ( ) ( )

( )

(1) 2 21,1 2 2 2

2

1 cos 1 cos 1 cos( ) 1 1,1 1,1 1,1 1,1 1,1 1,1

4 4 41 cos

1,1 1,14

d J J J J

J J

β β ββ

β

+ − + −

− +

− − −≡ + + − −

Vamos calcular os restantes. Da expressão (3.5.36)

( )† 2 2 2 2, , , , ( 1)z zj m J J j m j m J J J j m j j m m+ + ⎡ ⎤= − − = + − −⎣ ⎦ temos o resultado para †1,1 1,1 1,1 1,1J J J J+ + − += , ficando: ( ) ( ) [ ]2

2 2

1 cos 1 cos1,1 1,1 2 1 1 0

4 4J J

β β− +

− −= − − =

9

Temos novamente que a expressão se reduz a:

( ) ( ) ( )(1) 2 21,1 2 2 2

1 cos 1 cos 1 cos( ) 1 1,1 1,1 1,1 1,1 1,1 1,1

4 4 4d J J J J

β β ββ + − + −

− − −≡ + + −

Para o termo 1,1 1,1J J+ − , temos das expressões:

2 2z zJ J J J J+ − = − + e

2 2, ( 1) ,

, ,z

J j m j j j m

J j m m j m

= +

=

que se torna

( )2 2 2 2, , , , ( 1)z zj m J J j m j m J J J j m j j m m+ − ⎡ ⎤= − + = + − +⎣ ⎦ . Para ( ) ( ) [ ]

( ) ( )

22 2

2

1 cos 1 cos1,1 1,1 2

4 41 cos 11,1 1,1 1 cos

4 2

J J

J J

β β

ββ

+ −

+ −

− −=

−= −

Voltando ao termo principal:

( )(1) 21,1 2

1 cos( ) 1 1,1 1,1

4d J

ββ +

−≡ +

( ) 22

1 cos1,1 1,1

4J

β−

−+ ( )

( )(1)1,1

(1)1,1

(1)1,1

1 1 cos2

1( ) 1 1 cos21 1( ) 1 cos2 2

1( ) (1 cos )2

d

d

d

β

β β

β β

β β

− −

≡ − −

≡ − +

≡ +

considerando que os dois termos 21,1 1,1J+ e 21,1 1,1J− são nulos, desde que :

' ', 1' ' , ( )( 1) j j m mj m J j m j m j m δ δ± ±= ± +∓ . ...........................................................................................................................

1

Problema 3.23 Expresse os elementos matriciais de 2

2 2 2 3 1 1 1Jα β γ α β γ em termos de uma série em

( )jmnD jmnαβγ αβγ= .

Solução: Temos que:

2 22 2 2 3 1 1 1 2 2 2 3 1 1 1

' ' '' ' ' ' ' '

jmn j m njmn jmn J j m n j m n Jα β γ α β γ α β γ α β γ=∑ ∑ . (1)

Podemos notar também que:

2 23 ' ' '' ' ' ' nn jj mmjmn J j m n n δ δ δ= . (2)

Portanto,

( ) ( )2 2 *2 2 2 3 1 1 1 2 2 2 1 1 1

j jmn mn

jmnJ n D Dα β γ α β γ α β γ α β γ=∑ . (3)

Observação: Cortesia do Prof. Thomas Fulton.

1

Problema 3.24 Considere um sistema constituído de duas partículas de spin ½. Observador A se especializa em medir as componentes do spin de uma das partículas ( 1zs , 1xs , e assim por diante), enquanto o observador B, mede a componente de spin da outra partícula. Suponha que o sistema está em um estado de spin singleto, isto é, 0TotalS = . a. Qual é a probabilidade para o observador A obter 1 2zs = quando o

observador B não faz medida? O mesmo problema para 1 2xs = .

b. O observador B determina que o spin da partícula 2 está no estado 2 2zs =

com certeza. O que nós podemos então concluir sobre o resultado da medida do observador A se (i) A mede 1zs e (ii) A mede 1xs ? Justifique sua resposta. Solução: Nós representaremos estados como na (3.7.15):

( )

( )

1, 1

11, 02

1, 1

10, 02

s m

s m

s m

s m

= = = + +

⎛ ⎞= = = + − + − +⎜ ⎟⎝ ⎠

= = − = − −

⎛ ⎞= = = + − − − +⎜ ⎟⎝ ⎠

(3.7.15)

Para 0TotalS = , temos:

12⎡ ⎤+ − − − +⎣ ⎦ . (1)

2

Figura 1: Correlação de spin em um estado de spin-singleto. a. Desde que B não faça medidas, existem probabilidades iguais para medir

1zs , que pode ser 2

+ e 2

− . O mesmo é verdadeiro para 1xs , porque não

existe direção preferida no espaço. b. Agora, nesta situação, B mede 2 2zs = .

(i) A mede 1zs Desde que 1 2 0z zs s+ = , A deve obter

2− .

Também, 2zs seleciona o segundo pedaço de ψ , o qual é − − +∼ . Portanto:

{ }1 2zs − − + = + − + . (2)

3

(ii) A mede 1xs

Desde que nós conhecemos que 1 2zs ψ ψ⎛ ⎞= −⎜ ⎟⎝ ⎠

, nós não podemos predizer 1xS

porque [ ]1 1, 0x zs s ≠ (3) e

1 ˆ ˆˆ2

z x x⎡ ⎤− = + − −⎣ ⎦ (4)

como na equação (3.9.3)

( )

( )

1ˆ ˆ ˆ2

1 ˆ ˆˆ2

x z z

z x x

⎛ ⎞± = + ± −⎜ ⎟⎝ ⎠

⎛ ⎞± = + ± −⎜ ⎟⎝ ⎠

(3.9.3)

fornecendo iguais probabilidades para

1 2xs = e 1 2xs = − . (5)

1

Problema 3.25 Considere um tensor esférico de ordem 1 (isto é, um vetor)

(1)1 2

x yV iVV±

±= ∓ , (1)

0 zV V= .

Usando a expressão para ( )1jd = dado no problema 22, calcule:

( )(1) (1)' '

'qq q

qd Vβ∑ .

Mostre que seus resultados são apenas o que você esperaria a partir das propriedades de transformação de , ,x y zV sob rotações em torno do eixo y . Solução: ........................................................................................................................... Lembre-se: Elementos matriciais do operador rotação (3.5.50):

( )( )' , , , ' exp exp exp ,yj z z

m m

iJiJ iJD j m j mβα γα β γ

⎛ ⎞⎛ ⎞ ⎛ ⎞= − − −⎜ ⎟⎜ ⎟ ⎜ ⎟⎝ ⎠ ⎝ ⎠⎝ ⎠

(1)

Aplicando zJ , temos:

( )( ) ( ' )' , , , ' exp ,yj i m m

m m

iJD e j m j mα γ β

α β γ − + ⎛ ⎞= −⎜ ⎟

⎝ ⎠ (2)

2

Define-se uma nova matriz

( )( ) , ' exp ,yj iJd j m j m

ββ

⎛ ⎞= −⎜ ⎟

⎝ ⎠ (3)

Para 1j = (3.5.57), temos:

( )(1)

1 cos 2 1 cos1 2 2cos 22

1 cos 2 1 cos

sen

d sen sen

sen

β β β

β β β β

β β β

⎛ ⎞+ − −⎜ ⎟

= −⎜ ⎟⎜ ⎟⎜ ⎟− +⎝ ⎠

(4)

...........................................................................................................................

(1) (1)

(1) (1) (1) (1)' ' 0 0

' (1) (1)

1 cos 2 1 cos '1 2 2cos 2 '2

'1 cos 2 1 cosqq q

q

sen V Vd V sen sen V V

V Vsen

β β β

β β β

β β β

+ +

− −

⎛ ⎞+ − − ⎛ ⎞ ⎛ ⎞⎜ ⎟⎜ ⎟ ⎜ ⎟= − =⎜ ⎟⎜ ⎟ ⎜ ⎟

⎜ ⎟⎜ ⎟ ⎜ ⎟⎜ ⎟− + ⎝ ⎠ ⎝ ⎠⎝ ⎠

∑ (5)

Substituindo, os valores para

(1)1 2

x yV iVV±

±= ∓ (6)

(1)0 zV V= (7)

3

na equação:

(1) (1)

(1) (1)0 0(1) (1)

1 cos 2 1 cos '1 2 2cos 2 '2

'1 cos 2 1 cos

sen V Vsen sen V V

V Vsen

β β β

β β β

β β β

+ +

− −

⎛ ⎞+ − − ⎛ ⎞ ⎛ ⎞⎜ ⎟⎜ ⎟ ⎜ ⎟− =⎜ ⎟⎜ ⎟ ⎜ ⎟

⎜ ⎟⎜ ⎟ ⎜ ⎟⎜ ⎟− + ⎝ ⎠ ⎝ ⎠⎝ ⎠

(8)

temos:

(1)

(1)0(1)

1 cos 2 1 cos '21 2 2cos 2 '2

'1 cos 2 1 cos2

x y

z

x y

V iVsen V

sen sen V VV iV Vsen

β β β

β β β

β β β

+

+⎛ ⎞−⎜ ⎟⎛ ⎞+ − − ⎛ ⎞⎜ ⎟⎜ ⎟ ⎜ ⎟⎜ ⎟− =⎜ ⎟ ⎜ ⎟⎜ ⎟⎜ ⎟ ⎜ ⎟−⎜ ⎟− + ⎜ ⎟ ⎝ ⎠⎝ ⎠⎜ ⎟⎝ ⎠

(9)

Calculando a matriz, temos:

( ) ( )( ) ( )

( ) ( )( ) ( )

( ) ( )( ) ( )

(1)

(1)0(1)

1 cos 2 1 cos2 2

'1 2 2cos 2 '2 2 2 '

1 cos 2 1 cos2 2

x y x yz

x y x yz

x y x yz

V iV V iVsen V

VV iV V iV

sen V sen VV

V iV V iVsen V

β β β

β β β

β β β

+

⎛ ⎞+ −⎛ ⎞ ⎛ ⎞+ − + − + −⎜ ⎟⎜ ⎟ ⎜ ⎟

⎝ ⎠ ⎝ ⎠⎜ ⎟ ⎛ ⎞⎜ ⎟+ −⎛ ⎞ ⎛ ⎞ ⎜ ⎟⎜ ⎟− + + − =⎜ ⎟ ⎜ ⎟ ⎜ ⎟⎜ ⎟⎝ ⎠ ⎝ ⎠ ⎜ ⎟⎜ ⎟ ⎝ ⎠+ −⎛ ⎞ ⎛ ⎞⎜ ⎟− − + + +⎜ ⎟ ⎜ ⎟⎜ ⎟⎝ ⎠ ⎝ ⎠⎝ ⎠

(10)

Igualando termo a termo, temos:

4

( ) ( )( ) ( )

( ) ( )( ) ( )

( ) ( )( ) ( )

(1)

(1)0

(1)

1 1 cos 2 1 cos '2 2 2

1 2 2cos 2 '2 2 2

1 1 cos 2 1 cos '2 2 2

x y x yz

x y x yz

x y x yz

V iV V iVsen V V

V iV V iVsen V sen V

V iV V iVsen V V

β β β

β β β

β β β

+

⎡ ⎤+ −⎛ ⎞ ⎛ ⎞+ − + − + − =⎢ ⎥⎜ ⎟ ⎜ ⎟

⎝ ⎠ ⎝ ⎠⎣ ⎦

⎡ ⎤+ −⎛ ⎞ ⎛ ⎞− + + − =⎢ ⎥⎜ ⎟ ⎜ ⎟⎝ ⎠ ⎝ ⎠⎣ ⎦

⎡ ⎤+ −⎛ ⎞ ⎛ ⎞− − + + + =⎢ ⎥⎜ ⎟ ⎜ ⎟

⎝ ⎠ ⎝ ⎠⎣ ⎦

(11)

Para a componente (1) 'V+ , temos:

( ) ( )( ) ( ) (1)

(1)

(1)

1 1 cos 2 1 cos '2 2 2

1 cos 1 1 cos '2 2 2 22 2 2

cos 1 cos '2 22 2 2 2 2 2 2

2

x y x yz

x y x yz

x y x y x y x yz

x

V iV V iVsen V V

V iV V iVsen V V

V iV V iV V iV V iVsen V V

V

β β β

β ββ

β ββ

+

+

+

⎡ ⎤+ −⎛ ⎞ ⎛ ⎞+ − + − + − =⎢ ⎥⎜ ⎟ ⎜ ⎟

⎝ ⎠ ⎝ ⎠⎣ ⎦

+ −⎛ ⎞ ⎛ ⎞⎛ ⎞ ⎛ ⎞+ − − + − =⎜ ⎟ ⎜ ⎟⎜ ⎟ ⎜ ⎟⎝ ⎠ ⎝ ⎠⎝ ⎠ ⎝ ⎠

+ + − −− − − + − =

− (1)

(1)

cos cos 1 cos cos '2 2 2 22 2 2 2 2 2 2 2 2 2 2 2

1cos '2 2 2

y y y yx x xz

y xz

iV iV iV iVV V Vsen V V

iV V sen V V

β β β ββ

β β

+

+

− − − − + − − + =

− − − =

Para a componente (1)

0 'V , temos:

5

( ) ( )( ) ( ) (1)0

(1)0

(1)0

(1)0

1 2 2cos 2 '2 2 2

1 cos '2 2 22 2

cos '

cos '

x y x yz

y yx xz

x z

z x

V iV V iVsen V sen V

iV iV senV sen Vsen sen V V

sen V V V

V sen V V

β β β

βββ β β

β β

β β

⎡ ⎤+ −⎛ ⎞ ⎛ ⎞− + + − =⎢ ⎥⎜ ⎟ ⎜ ⎟⎝ ⎠ ⎝ ⎠⎣ ⎦

− − + − + =

− + =

− =

(13)

Para a componente (1) 'V− , temos:

( ) ( )( ) ( ) (1)

(1)

(1)

1 1 cos 2 1 cos '2 2 2

1 cos 1 1 cos '2 2 2 22 2 2

cos 1 cos '2 22 2 2 2 2 2 2

2 2

x y x yz

x y x yz

x y x y x y x yz

x

V iV V iVsen V V

V iV V iVsen V V

V iV V iV V iV V iVsen V V

V

β β β

β ββ

β ββ

+

+

+

⎡ ⎤+ −⎛ ⎞ ⎛ ⎞− − + + + =⎢ ⎥⎜ ⎟ ⎜ ⎟

⎝ ⎠ ⎝ ⎠⎣ ⎦

+ −⎛ ⎞ ⎛ ⎞⎛ ⎞ ⎛ ⎞− − + + + =⎜ ⎟ ⎜ ⎟⎜ ⎟ ⎜ ⎟⎝ ⎠ ⎝ ⎠⎝ ⎠ ⎝ ⎠

+ + − −− + + + + =

− (1)

(1)

cos cos 1 cos cos '2 2 2 22 2 2 2 2 2 2 2 2 2 2

1cos '2 2 2

y y y yx x xz

y xz

iV iV iV iVV V Vsen V V

iV V sen V V

β β β ββ

β β

+

+

− + + + + − + − =

− + + =

6

Resumindo, temos:

(1)1cos '2 2 2y x

z

iV V sen V Vβ β +− − − = (15)

(1)

0cos 'z xV sen V Vβ β− = (16)

(1)1cos '2 2 2y x

z

iV V sen V Vβ β −− + + = (17)

Reescrevendo (1) (1)

' ''

qq qq

d V∑ , termos:

(1) (1)' '

'

cos / 2 / 2 / 2cos

cos / 2 / 2 / 2

x y z

qq q x zq

x y z

V iV sen Vd V sen V V

V iV sen V

β ββ β

β β

⎛ ⎞− − −⎜ ⎟

= − +⎜ ⎟⎜ ⎟⎜ ⎟− +⎝ ⎠

∑ (18)

...........................................................................................................................Lembre-se:

( )jd β é operador rotação, e representa uma rotação em torno do eixo y .

( )( ) , ' exp ,yj iJd j m j m

ββ

⎛ ⎞= −⎜ ⎟

⎝ ⎠ (19)

...........................................................................................................................

7

...........................................................................................................................Lembre-se: Uma rotação em torno do eixo y pode ser representada como (3.1.5b):

( )

2

2

1 0 cos 020 1 0 0 1 0

0 cos0 1

2

y

senR

sen

β β β ββ

β βββ

⎛ ⎞−⎜ ⎟ ⎛ ⎞⎜ ⎟ ⎜ ⎟= =⎜ ⎟ ⎜ ⎟⎜ ⎟ ⎜ ⎟−⎝ ⎠⎜ ⎟− −⎜ ⎟

⎝ ⎠

(20)

Portanto,

1 1

2 2

3 3

1 1 3

2 2

3 1 3

1 1 3

2 2

3 1 3

'

' cos 0' 0 1 0' 0 cos

' cos'' cos

' cos'

' cos

x Rx

x sen xx xx sen x

x x x senx xx x sen x

x x x senx x

x x sen x

β β

β β

β β

β β

β β

β β

=

⎛ ⎞ ⎛ ⎞⎛ ⎞⎜ ⎟ ⎜ ⎟⎜ ⎟=⎜ ⎟ ⎜ ⎟⎜ ⎟⎜ ⎟ ⎜ ⎟⎜ ⎟−⎝ ⎠ ⎝ ⎠⎝ ⎠

+⎛ ⎞ ⎛ ⎞⎜ ⎟ ⎜ ⎟=⎜ ⎟ ⎜ ⎟⎜ ⎟ ⎜ ⎟− +⎝ ⎠ ⎝ ⎠

= +⎧⎪ =⎨⎪ = − +⎩ (21) ...........................................................................................................................

8

Uma rotação de um ângulo β em torno de um eixo y produz os seguintes resultados:

' cos

'

' cos

x x x z

y y y

z z z x

V V V V sen

V V V

V V V V sen

β β

β β

→ = +

→ =

→ = −

(22)

Por outro lado, temos do enunciado as seguintes expressões:

(1)1 2

x yV iVV±

±= ∓ (23)

e

(1)0 zV V= (24)

Aplicando as propriedades de transformação acima,

' cos

'

' cos

x x z

y y

z z x

V V V sen

V V

V V V sen

β β

β β

= +

=

= −

, (25)

9

temos os mesmos resultados:

( )

( )

(1)

(1)0

(1)

' ' 1' cos2 2 2 2

' ' cos

' ' 1' cos2 2 2 2

x y y zx

z x z

x y y zx

V iV V VV V i sen

V V sen V V

V iV V VV V i sen

β β

β β

β β

+

+= − = − − −

= = − +

−= = − +

(26)

isto é,

(1) (1)' '

'

cos / 2 / 2 / 2cos

cos / 2 / 2 / 2

x y z

qq q x zq

x y z

V iV sen Vd V sen V V

V iV sen V

β ββ β

β β

⎛ ⎞− − −⎜ ⎟

= − +⎜ ⎟⎜ ⎟⎜ ⎟− +⎝ ⎠

∑ . (27)

Ou seja, são as transformações esperadas a partir das propriedades de transformação de (1)

, ,x y zV sob uma rotação em torno do eixo y .

1

Problema 3.26 a. Construa um tensor esférico de ordem 1 a partir de dois vetores diferentes

( ), ,x y zU U U U= e ( , , )x y zV V V V= . Explicitamente escreva (1)1,0T± em termos de

, ,x y zU e , ,x y zV . b. Construa um tensor esférico de ordem 2 a partir de dois vetores diferentes U e V . Escreva abaixo explicitamente (2)

2, 1,0T± ± em termos de , ,x y zU e , ,x y zV . Solução: a. Primeiramente, vamos recordar a expressão (3.10.27):

( ) ( )1 2

1 2

1 2

( )1 2 1 2 1 2; ; k kk

q q qq q

T k k q q k k kq X Z=∑∑ . (3.10.27)

........................................................................................................................... Lembre-se: Teorema. Considere 1

1

( )kqX e 2

2

( )kqZ serem tensores esféricos irredutíveis de

ordem 1k e 2k , respectivamente. Então

( ) ( )1 2

1 2

1 2

( )1 2 1 2 1 2; ; k kk

q q qq q

T k k q q k k kq X Z=∑∑ (3.10.27)

é um tensor esférico irredutível de ordem k . ........................................................................................................................... Portanto, 1

1

( )kqX e 2

2

( )kqZ são tensores esféricos irredutíveis de ordem 1k e 2k ,

respectivamente. Então,

( ) ( )1 2

1 2

1 2

( )1 2 1 2 1 2; ; k kk

q q qq q

T k k q q k k kq X Z=∑∑ . (3.10.27)

é um tensor esférico irredutível de ordem k .

2

Para o nosso problema, temos:

1 2 1k k k= = = . (1) Então, podemos escrever a equação acima como:

1 2

1 2

(1) (1) (1)1 211; 11;1q q q

q qT q q q U V=∑∑ . (2)

........................................................................................................................... Demonstração:

1 2

1 2

(1) (1) (1)1 211; 11;1q q q

q qT q q q U V=∑∑ (3)

Vamos primeiramente calcular o tensor para 1q = − :

1 2

1 2

(1) (1) (1)1 1 211; 11;1 1 q q

q qT q q U V− = −∑∑ (4)

Abrindo o primeiro somatório, considerando que,

2 1,0, 1q = + − , (5) temos:

1 1 1

1

(1) (1) (1) (1) (1) (1) (1)1 1 1 1 0 1 111; 1 11;1 1 11; 0 11;1 1 11; 1 11;1 1q q q

q

T q U V q U V q U V− −⎡ ⎤= − + − + − −⎣ ⎦∑

Para o segundo somatório, temos que considerar:

1 1,0, 1q = + − . (7) Abrindo o somatório, temos:

(1) (1) (1) (1) (1) (1) (1)1 1 1 0 1 1 1

(1) (1) (1) (1) (1) (1)1 0 0 0 1 0

(1) (1) (1) (1)1 1 0 1

11;11 11;1 1 11;01 11;1 1 11; 11 11;1 1

11;10 11;1 1 11;00 11;1 1 11; 10 11;1 1

11;1 1 11;1 1 11;0 1 11;1 1 1

T U V U V U V

U V U V U V

U V U V

− −

− −

⎡ ⎤= − + − + − − +⎣ ⎦⎡ ⎤− + − + − − +⎣ ⎦

− − + − − + (1) (1)1 11; 1 1 11;1 1 U V− −⎡ ⎤− − −⎣ ⎦

3

Temos agora que calcular os coeficientes de Clebsch-Gordan. Podemos utilizar:

0j =

( )10, 0 003

j m= = = + − − + − + (9)

1j =

( )11, 1 0 02

j m= = = + − + (10)

( )11, 02

j m= = = + − − − + (11)

( )11, 1 0 02

j m= = − = − − − (12)

2j =

2, 2j m= = = + + (13)

( )12, 1 0 02

j m= = = + + + (14)

( )12, 0 2 006

j m= = = − + + + + − (15)

( )12, 1 0 02

j m= = − = − + − (16)

2, 2j m= = − = − − (17) A) Primeiro termo:

1 2 1 2 1 2; ; 11;11 11;1 1 11; 11;1 1j j m m j j jm = − = + + − (18) O ket , 1, 1j m = − pode ser escrito como:

( )11, 1 0 02

j m= = − = − − − . (19)

Fazendo o produto escalar, temos:

( ) ( )111; 11;1 1 0 0 02

⎡ ⎤+ + − = + + − − − =⎢ ⎥⎣ ⎦. (20)

4

B) Segundo termo:

1 2 1 2 1 2; ; 11;01 11;1 1 11;0 11;1 1j j m m j j jm = − = + − (21) O ket , 1, 1j m = − pode ser escrito como:

( )11, 1 0 02

j m= = − = − − − . (22)

Fazendo o produto escalar, temos:

( ) ( )111;0 11;1 1 0 0 0 02

⎡ ⎤+ − = + − − − =⎢ ⎥⎣ ⎦. (23)

C) Terceiro termo:

1 2 1 2 1 2; ; 11; 11 11;1 1 11; 11;1 1j j m m j j jm = − − = − + − (24) O ket , 1, 1j m = − pode ser escrito como:

( )11, 1 0 02

j m= = − = − − − . (25)

Fazendo o produto escalar, temos:

( ) ( )111; 11;1 1 0 0 02

⎡ ⎤− + − = − + − − − =⎢ ⎥⎣ ⎦. (26)

D) Quarto termo:

1 2 1 2 1 2; ; 11;10 11;1 1 11; 0 11;1 1j j m m j j jm = − = + − (27) O ket , 1, 1j m = − pode ser escrito como:

( )11, 1 0 02

j m= = − = − − − . (28)

Fazendo o produto escalar, temos:

5

( ) ( )111; 0 11;1 1 0 0 0 02

⎡ ⎤+ − = + − − − =⎢ ⎥⎣ ⎦. (29)

E) Quinto termo:

1 2 1 2 1 2; ; 11;00 11;1 1 11;00 11;1 1j j m m j j jm = − = − (30) O ket , 1, 1j m = − pode ser escrito como:

( )11, 1 0 02

j m= = − = − − − . (31)

Fazendo o produto escalar, temos:

( ) ( )111;00 11;1 1 00 0 0 02

⎡ ⎤− = − − − =⎢ ⎥⎣ ⎦. (32)

F) Sexto termo:

1 2 1 2 1 2; ; 11; 10 11;1 1 11; 0 11;1 1j j m m j j jm = − − = − − (33) O ket , 1, 1j m = − pode ser escrito como:

( )11, 1 0 02

j m= = − = − − − . (34)

Fazendo o produto escalar, temos:

( ) ( )1 111; 0 11;1 1 0 0 02 2

⎡ ⎤− − = − − − − = −⎢ ⎥⎣ ⎦. (35)

G) Sétimo termo:

1 2 1 2 1 2; ; 11;1 1 11;1 1 11; 11;1 1j j m m j j jm = − − = + − − (36) O ket , 1, 1j m = − pode ser escrito como:

6

( )11, 1 0 02

j m= = − = − − − . (37)

Fazendo o produto escalar, temos:

( ) ( )111; 11;1 1 0 0 02

⎡ ⎤+ − − = + − − − − =⎢ ⎥⎣ ⎦. (38)

H) Oitavo termo:

1 2 1 2 1 2; ; 11;0 1 11;1 1 11;0 11;1 1j j m m j j jm = − − = − − (39) O ket , 1, 1j m = − pode ser escrito como:

( )11, 1 0 02

j m= = − = − − − . (40)

Fazendo o produto escalar, temos:

( ) ( )1 111;0 11;1 1 0 0 02 2

⎡ ⎤− − = − − − − =⎢ ⎥⎣ ⎦. (41)

I) Nono termo:

1 2 1 2 1 2; ; 11; 1 1 11;1 1 11; 11;1 1j j m m j j jm = − − − = − − − (42) O ket , 1, 1j m = − pode ser escrito como:

( )11, 1 0 02

j m= = − = − − − . (43)

Fazendo o produto escalar, temos:

( ) ( )111; 11;1 1 0 0 02

⎡ ⎤− − − = − − − − − =⎢ ⎥⎣ ⎦. (44)

...........................................................................................................................

7

Calculando para os outros termos, temos finalmente:

( )

( )

( )

(1) (1) (1) (1) (1)1 1 0 0 1

(1) (1) (1) (1) (1)0 1 1 1 1

(1) (1) (1) (1) (1)1 0 1 1 0

12

12

12

T U V U V

T U V U V

T U V U V

− − −

− −

= − +

= −

= − +

(45)

Em termos de componentes , ,x y zU e , ,x y zV , temos:

( ) ( )

( ) ( )

(1)1

(1)0

(1)1

12

212

x y z x y z

x y y x

x y z x y z

T U iU V V iV U

iT U V U V

T U iU V V iV U

−⎡ ⎤= − − + −⎣ ⎦

⎡ ⎤= −⎣ ⎦

⎡ ⎤= − + + +⎣ ⎦

(46)

........................................................................................................................... Lembre-se:

( )q qU V é a q − ésima componente de um tensor esférico de ordem 1, correspondendo a um vetor ( )U V . Temos por definição (página 237) que:

( )( )

1

1

0

/ 2

/ 2

x y

x y

z

U U iU

U U iU

U U

+

≡ − +

≡ −

( )

( )1

1

0

/ 2

/ 2

x y

x y

z

V V iV

V V iV

V V

+

≡ − +

≡ −

(47)

Para (1)

1T− , temos:

8

( )

( ) ( )

( ) ( )

(1) (1) (1) (1) (1)1 1 0 0 1

(1)1

(1)1

12

12 2 2

12

x y x yz z

x y z x y z

T U V U V

U iU V iVT V U

T U iU V V iV U

− − −

= − +

⎛ ⎞− −⎜ ⎟= − +⎜ ⎟⎝ ⎠

⎡ ⎤= − − + −⎣ ⎦

(48)

........................................................................................................................... b. Para

1 2 1k k= = (49) e

2k = (50) temos:

1 2

1 2

(2) (1) (1)1 211; 11;2q q q

q qT q q q U V=∑∑ . (51)

........................................................................................................................... Demonstração: Para 2q = − , temos:

1 2

1 2

(2) (1) (1)2 1 211; 11;2 2 q q

q qT q q U V− = −∑∑ . (52)

Abrindo o somatório para

2 1,0, 1q = + − , (53)

temos:

1 1 1

1

(2) (1) (1) (1) (1) (1) (1)2 1 1 1 0 1 1( 11; 1 11;2 2 11; 0 11;2 2 11; 1 11;2 2 )q q q

qT q U V q U V q U V− + −= + − + − + − −∑

9

ou

1

1

1

1

1

1

(2) (1) (1)2 1 1

(1) (1)1 0

(1) (1)1 1

11; 1 11;2 2

11; 0 11;2 2

11; 1 11;2 2

qq

qq

qq

T q U V

q U V

q U V

− +

= + − +

− +

− −

(55)

Abrindo o somatório para

2 1,0, 1q = + − , (56)

temos:

(2) (1) (1) (1) (1) (1) (1)2 1 1 0 1 1 1

(1) (1) (1) (1) (1) (1)1 0 0 0 1 0

(1) (1) (1) (1 1 0 1

11; 1 1 11;2 2 11;0 1 11;2 2 11; 1 1 11;2 2

11; 10 11;2 2 11;00 11;2 2 11; 10 11;2 2

11; 1 1 11;2 2 11;0 1 11;2 2

T U V U V U V

U V U V U V

U V U V

− + + + − +

+ −

+ − −

= + + − + + − + − + − +

+ − + − + − − +

+ − − + − − 1) (1) (1)1 111; 1 1 11;2 2 U V− −+ − − −

Temos agora que calcular os coeficientes de Clebsch-Gordan. Podemos utilizar:

0j =

( )10, 0 003

j m= = = + − − + − + (58)

1j =

( )11, 1 0 02

j m= = = + − + (59)

( )11, 02

j m= = = + − − − + (60)

( )11, 1 0 02

j m= = − = − − − (61)

2j =

2, 2j m= = = + + (62)

( )12, 1 0 02

j m= = = + + + (63)

10

( )12, 0 2 006

j m= = = − + + + + − (64)

( )12, 1 0 02

j m= = − = − + − (65)

2, 2j m= = − = − − (66) A) Primeiro termo:

1 2 1 2 1 2; ; 11; 1 1 11;2 2 11; 11;2 2j j m m j j jm = + + − = + + − (67) O ket , 2, 2j m = − pode ser escrito como:

2, 2j m= = − = − − . (68) Fazendo o produto escalar, temos:

( )( )11; 11;2 2 0+ + − = + + − − = . (69) B) Segundo termo:

1 2 1 2 1 2; ; 11;0 1 11;2 2 11;0 11;2 2j j m m j j jm = + − = + − (70) O ket , 2, 2j m = − pode ser escrito como:

2, 2j m= = − = − − . (71) Fazendo o produto escalar, temos:

( )( )11;0 11;2 2 0 0+ − = + − − = . (72) C) Terceiro termo:

1 2 1 2 1 2; ; 11; 11 11;2 2 11; 11;2 2j j m m j j jm = − − = − + − (73) O ket , 2, 2j m = − pode ser escrito como:

11

2, 2j m= = − = − − . (74) Fazendo o produto escalar, temos:

( )( )11; 11;2 2 0− + − = − + − − = . (75) D) Quarto termo:

1 2 1 2 1 2; ; 11;10 11;2 2 11; 0 11;2 2j j m m j j jm = − = + − (76) O ket , 2, 2j m = − pode ser escrito como:

2, 2j m= = − = − − . (77) Fazendo o produto escalar, temos:

( )( )11; 0 11;2 2 0 0+ − = + − − = . (78) E) Quinto termo:

1 2 1 2 1 2; ; 11;00 11;2 2 11;00 11;2 2j j m m j j jm = − = − (79) O ket , 2, 2j m = − pode ser escrito como:

2, 2j m= = − = − − . (80) Fazendo o produto escalar, temos:

( )( )11;00 11;2 2 00 0− = −− = . (81) F) Sexto termo:

1 2 1 2 1 2; ; 11; 10 11;2 2 11; 0 11;2 2j j m m j j jm = − − = − − (82) O ket , 2, 2j m = − pode ser escrito como:

12

2, 2j m= = − = − − . (83) Fazendo o produto escalar, temos:

( )( )11; 0 11;2 2 0 0− − = − − − = . (84) G) Sétimo termo:

1 2 1 2 1 2; ; 11;1 1 11;2 2 11; 11;2 2j j m m j j jm = − − = + − − (85) O ket , 2, 2j m = − pode ser escrito como:

2, 2j m= = − = − − . (86) Fazendo o produto escalar, temos:

( )( )11; 11;2 2 0+ − − = + − − − = . (87) H) Oitavo termo:

1 2 1 2 1 2; ; 11;0 1 11;2 2 11;0 11;2 2j j m m j j jm = − − = − − (88) O ket , 2, 2j m = − pode ser escrito como:

2, 2j m= = − = − − . (89) Fazendo o produto escalar, temos:

( )( )11;0 11;2 2 0 0− − = − − − = . (90) I) Nono termo:

1 2 1 2 1 2; ; 11; 1 1 11;2 2 11; 11;2 2j j m m j j jm = − − − = − − − (91) O ket , 2, 2j m = − pode ser escrito como:

13

2, 2j m= = − = − − . (92) Fazendo o produto escalar, temos:

( )( )11; 11;1 1 1− − − = − − − − = . (93) Finalmente, temos:

(2) (1) (1)2 1 1T U V− − −= (94)

........................................................................................................................... Repetindo a mesma operação acima, temos:

( )

( )

( )

(2) (1) (1)2 1 1

(2) (1) (1) (1) (1)1 1 0 0 1

(2) (1) (1) (1) (1) (1) (1)0 1 1 0 0 1 1

(2) (1) (1) (1) (1)1 1 0 0 1

(2) (1) (1)2 1 1

12

1 26

12

T U V

T U V U V

T U V U V U V

T U V U V

T U V

− − −

− − −

− + + −

+ +

+ +

=

= +

= + +

= +

=

. (95)

Em termos de componentes de , ,x y zU e , ,x y zV ,

( )( )

1

1

0

/ 2

/ 2

x y

x y

z

U U iU

U U iU

U U

+

≡ − +

≡ −

( )

( )1

1

0

/ 2

/ 2

x y

x y

z

V V iV

V V iV

V V

+

≡ − +

≡ −

(96)

temos:

14

( )( )

( ) ( )

( )( ) ( )( )

( ) ( )

( )( )

(2)2

(2)1

(2)0

(2)1

(2)2

1212

1 42 6

12

12

x y x y

x y z z x y

x y x y z z x y x y

x y z z x y

x y x y

T U iU V iV

T U iU V U V iV

T U iU V iV U V U iU V iV

T U iU V U V iV

T U iU V iV

= − −

⎡ ⎤= − + −⎣ ⎦

⎡ ⎤= − − + + − + −⎣ ⎦

⎡ ⎤= − + + +⎣ ⎦

= + +

(97)

1

Problema 3.27 Considere uma partícula sem spin ligada a um centro fixo por um potencial de força central. a. Relacione, o quanto possível, os elementos matriciais

( )1', ', ' , ,2

n l m x iy n l m±∓ e ', ', ' , ,n l m z n l m

usando apenas o teorema Wigner-Eckart. Esteja certo em declarar sob quais condições os elementos matriciais são não nulos. b. Faça o mesmo problema usando as funções de onda ( ) ( )( ) ,m

nl lx R r Yψ θ φ= . Solução: a. Utilizando a equação (3.10.31) (teorema de Wigner-Eckart) para o nosso problema, em que

(1)1

(1)0

1 ( )2

R x iy

R z

± = ±

=

, ( 1q = + , 1q = − e 0q = ) (1)

temos três componentes de um tensor esférico de ordem 1, e que podem ser escritas como:

(1)(1)

' '', ', ' , , 1; 1; ' '

2 1q

n l R nln l m R n l m l mq l l m

l=

+. (2)

Os elementos matriciais de “dupla barra” são independentes de m e 'm .

2

........................................................................................................................... Lembre-se: Teorema de Wigner-Eckart: Os elementos matriciais do operador tensor com respeito aos autoestados de momento angular satisfazem

( )( )

' ' ,', ' ' , ; ; ' '

2 1

kk

q

j T jj m T jm jk mq jk j m

j

α αα α =

+ (3.10.31)

onde o elemento matricial de dupla barra é independente de m e 'm . ........................................................................................................................... Desde que

1; 1; ' ' 0l mq l l m = , (3) a menos que

'm m q= + , (4) e

' 1 ,l l l= ± , Duvida! (5)

3

........................................................................................................................... Lembre-se: Indeed, from the requirement that the Clebsch-Gordan coefficient be nonvanishing, we immediately obtain the m − selection rule (3.10.28),

( )', ' ' , 0kqj m T jmα α = unless 'm m q= + , (6)

derived before and also the triangular relation

'j k j j k− ≤ < + . (7)

Sakuray, pag. 239. ........................................................................................................................... portanto

(1)', ', ' , , 0qn l m R n l m = (8) a menos que

'm m q= + , (9) e

' 1 ,l l l= ± . (10)

4

Além do mais, desde que estamos lidando com um potencial de força central, os , ,n l m são autoestados do operador paridade PU . Dúvida! Então

( ), , 1 , ,lPU n l m n l m= − , (11)

e

( 1) (1) (1)P PU R U R− = − , (12)

........................................................................................................................... Lembre-se: Therefore, we can conclude that

, ( 1) ,llm lmπ α α= − . (4.2.30)

Sakuray, pag. 255. ........................................................................................................................... ........................................................................................................................... Lembre-se:

1 †π π π− = = (4.2.7)

†x xπ π = − (4.2.3)

Sakuray, pag. 252. ...........................................................................................................................

5

e nós temos

( ) ( ) '(1) (1)', ', ' , , 1 1 ', ', ' , ,l ln l m R n l m n l m R n l m− = − − (13) ou

'l l+ = ímpar. (14) ........................................................................................................................... Demonstração: Considerando a relação

( 1) (1) (1)P PU R U R− = − , (15)

e multiplicando por , ,n l m e ', ', 'n l m , temos:

( 1) (1) (1)', ', ' , , ', ', ' , ,P Pn l m U R U n l m n l m R n l m− = − . (16) Mas,

( ), , 1 , ,lPU n l m n l m= − , (17)

então

6

' (1) (1)( 1) ', ', ' , , ( 1) ', ', ' , ,l ln l m R n l m n l m R n l m− − = − . (18) Como resultado final, temos

( ) ( )

( )

'(1) (1)

'(1) (1)

', ', ' , , 1 1 ', ', ' , ,

', ', ' , , 1 ', ', ' , ,

l l

l l

n l m R n l m n l m R n l m

n l m R n l m n l m R n l m+

− = − −

− = −

, (19)

com

'l l+ = ímpar. (20) ........................................................................................................................... Ou seja, a expressão abaixo

(1)(1)

1; 1; ' ' ' '', ', ' , ,

2 1q

l mq l l m n l R nln l m R n l m

l=

+, (21)

se torna nula, utilizando as regras de seleção de Clebsch-Gordan,

(1)', ', ' , , 0qn l m R n l m = , (22) a menos que

'm m q= + , (23)

7

e

' 1l l= ± . (24) De novo, a partir de

(1)(1)

1; 1; ' ' ' '', ', ' , ,

2 1q

l mq l l m n l R nln l m R n l m

l=

+ (25)

temos:

(1)1 1 1 1 1

(1)2 0 2 2 2

', ', ' , , 1; , 1 1; ' '', ', ' , , 1; ,0 1; ' '

n l m R n l m l m l l mn l m R n l m l m l l m

± ±= (26)

onde ', 'l m satisfazem as regras de seleção

(1)', ', ' , , 0qn l m R n l m = (27) a menos que

'm m q= + , (28) e

' 1l l= ± . (29)

8

b. Vamos usar agora as funções de onda ( ) ( ) ( , )m

nl lx R r Yψ θ φ= . (30) Temos:

( ) ( ) ( )(1) * '* (1) 31,0 ' ' ' 1,0', ', ' , , , ( ) ,m m

n l l nl ln l m R n l m R r Y R R r Y d xθ φ θ φ± ±⎡ ⎤= ⎣ ⎦∫ (31) ........................................................................................................................... Lembre-se:

( ) ( )k m qq l kR Y V=

== (3.10.15)

01

3 3cos4 4

zYr

θπ π

= = → (1)0

34

R zπ

= (3.10.16)

11

34 2

x iyYrπ±

±= ∓ → (1)

13

4 2x iyR

π±±⎛ ⎞= ⎜ ⎟

⎝ ⎠∓

...........................................................................................................................

( ) ( ) ( ) ( )(1) * 3 '* 1,01,0 ' ' ' 1

4', ', ' , , ( ) , , ,3

m mn l nl l ln l m R n l m R r r R r dr d Y Y Yπ θ φ θ φ θ φ±

± = Ω∫ ∫ (32)

9

........................................................................................................................... Lembre-se: De acordo com a tabela Schaum, equação 22.71, temos:

1 2 3 1 2 3dV h h h du du du= (33) Especificamente, para coordenadas esféricas, temos:

1

2

3

1h

h r

h rsenθ

=

=

=

(34)

Portanto, dV , pode ser escrito como:

2dV r sen drd dθ θ φ= (35) Podemos substituir

3d x → 2r sen drd dθ θ φ (36) ou

3 sen d dr drr

θ θ φ∫ ∫ . (37)

10

De uma maneira mais compacta, temos:

3d x → 3r dr dΩ∫ ∫ . (38) ........................................................................................................................... Considerando que

3 3 *' , ' ' '

0

( ) ( )n n l l n l nlr r R r R r dr∞

= ∫ , (39)

então a equação

( ) ( ) ( ) ( )(1) * 3 '* 1,0,0 ' ' ' 1

0

4', ', ' , , ( ) , , ,3

m mn l nl l ln l m R n l m R r r R r dr d Y Y Yπ θ φ θ φ θ φ

∞±

± = Ω∫ ∫ , (40)

usando (3.7.73),

1 2

1 2

* 1 21 2 1 2 1 2 1 2 1 2

(2 1)(2 1)( , ) ( , ) ( , ) ;00 ; 0 ; ;4 (2 1)

m mml l l

l ld Y Y Y l l l l l l l m m l l lml

θ φ θ φ θ φπ+ +

Ω =+∫ ,

Pode ser escrita como

( )( )

( )( )

(1) 3' , '

(1) 3' , '

2 1 34', ', ' , , 1;00 1; '0 1; 1; ' '3 4 2 ' 1

2 1', ', ' , , 1;00 1; '0 1; 1; ' ' '

2 ' 1

n n l lq

n n l lq

ln l m R n l m r l l l l mq l l m

l

ln l m R n l m r l l l l mq l l m l l

l

ππ

+=

+

+= ≠

+

11

........................................................................................................................... Equivalências:

( ) ( ) ( ) ( )(1) * 3 '* 1,0,0 ' ' ' 1

0

4', ', ' , , ( ) , , ,3

m mn l nl l ln l m R n l m R r r R r dr d Y Y Yπ θ φ θ φ θ φ

∞±

± = Ω∫ ∫ (43)

3 3 *' , ' ' '

0

( ) ( )n n l l n l nlr r R r R r dr∞

= ∫ (44)

1 2

1 2

* 1 21 2 1 2 1 2 1 2 1 2

(2 1)(2 1)( , ) ( , ) ( , ) ;00 ; 0 ; ;4 (2 1)

m mml l l

l ld Y Y Y l l l l l l l m m l l lml

θ φ θ φ θ φπ+ +

Ω =+∫

( )( )

(1) 3' , '

2 1 34', ', ' , , 1;00 1; '0 1; 1; ' '3 4 2 ' 1

n n l lq

ln l m R n l m r l l l l mq l l m

π+

=+

(46)

Comparando, temos: l → 'l (47)

1l → l (48)

2l → 1 (49)

1m → m (50)

2m → q (51) m → 'm (52) ...........................................................................................................................

12

Para o caso particular em que 'l l= , temos (Dúvida!):

( )( )

(1) 3' , '

(1) 3' ,

(1)

2 1 34', ', ' , , 1;00 1; '0 1; 1; ' '3 4 2 ' 1

', ', ' , , 1;00 1; 0 1; 1;

', ', ' , , 0 '

n n l lq

n n llq

q

ln l m R n l m r l l l l mq l l m

l

n l m R n l m r l l l l mq l lm

n l m R n l m l l

ππ

+=

+

=

= =

em que

1,0q = ± . (54) Nós temos a regra de seleção

(1)', ', ' , , 0qn l m R n l m = (55) a menos que

'm m q= + (56) e

' 1l l= ± , (57) o qual é idêntico a parte (a).

13

Também, note a partir da equação,

( )( )

(1) 3' , '

2 1', ', ' , , 1;00 1; '0 1; 1; ' ' '

2 ' 1n n l lq

ln l m R n l m r l l l l mq l l m l l

l+

= ≠+

,

que temos mais uma vez a igualdade da razão

(1)1 1 1 1 1

(1)2 0 2 2 2

', ', ' , , 1; , 1 1; ' '', ', ' , , 1; ,0 1; ' '

n l m R n l m l m l l mn l m R n l m l m l l m

± ±= (58)

onde

' 1l l= ± , (59)

1 1' 1m m= ± , ( 'm m q= + ) (60)

2 2'm m= . (61)

1

Problema 3.28 a. Escreva xy , xz , e 2 2( )x y− como componentes de um tensor esférico (irredutível) de ordem 2. b. O valor esperado

2 2, , (3 ) , ,Q e j m j z r j m jα α≡ = − = é conhecido como o momento de quadrupolo. Calcule

( )2 2, , ' , ,e j m x y j m jα α− = , (onde 'm j= , 1j − , 2j − ,...) em termos de Q e os coeficientes apropriados de Clebsch-Gordan. Solução: a. Primeiramente, devemos recordar a equação:

( )22

2 2

1532

x iyY

rπ± ±= . (3.10.17)

Com base nesta equação, temos:

( )2 22

2 2

21532

x y ixyY

rπ±

− ±= . (1)

Separadamente, temos:

( )2 22

2 2

21532

x y ixyY

rπ+

− += , (2)

e

( )2 22

2 2

21532

x y ixyY

rπ−

− −= . (3)

2

Subtraindo (2) de (3), temos:

( ) ( )

[ ]

2 2 2 22 2

2 2 2 2

2 22 2 2

2 21532

15 1 432

x y ixy x y ixyY Y

r r

Y Y ixyr

π

π

+ −

+ −

⎡ ⎤− + − −⎢ ⎥− = −⎢ ⎥⎣ ⎦

− =

. (4)

Isolando xy , temos:

( ) [ ]

( )

( )

2 2 22 2

22 2

2 2

22 2

2 2

32 415

3215 4

215

r Y Y ixy

rxy Y Yi

rxy Y Yi

π

π

π

+ −

+ −

+ −

− =

= −

= −

. (5)

Multiplicando por i , temos:

( )

( )

2 2 22 2

2 2 22 2

215

215

xy i r Y Y

xy i r Y Y

π

π

+ −

− +

= − −

= −

. (6)

Para o cálculo do termo xz , devemos primeiramente nos lembrar que

( )12 2 2 2

15 158 8

x iy z xz iyzYr r rπ π

± ± ⎡ ⎤= = ⎢ ⎥⎣ ⎦∓ ∓ ∓ . (7)

........................................................................................................................... Lembre-se: De acordo com o apêndice A.5, temos para o harmônico esférico 1

2Y ± a seguinte expressão:

12

15 ( cos )8

iY sen e φθ θπ

± ±= ∓ . (A.5.7)

3

Temos também que:

cos

cos

x rseny rsen senz r

θ φθ φθ

===

. (8)

Portanto, podemos reescrever o referido harmônico esférico em ternos de x , y e z . Por exemplo, para 1

2Y + , temos:

( )

( )

( )

( )( )

12 2

12 2

12

12

158

cos cos15815 cos cos cos815 cos os8

x iy zY

r

rsen irsen sen rY

r

Y sen isen sen

Y sen c isen

π

θ φ θ φ θπ

θ φ θ θ φ θπ

θ θ φ φπ

+= −

+= −

= − +

= − +

. (9)

Considerando que

cosie isenφ φ φ= + , (10) temos a expressão requerida. ........................................................................................................................... Separadamente, temos:

( )12 2 2 2

15 15 158 8 8

x iy z xz iyzYr r rπ π π

+ += − = − − (11)

e

( )12 2 2 2

15 15 158 8 8

x iy z xz iyzYr r rπ π π

− −= + = − (12)

4

Subtraindo (9) de (8), temos:

1 12 2 2

1 12 2 2

1 12 2 2

1528

4.158152

xzY YrxzY Yr

xzY Yr

π

π

π

− +

− +

− +

− =

− =

− =

. (13)

Isolando xz , temos:

( )1 1 22 2

215

xz Y Y rπ − += − . (14)

Para o cálculo de 2 2x y− , vamos novamente usar o harmônico esférico dado pela equação (3.10.27),

( )22

2 2

1532

x iyY

rπ± ±= , (3.10.27)

que separadamente temos:

( )2 22

2 2

21532

x y ixyY

rπ+

− += , (15)

e

( )2 22

2 2

21532

x y ixyY

rπ−

− −= . (16)

Somando (15) e (16), temos:

2 2 2 22 2 2

2 15 ( )32

Y Y x yr π

+ −+ = − . (17)

5

Isolando 2 2( )x y− , temos:

( )

2 2 2 22 2 2

2 2 2 2 22 2

2 15 ( )32

8( )15

Y Y x yr

x y Y Y r

π

π

+ −

+ −

+ = −

− = +

. (18)

Pode-se notar que 2

mY para 0, 1, 2m = ± ± são componentes de um tensor esférico (irredutível) de ordem 2. b. Temos do enunciado que

2 2, , (3 ) , ,Q e j m j z r j m jα α≡ = − = . (19) Primeiramente, devemos notar que:

( )2 20

2 2

3516

z rY

−= (20)

ou

( )2 2 0 22

1635

z r Y rπ− = . (21)

........................................................................................................................... Lembre-se: Temos do apêndice A:

( )0 22

5 3cos 116

Y θπ

= − (A.5.7)

Temos também que:

cosz r θ= (22)

6

Substituindo, temos:

20

2 2

2 20

2 2

5 3 116

5 316

zYr

z rYr

π

π

⎛ ⎞= −⎜ ⎟

⎝ ⎠⎛ ⎞−

= ⎜ ⎟⎝ ⎠

(23)

que é a expressão desejada. ........................................................................................................................... Substituindo (20) em (19), temos:

2 02

16, , , ,5

Q e j m j r Y j m jπα α⎛ ⎞

≡ = =⎜ ⎟⎜ ⎟⎝ ⎠

. (24)

Aplicando o Teorema de Wigner-Eckart (3.10.31),

( )( )

' ' ,', ' ' , ; ; ' '

2 1

kk

q

j T jj m T jm jk mq jk j m

j

α αα α =

+ (3.10.31)

temos:

2 022; 0 2;16

5 2 1

j j j jj j r Y jQ e

j

α απ=

+ (25)

ou

2 02

5 2 116 2; 0 2;

jj r Y j Q

e j j j jjα α

π+

= (26)

7

........................................................................................................................... Lembre-se: Teorema de Wigner-Eckart: Os elementos matriciais do operador tensor com respeito aos autoestados de momento angular satisfazem

( )( )

' ' ,', ' ' , ; ; ' '

2 1

kk

q

j T jj m T jm jk mq jk j m

j

α αα α =

+ (3.10.31)

onde o elemento matricial de dupla barra é independente de m e 'm . ........................................................................................................................... Usando o teorema de Wigner-Eckart (3.10.31) novamente na expressão,

( ) ( )2 2 2 2 22 2

8, , ' , , , , ' , ,15

e j m x y j m j e j m r Y Y j m jπα α α α−− = = + = , (25)

........................................................................................................................... Lembre-se:

( )2 2 2 2 22 2

8( )15

x y Y Y rπ + −− = + (26)

........................................................................................................................... temos:

( )2 2 2 22 2 2

8 8, , ' , , ( 2; 2 2; '15(2 1) 15(2 1)

e j m r Y Y j m j e j j j jm j r Y jj jπ πα α α α−+ = =+ +

( )

22

2 2 2 22 2 2

2; 2 2; ' )

8 8, , ' , , 2; 2 2; '15(2 1) 15(2 1)

j j j jm j r Y j

e j m r Y Y j m j e j j j jm j r Y jj j

α α

π πα α α α−

+

+ = = −+ +

8

Substituindo (26)

2 02

5 2 116 2; 0 2;

jj r Y j Q

e j j j jjα α

π+

= (28)

na equação (27), temos finalmente:

( )

( )

( )

2 2 22

2 2

2 2

8, , ' , , 2; 2 2; '15(2 1)

5 2 18, , ' , , 2; 2 2; '15(2 1) 16 2; 0 2;

2; 2 2; '1, , ' , ,2; 0 2;6

e j m x y j m j e j j j jm j r Y jj

je j m x y j m j e j j j jm Q

j e j j j jj

j j j jme j m x y j m j Q

j j j jj

πα α α α

πα απ

α α

− = = −+

+− = = −

+

⎡ ⎤−− = = ⎢ ⎥

⎢ ⎥⎣ ⎦ ........................................................................................................................... Lembre-se: Pode-se perceber que os coeficientes acima, isto é,

2; 2 2; 'j j j jm− (30) e

2; 0 2;j j j jj (31) são os coeficientes de Clebsch-Gordan. Tais coeficientes, como os coeficientes do lado direito da expressão abaixo,

( )

( )

1, 1

11, 02

1, 1

10, 02

s m

s m

s m

s m

= = = + +

⎛ ⎞= = = + − + − +⎜ ⎟⎝ ⎠

= = − = − −

⎛ ⎞= = = + − − − +⎜ ⎟⎝ ⎠

, (32)

9

que são os exemplos mais simples dos Coeficientes de Clebsch-Gordan, são simplesmente os elementos da matriz transformação que conecta a base { }1 2,m m a base { },s m . Um exemplo mais geral, é a transformação unitária que conecta as duas bases:

1 2

1 2 1 2 1 2 1 2 1 2 1 2; ; ; ;m m

j j jm j j m m j j m m j j jm=∑∑ . (33)

...........................................................................................................................

1

Problema 3.29 Um núcleo de spin 3 / 2 situado na origem está sujeito a um campo elétrico não homogêneo externo. A interação de quadrupolo elétrico básico pode ser escrita como:

( )2 2 2

2 2 2int 2 2 2 2

0 0 02 1 x y zeQH S S S

s s x y zφ φ φ⎡ ⎤⎛ ⎞ ⎛ ⎞ ⎛ ⎞∂ ∂ ∂

= + +⎢ ⎥⎜ ⎟ ⎜ ⎟ ⎜ ⎟− ∂ ∂ ∂⎢ ⎥⎝ ⎠ ⎝ ⎠ ⎝ ⎠⎣ ⎦,

onde φ é o potencial eletrostático satisfazendo a equação de Laplace e os eixos coordenados são escolhidos tal que:

2 2 2

0 0 0

0x y y z x zφ φ φ⎛ ⎞ ⎛ ⎞ ⎛ ⎞∂ ∂ ∂

= = =⎜ ⎟ ⎜ ⎟ ⎜ ⎟∂ ∂ ∂ ∂ ∂ ∂⎝ ⎠ ⎝ ⎠ ⎝ ⎠.

Mostre que a energia de interação pode ser escrita como: ( ) ( )2 2 2 23 zA S S B S S+ −− + + ,

e expresse A e B em termos de ( )2 2

0/ xφ∂ ∂ e assim por diante.

Determine os autokets de energia (em termos de m , onde 3/ 2, 1/ 2m = ± ± ) e os autovalores de energia correspondentes. Existe alguma degenerescência? Solução: Na expressão para intH , nós reconhecemos que

{ }( )2 2 21 ,4xS S S S S+ − + −= + + (1)

e

2

{ }( )2 2 21 ,4yS S S S S+ − + −= − + − (2)

com

x yS S iS± = ± (3) e { } ( )2 2, 2 zS S S S+ − = − . (4) ........................................................................................................................... Demonstração da equação (1):

{ }( )2 2 21 ,4xS S S S S+ − + −= + + (5)

Considerando que

x yS S iS± = ± (6) temos:

3

{ }( )

( ) ( ) ( ) ( ){ }

( )( ) ( )( ) ( ) ( ){ }

( ) ( ) { } { } { } { }

2 2 2

2 22

2

2 2 2 2 2 2 2 2

2 2

1 ,4

1 ,4

1 ,4

1 , , , ,4

1 24

x

x x y x y x y x y

x x y x y x y x y x y x y

x x x y y x y x x y y x y x x x y y x y y

x x

S S S S S

S S iS S iS S iS S iS

S S iS S iS S iS S iS S iS S iS

S S iS S iS S i S S iS S iS S i S S S i S S i S S i S S

S S

+ − + −= + +

⎡ ⎤= + + − + + −⎢ ⎥⎣ ⎦

⎡ ⎤= + + + − − + + −⎣ ⎦

⎡ ⎤= + + + + − − + + − + −⎣ ⎦

= ( ) { } { }2 2 2 2

2 2 2 2 2

2 2

2 2 2 , ,

1 2 2 2 24

y x y x y y x

x x y x y x y y x y x x y

x x

i S S S i S S i S S

S S S S S iS S iS S iS S iS S

S S

⎡ ⎤+ + + − +⎣ ⎦

⎡ ⎤= − + + − − + +⎣ ⎦

=

........................................................................................................................... ........................................................................................................................... Demonstração da equação (2):

{ }( )2 2 21 ,4yS S S S S+ − + −= − + − (8)

Considerando que

x yS S iS± = ± (9)

4

Temos que:

{ }( )

( ) ( ) ( ) ( ){ }

( )( ) ( )( ) ( ) ( ){ }

( ) ( ) { } { } { } { }

2 2 2

2 22

2

2 2 2 2 2 2 2 2

2

1 ,4

1 ,4

1 ,4

1 , , , ,4

1

y

y x y x y x y x y

y x y x y x y x y x y x y

y x x y y x y x x y y x y x x x y y x y y

y

S S S S S

S S iS S iS S iS S iS

S S iS S iS S iS S iS S iS S iS

S S iS S iS S i S S iS S iS S i S S S i S S i S S i S S

S

+ − + −= − + −

⎡ ⎤= − + + − − + −⎢ ⎥⎣ ⎦

⎡ ⎤= − + + + − − − + −⎣ ⎦

⎡ ⎤= − + + + + − − + − + − +⎣ ⎦

= − ( ) { } { } { } { }

{ } { }

{ } { }

2 2 2 2

2 2 2 2

2 2 2

2 2 2 2 2

2 2

2 2 , , , ,4

1 2 2 , ,4

1 2 2 , ,4

1 2 2 2 24

x y x x y y x y y x

y x y x x y y x y y x y x x y

y x y x x y y

y x y x y

y y

S i S S S i S S i S S i S S

S S S S S i S S iS S iS S iS S iS S

S S S S S S S

S S S S S

S S

⎡ ⎤+ − + + −⎣ ⎦

⎡ ⎤= − − − + − + − −⎣ ⎦

⎡ ⎤= − − − −⎣ ⎦

⎡ ⎤= − − − −⎣ ⎦

=

........................................................................................................................... ........................................................................................................................... Demonstração da equação (4): { } ( )2 2, 2 zS S S S+ − = − (11)

5

Considerando que

x yS S iS± = ± (12) temos: { } ( )

{ } ( )

2 2

2 2

, 2

, 2

z

x y

S S S S

S S S S

+ −

+ −

= −

= +

(13)

Calculando: { } { }

{ } { } { } { } { }

{ }

{ } ( )

2

2 2

2 2

, ,

, , , , ,

, 2 2

, 2

x y x y

x x x y y x y y

x y x y y x y x x y

x y

S S S iS S iS

S S S S i S S i S S i S S

S S S S iS S iS S iS S iS S

S S S S

+ −

+ −

+ −

+ −

= + −

= − + −

= + − − + +

= +

(14)

ou { } ( )2 2, 2 zS S S S+ − = − (15) ...........................................................................................................................

6

Então

( ) ( )( ) ( )

( ) ( )

2 2 2 22 2 22 2 2 2 2

int 2 2 2 20 0 0

2 2 2 22 2

int 2 2 2 2 20 0 0 0

21 22 1 4 4

1 12 1 4 2

z

z z

S S S SeQH S S S S Ss s x y z

eQH S Ss s x y x y

φ φ φ

φ φ φ φ

+ −

+ −

+ −

⎡ ⎤⎡ ⎤− − −⎛ ⎞ ⎛ ⎞ ⎛ ⎞∂ ∂ ∂⎣ ⎦⎢ ⎥= + + − + +⎜ ⎟ ⎜ ⎟ ⎜ ⎟⎢ ⎥− ∂ ∂ ∂⎝ ⎠ ⎝ ⎠ ⎝ ⎠⎣ ⎦

⎧ ⎫ ⎧⎛ ⎞ ⎛ ⎞ ⎛ ⎞ ⎛ ⎞∂ ∂ ∂ ∂⎪ ⎪ ⎪= − + + +⎨ ⎬ ⎨⎜ ⎟ ⎜ ⎟ ⎜ ⎟ ⎜ ⎟− ∂ ∂ ∂ ∂⎪ ⎪ ⎪⎝ ⎠ ⎝ ⎠ ⎝ ⎠ ⎝ ⎠⎩ ⎭ ⎩( )

22 2 2

20

z zS S Szφ⎡ ⎤⎫ ⎛ ⎞∂⎪ − +⎢ ⎥⎬ ⎜ ⎟∂⎢ ⎥⎪ ⎝ ⎠⎭⎣ ⎦

Usando

2 0φ∇ = , (17) Pode-se escrever

( ) ( )2 2 2 2int 3 zH A S S B S S+ −= − + + (18)

em que

2

2 204 ( 1)

eQAs s z

φ⎛ ⎞∂= ⎜ ⎟− ∂⎝ ⎠

(19)

e

2 2

2 2 20 08 ( 1)

eQBs s x y

φ φ⎧ ⎫⎛ ⎞ ⎛ ⎞∂ ∂⎪ ⎪= −⎨ ⎬⎜ ⎟ ⎜ ⎟− ∂ ∂⎪ ⎪⎝ ⎠ ⎝ ⎠⎩ ⎭. (20)

7

........................................................................................................................... Demonstração: Considerando que

2 0φ∇ = , (21) temos:

2 2 22

2 2 2 0x y zφ φ φφ ∂ ∂ ∂

∇ = + + =∂ ∂ ∂

. (22)

Isolando um dos termos, temos:

2 2 2

2 2 2x y zφ φ φ∂ ∂ ∂+ = −

∂ ∂ ∂. (23)

Substituindo no Hamiltoniano,

( ) ( )2 2 2

2 2 2int 2 2 2 2

0 0 0

12 1 2 z z

eQH S S Ss s x y z

φ φ φ⎡ ⎤⎧ ⎫⎛ ⎞ ⎛ ⎞ ⎛ ⎞∂ ∂ ∂⎪ ⎪= + − +⎢ ⎥⎨ ⎬⎜ ⎟ ⎜ ⎟ ⎜ ⎟− ∂ ∂ ∂⎢ ⎥⎪ ⎪⎝ ⎠ ⎝ ⎠ ⎝ ⎠⎩ ⎭⎣ ⎦, (24)

temos:

8

( ) ( )

( ) ( )

( ) ( )

( )

2 22 2 2

int 2 2 20 0

22 2 2

int 2 20

22 2

int 2 20

2 2int

24 1

24 1

34 1

3

z z

z z

z

z

eQH S S Ss s z z

eQH S S Ss s z

eQH S Ss s z

H A S S

φ φ

φ

φ

⎡ ⎤⎧ ⎫⎛ ⎞ ⎛ ⎞∂ ∂⎪ ⎪= − − +⎢ ⎥⎨ ⎬⎜ ⎟ ⎜ ⎟− ∂ ∂⎢ ⎥⎪ ⎪⎝ ⎠ ⎝ ⎠⎩ ⎭⎣ ⎦

⎛ ⎞∂= − + + ⎜ ⎟− ∂⎝ ⎠

⎛ ⎞∂= − ⎜ ⎟− ∂⎝ ⎠

= −

(25)

com

( )2

2 204 1

eQAs s z

φ⎛ ⎞∂= ⎜ ⎟− ∂⎝ ⎠

(26)

........................................................................................................................... Aplicando intH sobre os estados ,s m , onde 3 / 2s = , temos:

( ) ( )( )( )( )( )

( )( )( ) ( )( )

22 2 2 2 2 2

int

2

2

153 34

1 1 2 , 2

( 1) 1 2 , 2

zAH sm A S S sm B S S sm Am sm sm

B s m s m s m s m s m

B s m s m s m s m s m

+ −= − + + = − +

− + + − − + + + +

+ − − + − − − −

(27)

........................................................................................................................... Lembre-se:

2 2, ( 1) ,S s m s s s m= + (3.5.34a)

9

, ,zS s m m s m= (3.5.34b)

( )( ), 1 , 1S s m s m s m s m+ = − + + + (3.5.40)

( )( ), 1 , 1S s m s m s m s m− = + − + − (3.5.41) ........................................................................................................................... Na base

3 / 2, 1/ 2, 1/ 2, 3 / 2m = − − (28) ........................................................................................................................... Lembre-se: Os possíveis estados são:

, 1,..., 1,m s s s s= − − + − . (29) Para 2 1N s= + estados, considerando 3 / 2s = , temos

32 1 2 1 42

N s= + = + = , (30)

ou seja, 4 estados.

10

Os estados possíveis são:

, 1,..., 1,

3 / 2, 1/ 2, 1/ 2, 3 / 2

m s s s s

m

= − − + −

= − −

(31)

........................................................................................................................... a matriz intH usando (2) pode ser escrita como: 3 / 2, 1/ 2, 1/ 2, 3 / 2m = − −

' 2int

3 2 3 0 0

2 3 3 0 0

0 0 3 2 3

0 0 2 3 3

mm

A B

B AH

A B

B A

⎛ ⎞⎜ ⎟

−⎜ ⎟= ⎜ ⎟

−⎜ ⎟⎜ ⎟⎝ ⎠

' 3 / 2

' 1/ 2

' 1/ 2

' 3 / 2

m

m

m

m

= −

= −

= +

= +

(32)

........................................................................................................................... Demonstração: Vamos agora calcular os termos. Para o primeiro termo, temos ' 3 / 2m m= = − .

( )( )( )( )

( )( )( ) ( )( )

22 2 2

int

2

153 1 1 2 , 24

( 1) 1 2 , 2

AH sm Am sm sm B s m s m s m s m s m

B s m s m s m s m s m

= − + − + + − − + + + +

+ − − + − − − −

11

Multiplicando por 'sm , temos:

( )( )( )( )

( )( )( ) ( )( )

22 2 2

int

2

15' ' 3 ' ' 1 1 2 , 24

' ( 1) 1 2 , 2

Asm H sm sm Am sm sm sm sm B s m s m s m s m s m

sm B s m s m s m s m s m

= − + − + + − − + + + +

+ − − + − − − −

Para ' 3 / 2m m= = − , temos:

22 2

int

2 22

int

22

int

2int

15' ' 3 '4

3 15' 32 4

27 15'4 4

' 3

Asm H sm sm Am sm sm sm

Asm H sm A

Asm H sm A

sm H sm A

= −

⎛ ⎞= − −⎜ ⎟⎝ ⎠

= −

=

(35)

........................................................................................................................... Diagonalizando cada bloco de (32), vemos que

( )1/22 2 2 212 9B Aλ± = ± + (36) são os autovalores de energia para ambas as bases , ' 3 / 2, 1/ 2m m = − − e

, ' 1/ 2,3 / 2m m = . Ou seja,

( )1/22 2 2 212 9B Aλ± = ± + , ' 3 / 2, 1/ 2m m = − − (37)

12

( )1/22 2 2 212 9B Aλ± = ± + , ' 1/ 2,3 / 2m m = (38)

3 / 2, 1/ 2, 1/ 2, 3 / 2m = − −

' 2int

3 2 3 0 0

2 3 3 0 0

0 0 3 2 3

0 0 2 3 3

mm

A B

B AH

A B

B A

⎛ ⎞⎜ ⎟

−⎜ ⎟= ⎜ ⎟

−⎜ ⎟⎜ ⎟⎝ ⎠

' 3 / 2

' 1/ 2

' 1/ 2

' 3 / 2

m

m

m

m

= −

= −

= +

= +

(39)

........................................................................................................................... Demonstração: Diagonalizando a matriz

' 2int

3 2 3

2 3 3mm A B

HB A

⎛ ⎞= ⎜ ⎟⎜ ⎟−⎝ ⎠

, (40)

temos:

( )( )

'int

2 2 2 2

2 2 2 2 2 2 2 2

2 2 2 2 2 2

( ) 0

3 2 33 3 12 0

2 3 3

9 3 3 12 0

9 12 0

mmDet H I

A BDet A A B

B A

A A A B

A B

λ

λλ λ

λ

λ λ λ

λ

− =

⎛ ⎞−= − − − − =⎜ ⎟⎜ ⎟− −⎝ ⎠

− − + + − =

− − =

(41)

13

( )

( )

( )

( )

2 2 2 2

1/22 2

1/22 2 2 2

1/22 2 2 2

9 12

9 12

9 12

12 9

A B

A B

A B

B A

λ

λ

λ

λ±

= +

= ± +

= ± +

= ± +

(42)

........................................................................................................................... Os autoestados

1

2

αα⎛ ⎞⎜ ⎟⎝ ⎠

(43)

podem ser determinados para cada bloco da matriz 2 2× como: 3 / 2, 1/ 2m = − −

' 3 / 2' 1/ 2

mm

= −= −

1 1

2 2

3 2 3

2 3 3

A B

B A

α αλ

α α±

⎛ ⎞⎛ ⎞ ⎛ ⎞=⎜ ⎟⎜ ⎟ ⎜ ⎟⎜ ⎟− ⎝ ⎠ ⎝ ⎠⎝ ⎠

( )1/22 212 9B Aλ± = ± + Bloco I

1/ 2,3 / 2m =

' 1/ 2' 3 / 2

mm

==

1 1

2 2

3 2 3

2 3 3

A B

B A

α αλ

α α±

⎛ ⎞− ⎛ ⎞ ⎛ ⎞=⎜ ⎟⎜ ⎟ ⎜ ⎟⎜ ⎟⎝ ⎠ ⎝ ⎠⎝ ⎠

( )1/22 212 9B Aλ± = ± + Bloco II

14

........................................................................................................................... Bloco I: Vamos agora calcular a razão 2 1/α α para o bloco I:

1 1

2 2

3 2 3

2 3 3

A B

B A

α αλ

α α±

⎛ ⎞⎛ ⎞ ⎛ ⎞=⎜ ⎟⎜ ⎟ ⎜ ⎟⎜ ⎟− ⎝ ⎠ ⎝ ⎠⎝ ⎠

. (46)

Temos duas equações:

1 2 13 2 3A Bα α λ α±+ = (47)

1 2 22 3 3B Aα α λ α±− = (48) Resolvendo a (47), temos: ( ) 1 23 2 3A Bλ α α± − = (49)

( )2

1

32 3

AB

λαα

± −= Base: , ' 3 / 2, 1/ 2m m = − − (50)

...........................................................................................................................

15

........................................................................................................................... Bloco II: Vamos agora calcular a razão 2 1/α α para o bloco II:

1 1

2 2

3 2 3

2 3 3

A B

B A

α αλ

α α±

⎛ ⎞− ⎛ ⎞ ⎛ ⎞=⎜ ⎟⎜ ⎟ ⎜ ⎟⎜ ⎟⎝ ⎠ ⎝ ⎠⎝ ⎠

. (51)

Temos duas equações:

1 2 13 2 3A Bα α λ α±− + = (52)

1 2 22 3 3B Aα α λ α±+ = (53) Resolvendo a (53), temos: ( ) 1 23 2 3A Bλ α α± + = (54)

( )2

1

32 3

AB

λαα

± += Base: , ' 1/ 2,3 / 2m m = (55)

...........................................................................................................................

16

Os autoestados de energia são:

( )2 3 3 / 2, 3 / 2 3 3 / 2, 1/ 2B Aλ λ± ±= − + − − (56)

( )2 3 3 / 2,1/ 2 3 3 / 2,3 / 2B Aλ λ± ±= + + (57) Note que a partir das equações acima existe uma degenerescência dupla, dois estados correspondendo a cada valor de λ (λ+ e λ− ).

1

Problema 3.4 Considere uma partícula de spin 1. Calcule os elementos matriciais de

( )( )z z zS S S+ − (1) e ( )( )x x xS S S+ − . (2) Solução : Os elementos de matriz do operador (1) são escritos de forma geral como:

( )( ), ' ,z z zj m S S S j m+ − (3) Usando

, ,zS j m m j m= , temos:

( )( ) ( )( )( )( ) ( )( )

',

3',

, ' ,

, ' , 1 1z z z m m

z z z m m

j m S S S j m m m m

j m S S S j m m m m

δ

δ

+ − = + −

+ − = + − (4)

Temos que a matriz do operador (1), na base { },j m é diagonal. Para 1j = , todos os elementos desta matriz são nulos. Então,

( )( ) 0z z zS S S+ − = (5) onde 0 é a matriz nula. ........................................................................................................................... Lembre-se: De acordo com a (3.5.33), temos

SLJ += jjm ,...,−= 1,0,1 +−=m ...........................................................................................................................

2

Para calcularmos os elementos da matriz do operador em (2) devemos escrever o operador xS em termos dos operadores S+ e S− , que são os operadores escada:

1 ( )2xS S S+ −= + (6)

........................................................................................................................... Lembre-se:

yx iSSS ±=± ........................................................................................................................... Substituindo, temos:

( ) ( ) ( )

( ) ( )

( ) ( )

−++−−−+−++−

−−++−+−++−+

−+−+

−+−+

−+−+−+

−−++

+++++=−+

⎥⎦

⎤⎢⎣

⎡+−+=−+

⎥⎦⎤

⎢⎣⎡ −+⎥⎦⎤

⎢⎣⎡ +=−+

⎥⎦⎤

⎢⎣⎡ −+⎥⎦⎤

⎢⎣⎡ ++⎥⎦⎤

⎢⎣⎡ +=−+

SSSSSSSSSSS

SSSSSSSSSSSSSS

SSSSSSS

SSSSSSS

SSSSSSSSS

xxx

xxx

xxx

xxx

22)

(81))((

281))((

41

21))((

21

21

21))((

22

33

23

22

(7)

Considerando agora as equações (3.5.39) e (3.5.40),

( )( ){ }1/ 2, 1 , 1S j m j m j m j m± = ± + ±∓ (8)

........................................................................................................................... Lembre-se:

mjmjXmjmjXmj mj

,,','','',' ,

∑∑=

...........................................................................................................................

3

podemos escrever o operador (2) como:

( )( )( )( )( )( )

( )( )( )( )( )( )

( )( )( )( )( )( )

( )( )( )( )( )( )

( )( )( )

1/2 3', 3

1/2 3', 1

1/2 3', 1

1/2 3', 1

1{ 2 3 1 2 181{ 1 1 181{ 2 1 1 2 181{ 1 1 181{ 1 18

m m

m m

m m

m m

j m j m j m j m j m j m

j m j m j m j m j m j m

j m j m j m j m j m j m

j m j m j m j m j m j m

j m j m j m

δ

δ

δ

δ

+

+

+

⎡ ⎤− − + + − − + + − + + +⎣ ⎦

⎡ ⎤− + + + + − − + + +⎣ ⎦

⎡ ⎤+ + − − − − + + − + + +⎣ ⎦

⎡ ⎤+ − + + + − − + + +⎣ ⎦

− + + − + ( )( )( )

( )( )( )( )( )( )

( )( )( )( )( )( )

( )( )( )( )( )( )

( )( ) ( )

1/2 3', 1

1/2 3', 1

1/2 3', 1

1/2 3', 3

21/2

', 1

1

1{ 2 1 1 2 181{ 1 1 181{ 2 3 1 2 1 }8

12

m m

m m

m m

m m

m m

j m j m j m

j m j m j m j m j m j m

j m j m j m j m j m j m

j m j m j m j m j m j m

j m j m j m j

δ

δ

δ

δ

δ

+

+

⎡ ⎤+ + − + +⎣ ⎦

⎡ ⎤− + + − + − − + + − + +⎣ ⎦

⎡ ⎤+ − + − + + + − + +⎣ ⎦

⎡ ⎤+ − − + + − − + + − + −⎣ ⎦

⎡ ⎤− + + + + −⎣ ⎦ ( ){ }1/2', 11 m mm δ −⎡ ⎤+⎣ ⎦ (9)

Para 1j = , ',m m assumem os valores 1,0, 1+ − . Assim, temos de (9):

( )( )

1 0 1

3

' 1' 0' 1

0 0 00 0 00 0 0

m m m

x x x

mS S S m

m

= = =−

=+ − = =

= −

⎛ ⎞⎜ ⎟⎜ ⎟⎜ ⎟⎝ ⎠

1

Problema 3.5 Considere que o Hamiltoniano de um corpo rígido seja

22 231 2

1 2 3

12

KK KHI I I

⎛ ⎞= + +⎜ ⎟

⎝ ⎠,

onde K é o momento angular no sistema de referência do corpo. A partir desta expressão obtenha a expressão de Heisenberg do movimento para K , e então encontre as equações de movimento de Euler no limite correspondente. Solução : A equação de movimento de Heisenberg é a seguinte:

[ ]KHidt

Kd ,1= . (1)

Substituindo K e H na equação acima, temos:

⎥⎦

⎤⎢⎣

⎡++++= 332211

3

23

2

22

1

21 ˆˆˆ,12 eKeKeK

IK

IK

IK

idtKd . (2)

Olhando para a componente 1, temos:

⎥⎦

⎤⎢⎣

⎡+=⎥

⎤⎢⎣

⎡++= 1

3

23

2

22

13

23

2

22

1

211 ,1,12 K

IK

IK

iK

IK

IK

IK

idtdK . (3)

Calculando o primeiro termo, temos:

[ ] [ ] [ ]{ }1222

2122

1222

12

22 ,,1,1,1, KKK

IKKK

IKKK

IK

IK

=+=⎥⎦

⎤⎢⎣

⎡ (4)

Temos também que: [ ] 321 , KiKK = . (5)

2

Portanto, temos que:

{ }322

12

22 ,, KK

IiK

IK

−=⎥⎦

⎤⎢⎣

⎡ (6)

{ }233

13

23 ,, KK

IiK

IK

=⎥⎥⎦

⎢⎢⎣

⎡ (7)

Então, da equação (3),

⎥⎦

⎤⎢⎣

⎡+=⎥

⎤⎢⎣

⎡++= 1

3

23

2

22

13

23

2

22

1

211 ,1,12 K

IK

IK

iK

IK

IK

IK

idtdK ,

temos que:

{ } { }

{ } { }233

322

1

233

322

1

,1,12

,1,12

KKI

KKIdt

dK

KKIi

iKK

Ii

idtdK

+−=

+−=

{ } { }

{ } { }

{ }

{ }

{ }3232

321

3232

231

3232

1

233

322

1

233

322

1

,21

,21

,1121

,1,12

,1,12

KKIIII

dtdK

KKII

IIdt

dK

KKIIdt

dK

KKI

KKIdt

dK

KKI

KKIdt

dK

⎟⎟⎠

⎞⎜⎜⎝

⎛ −=

+−=

⎥⎦

⎤⎢⎣

⎡+−=

+−=

+−=

(8)

3

Portanto, temos as equações para as três componentes:

{ }

{ }

{ }2121

213

1313

132

3232

321

,2

,2

,2

KKIIII

dtdK

KKIIII

dtdK

KKIIII

dtdK

−=

−=

−=

(9)

No limite correspondente, devemos ter:

ijji KKKK = . (10) Sabendo também que:

iii wIK = , (11) temos que:

iii wI

dtdK

= . (12)

Então, a equação de Heisenberg do movimento para K , se reduz a

( ) kjkjii wwIIwI −= ( kji ,, permutação cíclica de 3,2,1 ) (13) que é a equação de Euler de movimento.

1

Problema 3.6 Considere 3 32iG iGiGU e e eα γβ= , onde ( ), ,α β γ são os ângulos Eulerianos. Considerando que U representa uma rotação ( ), ,α β γ , quais são as regras de comutação satisfeitas pelo kG ? Relacione G aos operadores momento angular. Solução: Se U representa uma rotação para os ângulos de Euler α , β e γ , então U deve satisfazer para uma rotação infinitesimal de um ângulo ε a seguinte relação (3.1.7):

1)()()()()( 2 −=− εεεεε zxyyx UUUUU . (1) Temos ainda que,

ε

ε

ε

ε

ε

ε

3

2

1

)(

)(

)(

iGz

iGy

iGx

eU

eU

eU

=

=

=

, (2)

que representam rotações infinitesimais em torno dos eixos x , y e z , respectivamente. Temos ainda para ( )xU ε , ( )yU ε e 2( )zU ε , as seguintes expressões:

1

2

23

2 2 21

1 1

2 2 22

2 2

2 2 42 3

3 3

1 ...2!

1 ...2!

1 ...2!

iG

iG

iG

i GU e iG

i GU e iG

i GU e iG

ε

ε

ε

εε

εε

εε

= = + + +

= = + + +

= = + + +

(3)

2

........................................................................................................................... Lembre-se:

2

exp 1 ...2!XX X≡ + + + (4)

........................................................................................................................... Substituindo as três expressões (3) na expressão(1), temos:

2

2 2 2 2 2 21 2

1 2

2 2 42 2 2 2 2 22 32 1

2 1 3

( ) ( ) ( ) ( ) ( ) 1

1 ... 1 ...2! 2!

1 ... 1 ... 1 ... 12! 2! 2!

x y y x zU U U U U

i G i GiG iG

i Gi G i GiG iG iG

ε ε ε ε ε

ε εε ε

εε εε ε ε

− = −

⎡ ⎤ ⎡ ⎤+ + + + + + −⎢ ⎥ ⎢ ⎥

⎣ ⎦ ⎣ ⎦⎡ ⎤⎡ ⎤ ⎡ ⎤

+ + + + + + = + + + −⎢ ⎥⎢ ⎥ ⎢ ⎥⎣ ⎦ ⎣ ⎦ ⎣ ⎦

Comparando os coeficientes da ordem de 2ε , temos:

[ ]

2 2 2 2 2 2 2 2 2 2 2 22 2 2 2 22 1 1 2

1 2 2 1 3

2 2 2 2 2 2 2 2 2 2 2 22 2 2 2 22 1 1 2

1 2 2 1 3

2 2 2 2 21 2 2 1 3

1 2 2 1 3

2 1 1 2 3

1 2 3

2! 2! 2! 2!

2! 2! 2! 2!

,

i G i G i G i Gi G G i G G iG

i G i G i G i Gi G G i G G iG

i G G i G G iGG G G G iG

G G G G iGG G iG

ε ε ε εε ε ε

ε ε ε εε ε ε

ε ε ε

⎡ ⎤ ⎡ ⎤+ + − + + =⎢ ⎥ ⎢ ⎥

⎣ ⎦ ⎣ ⎦

+ + − − − =

− =− + =

− =

= −

(6)

Temos também que: [ ]3 1 2,G G iG= (7)

3

Temos ainda que: [ ]2 3 1,G G iG= (8) Ou de uma forma geral:

,i j ijk kG G i Gε⎡ ⎤ =⎣ ⎦ . (9) Comparando com as relações de comutação para o momento angular:

,i j ijk kJ J i Jε⎡ ⎤ =⎣ ⎦ (10) Temos que:

ii

JG = . (11)

1

Problema 3.7 Qual é o significado da seguinte equação:

1k kl lU A U R A− = ∑ ,

onde as três componentes de A são matrizes? A partir desta equação mostre que os elementos matriciais km A n se transformam como vetores. Solução:

lA (1) são os operadores não rotacionados, enquanto

UAU k1− , (2)

são os operadores sob rotação. ...........................................................................................................................Lembre-se:

αα )(RD→ O valor esperado de V deve mudar como:

αααααα jj

ijii VRRDVRDV ∑=→ + )()( .

Isto deve ser verdadeiro para um ket arbitrário, portanto

∑=+

jjiji VRRDVRD )()( .

...........................................................................................................................

2

Então

∑=−

llklk ARUAU 1 (3)

é a equação que esta conectando os operadores rotacionados e não-rotacionados após a rotação. Ou seja, os operadores após a rotação são apenas combinações de operadores não-rotacionados. Também, a partir da relação

∑==−

llklkk ARAUAU '1 , (4)

nós obtemos para os elementos matriciais os seguintes valores:

∑=l

lklk nAmRnAm ' . (5)

Isto é o mesmo que uma transformação vetorial,

∑=l

lklk VRV ' , (6)

ou seja,

nAm k , (7) se transforma como um vetor.

1

Problema 3.8 Considere uma seqüência de rotações de Euler representada por:

1/ 2 3 32

( ) / 2 ( ) / 2

1/ 2

( ) / 2 ( ) / 2

( , , ) exp exp exp2 2 2

cos2 2

( , , )cos

2 2

i i

i i

i iiD

e e senD

e sen e

α γ α γ

α γ α γ

σ α σ γσ βα β γ

β β

α β γβ β

− + − −

− +

− −−⎛ ⎞ ⎛ ⎞⎛ ⎞= ⎜ ⎟⎜ ⎟ ⎜ ⎟⎝ ⎠⎝ ⎠ ⎝ ⎠

⎛ ⎞⎛ ⎞ ⎛ ⎞−⎜ ⎟ ⎜ ⎟⎜ ⎟⎝ ⎠ ⎝ ⎠⎜ ⎟=

⎜ ⎟⎛ ⎞ ⎛ ⎞⎜ ⎟⎜ ⎟ ⎜ ⎟

⎝ ⎠ ⎝ ⎠⎝ ⎠

(1)

Devido às propriedades de grupo das rotações, esperamos que esta seqüência de rotações seja equivalente a uma única rotação em torno de algum eixo dado por um ângulo θ . Encontre θ . Solução : A matriz de rotação de um ângulo θ em torno de um eixo arbitrário n̂ é dado pela equação (3.2.45):

( )( )

( )1/ 2

cos2 2 2ˆ.ˆ, exp

2cos

2 2 2

z x y

x y z

in sen in n seni nD n

in n sen in sen

θ θ θσ θθ

θ θ θ

⎛ ⎞⎛ ⎞ ⎛ ⎞ ⎛ ⎞− − −⎜ ⎟ ⎜ ⎟ ⎜ ⎟⎜ ⎟− ⎝ ⎠ ⎝ ⎠ ⎝ ⎠⎛ ⎞ ⎜ ⎟= =⎜ ⎟ ⎜ ⎟⎝ ⎠ ⎛ ⎞ ⎛ ⎞ ⎛ ⎞− + +⎜ ⎟⎜ ⎟ ⎜ ⎟ ⎜ ⎟⎝ ⎠ ⎝ ⎠ ⎝ ⎠⎝ ⎠

. (2)

Desde que );ˆ(2/1 θnD é equivalente a ),,(2/1 γβαD , temos que:

),,();ˆ( 2/12/1 γβαθ TrDnTrD = . (3) Então

( ) )2/()2/cos()2/()2/cos()2/cos(2/cos 2/)(2/)( θθθθββ γαγα seninseninee zzii ++−=+ ++−

(4)

2

Podemos reescrever como:

⎟⎠⎞

⎜⎝⎛

⎟⎠⎞

⎜⎝⎛ +

=⎟⎠⎞

⎜⎝⎛

2cos

2cos2

2cos2 βγαθ , (5)

ou

⎥⎦

⎤⎢⎣

⎡⎟⎠⎞

⎜⎝⎛ +

⎟⎠⎞

⎜⎝⎛= −

2cos

2coscos2 1 γαβθ . (6)

1

Problema 3.9 a. Considere um conjunto puro de sistemas identicamente preparados de spin ½. Suponha que os valores esperados xS e zS e o sinal de yS sejam conhecidos. Mostre como podemos determinar o vetor de estado. Por que não é necessário conhecer a magnitude de yS ? b. Considere um conjunto misto de sistemas de spin ½. Suponha que as médias dos conjuntos [ ]xS , yS⎡ ⎤⎣ ⎦ e [ ]zS sejam todas conhecidas. Mostre como nós podemos construir a matriz densidade 2 2× que caracteriza o conjunto. Solução: a. Um estado geral de um sistema de spin ½ pode ser escrito como (convenientemente normalizado):

−++=−

22

22cos

αα ββαii

esene . 0 2α π≤ ≤ e πβ ≤≤0 .

Figura 1: Vetor unitário n̂ (problema 1.11).

2

Podemos agora calcular xS .

( )

( )

( )

αβ

αβ

ββ

ββββ

ββββ

ββββαα

αα

αα

αααα

αααα

cos2

cos221

2

22cos

2

2cos

222cos

2

22cos

22cos

2

22cos

22cos

2

2222

2222

senS

senS

eesenS

esenesenS

eseneeseneS

eseneeseneSS

x

x

iix

iix

iiii

x

iiii

xx

=

⎟⎠⎞

⎜⎝⎛=

+=

⎟⎠⎞

⎜⎝⎛ +=

⎟⎟⎠

⎞⎜⎜⎝

⎛−++⎟⎟

⎞⎜⎜⎝

⎛++−=

⎟⎟⎠

⎞⎜⎜⎝

⎛−+++−+−+⎟

⎟⎠

⎞⎜⎜⎝

⎛−++==

−−

−−

Isolando α , temos:

βα

βα

senS

senS

x

x

2arccos

2cos

=

=

Ou seja, conhecendo xS , obtemos o valor de α . ........................................................................................................................... Lembre-se:

asenaasen cos22 = ( ) ααα cos2=+− ii ee ...........................................................................................................................

3

De maneira semelhante, podemos obter yS e zS :

βcos2

=zS 2cos zS

arcβ =

αβsensenS y 2

−=

Conhecendo xS e zS podemos obter β e αcos . No entanto, para conhecer o sinal do αsen e então especificar α precisamos conhecer o sinal de yS , mas não a magnitude de yS . Portanto, dado xS , zS , e o sinal de

yS , ψ pode ser determinado. b. Considere a matriz densidade na base zS :

⎟⎟⎠

⎞⎜⎜⎝

⎛=

dcba

ρ .

A média de um conjunto de um operador A é [ ] [ ]ATrA ρ= . Calculando os valores médios:

[ ] ( )cbcdab

Trdcba

TrSx +=⎟⎟⎠

⎞⎜⎜⎝

⎛=⎥

⎤⎢⎣

⎡⎟⎟⎠

⎞⎜⎜⎝

⎛⎟⎟⎠

⎞⎜⎜⎝

⎛=

220110

2

[ ] ( )cbiicidiaib

Tri

idcba

TrS y −=⎟⎟⎠

⎞⎜⎜⎝

⎛−−

=⎥⎦

⎤⎢⎣

⎡⎟⎟⎠

⎞⎜⎜⎝

⎛ −⎟⎟⎠

⎞⎜⎜⎝

⎛=

2200

2

[ ] ( )dadcba

Trdcba

TrSz −=⎟⎟⎠

⎞⎜⎜⎝

⎛−−

=⎥⎦

⎤⎢⎣

⎡⎟⎟⎠

⎞⎜⎜⎝

⎛−⎟⎟

⎞⎜⎜⎝

⎛=

221001

2

4

........................................................................................................................... Lembre-se:

0 11 02xS⎛ ⎞

= ⎜ ⎟⎝ ⎠

002y

iS

i−⎛ ⎞

= ⎜ ⎟⎝ ⎠

1 00 12zS⎛ ⎞

= ⎜ ⎟−⎝ ⎠

........................................................................................................................... Temos ainda a condição de normalização:

1=ρTr ou 1)( =+ da . Temos então quatro equações e quatro incógnitas: [ ] ( )cbSx +=

2

[ ] ( )cbiS y −=

2

[ ] ( )daSz −=

2 ou [ ] [ ] aSdSa xx −+=+= 122 ou [ ]

211

+= xSa

1)( =+ da ou ad −= 1

Portanto, para ρ , temos;

[ ] [ ] [ ][ ]

[ ] [ ][ ] [ ]⎟⎟⎟⎟

⎜⎜⎜⎜

⎥⎦⎤

⎢⎣⎡ −+

−⎥⎦⎤

⎢⎣⎡ +

=z

yx

yxz

SSiS

SiSS

21211

12121

ρ .